Download Neuro- Ophthalmology 2015 Neuro

Document related concepts

Gene therapy of the human retina wikipedia , lookup

Transcript
NeuroOphthalmology
2015
Neuro-Ophthalmology
Made Ridiculously Simple
Program Directors
Michael S Lee MD and Prem S Subramanian MD PhD
In conjunction with the
North American Neuro-Ophthalmology Society
Sands Expo/Venetian
Las Vegas, Nevada
Saturday, Nov. 14, 2015
Presented by:
The American Academy of Ophthalmology
2015 Neuro-Ophthalmology Planning Group
Michael S Lee MD
Program Director
Prem S Subramanian MD PhD
Program Director
Anne S Abel MD
Rudrani Banik MD
Sophia Mihe Chung MD
Sarita B Dave MD
Mays A El-Dairi MD
Andrew G Lee MD
Collin M McClelland MD
Mitchell B Strominger MD
Former Program Directors
2013
Andrew G Lee MD
Michael S Lee MD
2011
2009
2007
Andrew G Lee MD
Karl C Golnik MD
Leah Levi MBBS
Andrew G Lee MD
Karl C Golnik MD
Subspecialty Day Advisory Committee
William F Mieler MD
Associate Secretary
Donald L Budenz MD MPH
Daniel S Durrie MD
Francis S Mah MD
R Michael Siatkowski MD
Nicolas J Volpe MD
Jonathan B Rubenstein MD
Secretary for Annual Meeting
Staff
Melanie R Rafaty CMP, Director, Scientific
Meetings
Ann L’Estrange, Scientific Meetings Specialist
Christa Fernandez, Presenter Coordinator
Debra Rosencrance CMP CAE, Vice
President, Meetings & Exhibits
Patricia Heinicke Jr, Copyeditor
Mark Ong, Designer
Gina Comaduran, Cover Design
©2015 American Academy of Ophthalmology. All rights reserved. No portion may be reproduced without express written consent of the American Academy of Ophthalmology.
ii
2015 Subspecialty Day | Neuro-Ophthalmology
2015 Neuro-Ophthalmology
Subspecialty Day Planning Group
On behalf of the American Academy of Ophthalmology and the North American Neuro-Ophthalmology Society (NANOS), it is our
pleasure to welcome you to Las Vegas and Neuro-Ophthalmology 2015: Neuro-Ophthalmology Made Ridiculously Simple.
Michael S Lee MD
Prem Subramanian MD PhD
Eli Lilly & Company: S
Neuro-ophthalmix: E,P
NASA: C
National Eye Institute: S
Novartis Pharmaceuticals Corp.: S
US Department of Defense: S
Program Director
Program Director
Program Planning Group
Anne S Abel MD
Rudrani Banik MD
Sophia Mihe Chung MD
None
National Eye Institute: S
Eli Lilly & Company: S
Subspecialty Day Planning Group
2015 Subspecialty Day | Neuro-Ophthalmology
No photo
available
No photo
available
Sarita B Dave MD
Andrew G Lee MD
Mitchell B Strominger MD
New York State Department of
Health: S
CredentialProtection: O
None
Mays A El-Dairi MD
Prana Pharmaceuticals: C
Collin M McClelland MD
None
iii
2015 Subspecialty Day | Neuro-Ophthalmology
Neuro-Ophthalmology 2015 Contents
Program Planning Group ii
CME vi
Faculty Listing viii
Program Schedule xiv
Section I:
Vision Loss Made Ridiculously Simple 2, 28
Mini-Talk: How I Use Testing When I Suspect Optic Neuropathy 4
Advocating for Patients 8
Section II:
Your “What” Hurts? Eye Pain and Headache Made Ridiculously Simple 10, 33
Mini-Talk: “Where Do I Begin?” Headache History and Exam Made Ridiculously Simple 12
Section III:
Double Vision and Nystagmus Made Ridiculously Simple 15, 37
Mini-Talk: Orbital Pulleys and Muscles Made Ridiculously Simple 17
Section IV:
Test Interpretation Made Ridiculously Simple 23, 42
Mini-Talk: Bias in Testing—Do I Only Find What I Thought Was There? 25
Faculty Financial Disclosure 47
Presenter Index 50
v
vi
2015 Subspecialty Day | Neuro-Ophthalmology
CME Credit
Academy’s CME Mission Statement
Teaching at a Live Activity
The purpose of the American Academy of Ophthalmology’s
Continuing Medical Education (CME) program is to present ophthalmologists with the highest quality lifelong learning
opportunities that promote improvement and change in physician practices, performance, or competence, thus enabling such
physicians to maintain or improve the competence and professional performance needed to provide the best possible eye care
for their patients.
Teaching instruction courses or delivering a scientific paper or
poster is not an AMA PRA Category 1 Credit™ activity and
should not be included when calculating your total AMA PRA.
Category 1 Credits™: Presenters may claim AMA PRA Category 1 Credits™ through the American Medical Association.
Please contact the AMA to obtain an application form at www.
ama-assn.org.
2015 Neuro-Ophthalmology Subspecialty Day
Meeting Learning Objectives
Upon completion of this activity, participants should be able to:
• Recognize urgent signs and symptoms in the evaluation of
adults with diplopia
• Direct the initial workup of a patient with visual loss from
optic neuropathy
• Distinguish the key findings in the management of eye pain
and associated headache
• Determine when and how to order neuroimaging studies
for ophthalmic conditions
2015 Neuro-Ophthalmology Subspecialty Day
Meeting Target Audience
The intended audience for this program is comprehensive ophthalmologists.
2015 Neuro-Ophthalmology Subspecialty Day
Meeting CME Credit
The American Academy of Ophthalmology is accredited by the
Accreditation Council for Continuing Medical Education to provide continuing medical education for physicians.
The American Academy of Ophthalmology designates this
live activity for a maximum of 7 AMA PRA Category 1 Credits™. Physicians should claim only the credit commensurate with
the extent of their participation in the activity.
ABO Self-Assessment Credit
This activity meets the Self-Assessment CME requirements
defined by the American Board of Ophthalmology (ABO). Please
be advised that the ABO is not an accrediting body for purposes
of any CME program. The ABO does not sponsor this or any
outside activity, and the ABO does not endorse any particular
CME activity. Complete information regarding the ABO SelfAssessment CME Maintenance of Certification requirements is
available at http://abop.org/maintain-certification/part-2-lifelong-learning-self-assessment/cme/.
NOTE: Credit designated as “self-assessment” is AMA PRA
Category 1 Credit™ and is also preapproved by the ABO for the
Maintenance of Certification (MOC) Part II CME requirements.
Scientific Integrity and Disclosure of Financial
Interest
The American Academy of Ophthalmology is committed to
ensuring that all CME information is based on the application
of research findings and the implementation of evidence-based
medicine. It seeks to promote balance, objectivity, and absence of
commercial bias in its content. All persons in a position to control the content of this activity must disclose any and all financial
interests. The Academy has mechanisms in place to resolve all
conflicts of interest prior to an educational activity being delivered to the learners.
The Academy requires all presenters to disclose on their first
slide whether they have any financial interests from the past 12
months. Presenters are required to verbally disclose any financial
interests that specifically pertain to their presentation.
Attendance Verification for CME Reporting
Before processing your requests for CME credit, the Academy
must verify your attendance at Subspecialty Day and/or at AAO
2015. In order to be verified for CME or auditing purposes, you
must either:
• Register in advance, receive materials in the mail, and turn
in the Final Program and/or Subspecialty Day Syllabus
exchange voucher(s) onsite;
• Register in advance and pick up your badge onsite if materials did not arrive before you traveled to the meeting; or
• Register onsite.
CME Credit Reporting
Level 2; Academy Resource Center, Hall B, Booth 2632
Attendees whose attendance has been verified (see above) at
AAO 2015 can claim their CME credit online during the meeting. Registrants will receive an email during the meeting with a
link and instructions on how to claim credit. Onsite, you may
report credits earned during Subspecialty Day and/or AAO 2015
at the CME Credit Reporting booth.
Academy Members: The CME credit reporting receipt is not a
CME transcript. CME transcripts that include AAO 2015 credits entered onsite will be available to Academy members on the
Academy’s website beginning Dec. 10, 2015.
NOTE: CME credits must be reported by Jan. 13, 2016.
After AAO 2015, credits can be claimed at www.aao.org. The
2015 Subspecialty Day | Neuro-Ophthalmology
Academy transcript cannot list individual course attendance. It
will list only the overall credits spent in educational activities at
Subspecialty Day and/or AAO 2015.
Nonmembers: The Academy will provide nonmembers with
verification of credits earned and reported for a single Academysponsored CME activity, but it does not provide CME credit
transcripts. To obtain a printed record of your credits, you must
report your CME credits onsite at the CME Credit Reporting
booths.
Proof of Attendance
The following types of attendance verification will be available
during AAO 2015 and Subspecialty Day for those who need it
for reimbursement or hospital privileges, or for nonmembers
who need it to report CME credit:
• CME credit reporting/proof-of-attendance letters
• Onsite registration receipt
• Instruction course and session verification
Visit www.aao.org for detailed CME reporting information.
CME Credit
vii
viii
2015 Subspecialty Day | Neuro-Ophthalmology
Faculty
Anne S Abel MD
Rudrani Banik MD
Valerie Biousse MD
Wayzata, MN
Neuro-Ophthalmologist
Hennepin County Medical Center,
Minneapolis, MN
New York, NY
Associate Adjunct Surgeon
The New York Eye & Ear Infirmary
Associate Professor of Ophthalmology
New York Medical College
Atlanta, GA
Professor of Ophthalmology and
Neurology
Emory University School of Medicine
No photo
available
Marie D Acierno MD
Baton Rouge, LA
Professor of Ophthalmology
Residency Director of Ophthalmology
LSU Health Sciences Center/Ochsner,
New Orleans
Anthony C Arnold MD
Los Angeles, CA
Professor and Chief
Neuro-Ophthalmology Division
Jules Stein Eye Institute
Jeffrey Bennett MD PhD
Aurora, CO
Professor of Neurology and
Ophthalmology
University of Colorado School of
Medicine
M Tariq Bhatti MD
Durham, NC
Professor of Ophthalmology and
Neurology
Duke University Eye Center and Duke
University Medical Center
Gabrielle R Bonhomme MD
Pittsburgh, PA
Assistant Professor of Ophthalmology
University of Pittsburgh
M.D.
University of Pittsburgh Medical Center
Michael C Brodsky MD
Rochester, MN
Professor of Ophthalmology and
Neurology
Mayo Clinic
Faculty Listing
2015 Subspecialty Day | Neuro-Ophthalmology
ix
No photo
available
John J Chen MD
Sarita B Dave MD
Marc J Dinkin MD
Rochester, MN
Assistant Professor
Mayo Clinic
Jersey City, NJ
Neuro-Ophthalmologist, ECRIP
Investigator
New York Eye and Ear Infirmary of
Mount Sinai
New York, NY
Assistant Professor of Ophthalmology
and Director of NeuroOphthalmology
Weill Cornell Medical College, NY
Presbyterian Hospital
Clinical Affiliate
Memorial Sloan Kettering Cancer Center
Sophia Mihe Chung MD
St Louis, MO
Professor of Ophthalmology, Neurology
& Psychiatry, and Neurosurgery
Saint Louis University School of
Medicine
Joseph L Demer MD PhD
Los Angeles, CA
Leonard Apt Professor of
Ophthalmology
Professor of Neurology
David Geffen Medical School
University of California, Los Angeles
Director of Ocular Motility Laboratory
Stein Eye Institute
University of California, Los Angeles
Mays A El-Dairi MD
Durham, NC
Assistant Professor
Duke Eye Center
Fiona E Costello MD
Calgary, AB, Canada
Associate Professor
Department of Clinical Neurosciences
and Surgery
University of Calgary, Hotchkiss Brain
Institute
Clinician-Scientist
Hotchkiss Brain Institute
Rod Foroozan MD
Kathleen B Digre MD
Salt Lake City, UT
Professor, Neurology and
Ophthalmology
Moran Eye Center
University of Utah
Houston, TX
Associate Professor of Ophthalmology
Baylor College of Medicine
x
Faculty Listing
2015 Subspecialty Day | Neuro-Ophthalmology
Courtney E Francis MD
Gena Heidary MD
Bradley J Katz MD
Seattle, WA
Assistant Professor
Department of Ophthalmology
University of Washington
Cambridge, MA
Director, Pediatric Neuro
Ophthalmology Service
Boston Children’s Hospital
Assistant Professor in Ophthalmology
Harvard Medical School
Salt Lake City, UT
Associate Professor of Ophthalmology
and Neurology
University of Utah
Todd A Goodglick MD
Chevy Chase, MD
Department of Ophthalmology
Georgetown University/Washington
Hospital Center
Washington Eye Physicians
Lanning B Kline MD
Guy V Jirawuthiworavong MD
Torrance, CA
Regional Neuro-Ophthalmologist
Uveitis Consultant & Medical
Retinologist
Southern California Kaiser Permanente
Medical Group
Birmingham, AL
Professor of Ophthalmology
University of Alabama at Birmingham
Gregory S Kosmorsky DO
Highland Heights, OH
Head, Section of Neuro-Ophthalmology
Cleveland Clinic
Lynn K Gordon MD PhD
Los Angeles, CA
Professor of Ophthalmology
Jules Stein Eye Institute, David Geffen
School of Medicine
University of California, Los Angeles
Senior Associate Dean, Diversity Affairs
David Geffen School of Medicine
University of California, Los Angeles
Randy H Kardon MD PhD
Iowa City, IA
Professor of Ophthalmology and
Director of Neuro-Ophthalmology
University of Iowa
Professor of Ophthalmology
Director, Center for Prevention &
Treatment of Visual Loss
Surgery and Research Division
Department of Veterans Affairs
2015 Subspecialty Day | Neuro-Ophthalmology
Faculty Listing
xi
Jacqueline A Leavitt MD
Leah Levi MD
Neil R Miller MD
Rochester, MN
Consultant
Mayo Clinic
Associate Professor of Ophthalmology
Mayo Medical School
La Jolla, CA
Director of Neuro-Ophthalmology
Scripps Clinic Division of
Ophthalmology
Baltimore, MD
Frank B Walsh Professor of NeuroOphthalmology
Wilmer Eye Institute
Johns Hopkins University School of
Medicine
Professor of Ophthalmology, Neurology
and Neurosurgery
Johns Hopkins University School of
Medicine
Andrew G Lee MD
Grant T Liu MD
Houston, TX
Professor of Ophthalmology, Neurology,
and Neurosurgery
Weill Cornell Medical College
Chairman of Ophthalmology
Houston Methodist Hospital, Blanton
Eye Institute
Philadelphia, PA
Neuro-Ophthalmology Service
Division of Ophthalmology
Children’s Hospital of Philadelphia
Division of Neuro-Ophthalmology,
Department of Neurology
Hospital of the University of
Pennsylvania
Michael S Lee MD
Collin M McClelland MD
Minneapolis, MN
Professor, Departments of
Ophthalmology, Neurology and
Neurosurgery
University of Minnesota
Saint Louis, MO
Assistant Professor of NeuroOphthalmology
Department of Ophthalmology
Washington University in St. Louis
Heather Moss MD PhD
Chicago, IL
Assistant Professor of Ophthalmology
and Visual Sciences
Clinical Assistant Professor of Neurology
and Rehabilitation
University of Illinois at Chicago
Nancy J Newman MD
Atlanta, GA
LeoDelle Jolley Professor of
Ophthalmology
Professor of Ophthalmology and
Neurology
Emory University School of Medicine
xii
Faculty Listing
No photo
available
2015 Subspecialty Day | Neuro-Ophthalmology
No photo
available
Jeffrey G Odel MD
John Pula MD
Alfredo A Sadun MD PhD
New York, NY
Professor of Ophthalmology
Columbia University Medical Center
Chicago, IL
Clinician, Northshore University
Healthsystem
Associate Professor
University of Chicago College of
Medicine
Pasadena, CA
F Thornton Chair
Department of Ophthalmology
Doheny Eye Institute
University of California, Los Angeles
Paul H Phillips MD
Little Rock, AR
Professor of Ophthalmology
University of Arkansas for Medical
Sciences
Chief-of-Staff
Department of Ophthalmology
Arkansas Children’s Hospital
Jacinthe Rouleau MD
Montreal, QC, Canada
Clinical Assistant Professor and Chief,
Neuro-Ophthalmology Section
Université de Montréal
Peter J Savino MD
La Jolla, CA
Professor of Neuro-Ophthalmology
Shiley Eye Institute
University of California, San Diego
No photo
available
Stacy L Pineles MD
Janet C Rucker MD
Los Angeles, CA
Assistant Professor of Ophthalmology
University of California, Los Angeles
New York, NY
Associate Professor of Neurology
New York University School of
Medicine
Harold E Shaw Jr MD
Greenville, SC
Practicing Ophthalmologist
Jervey Eye Group, P.A.
Clinical Assistant Professor
University of South Carolina School of
Medicine - Greenville
2015 Subspecialty Day | Neuro-Ophthalmology
Faculty Listing
xiii
No photo
available
S Tonya Stefko MD
Prem S Subramanian MD PhD
Nicholas J Volpe MD
Gibsonia, PA
Assistant Professor of Ophthalmology
and Neurological Surgery
University of Pittsburgh Medical Center
Baltimore, MD
Professor of Ophthalmology, Neurology,
and Neurosurgery
University of Colorado School of
Medicine
Associate Professor of Surgery
Uniformed Services University of the
Health Sciences
Chicago, IL
George and Edwina Tarry Professor of
Ophthalmology and Chairman
Feinberg School of Medicine
Northwestern University
No photo
available
Mitchell B Strominger MD
Boston, MA
Professor of Ophthalmology and
Pediatrics
Tufts University School of Medicine
Kimberly M Winges MD
Gregory P Van Stavern MD
St Louis, MO
Associate Professor
Department of Ophthalmology and
Visual Sciences
Washington University in St. Louis
Portland, OR
Neuro-Ophthalmologist
Veterans Affairs Health System
Assistant Professor of Ophthalmology
and Neurology
Casey Eye Institute
Oregon Health and Sciences University
xiv
Program Schedule
2015 Subspecialty Day | Neuro-Ophthalmology
Neuro-Ophthalmology Subspecialty Day 2015:
Neuro-Ophthalmology Made Ridiculously Simple
In conjunction with the North American Neuro-Ophthalmology Society
(NANOS)
Saturday, Nov. 14
7:00 AM CONTINENTAL BREAKFAST
8:00 AM Welcome and Introductions
Section I: Vision Loss Made Ridiculously Simple
Moderators: Sophia Mihe Chung MD*, Mays A El-Dairi MD*
Panelists: Lynn K Gordon MD PhD*, Jacqueline A Leavitt MD, Grant T Liu MD, Neil R Miller MD*
8:05 AM “I suddenly lost vision in one eye.”
Marie D Acierno MD
2, 28
8:20 AM “I’m slowly losing vision in one of my eyes.”
Guy V Jirawuthiworavong
MD
2, 29
8:35 AM “My doctor says I might have glaucoma.”
Jacinthe Rouleau MD
3, 30
8:50 AM Mini-Talk: How I Use Testing When I Suspect Optic Neuropathy
Alfredo A Sadun MD PhD*
9:00 AM “I lose my vision and then it comes back.”
Jeffrey Bennett MD PhD
6, 31
9:15 AM “I am losing vision in both eyes.”
John J Chen MD
6, 31
9:30 AM “Can my child see?”
Gena Heidary MD*
7, 32
9:40 AM Recap
Prem S Subramanian MD PhD*
9:50 AM Advocating for Patients
Lynn K Gordon MD PhD* 9:55 AM
REFRESHMENT BREAK and AAO 2015 EXHIBITS
Michael S Lee MD*
4
8
Section II: Your “What” Hurts? Eye Pain and Headache Made Ridiculously Simple
Moderators: Andrew G Lee MD*, Anne S Abel MD
Panelists: Kathleen B Digre MD, Lanning B Kline MD, Gregory S Kosmorsky DO, Nicholas J Volpe MD
10:25 AM “Worst headache of my life!”
S Tonya Stefko MD
10, 33
10:40 AM “My head hurts for days on end...”
John Pula MD
10, 33
10:55 AM “My eye just aches all the time.”
Rod Foroozan MD*
11, 34
11:10 AM Mini-Talk: “Where Do I Begin?” Headache History and Exam Made Ridiculously Simple
Lynn K Gordon MD PhD*
11:20 AM “The light!! It hurts my eyes!!”
Bradley J Katz MD*
11:35 AM “It hurts when I talk, and this cough won’t go away.”
Gabrielle R Bonhomme MD 14, 36
11:50 AM Recap
Michael S Lee MD*
11:55 AM LUNCH and AAO 2015 EXHIBITS
* Indicates that the presenter has financial interest.
No asterisk indicates that the presenter has no financial interest.
12
14, 35
2015 Subspecialty Day | Neuro-Ophthalmology
xv
Program Schedule
Section III: Double Vision and Nystagmus Made Ridiculously Simple
Moderators: Mitchell B Strominger MD, Sarita B Dave MD
Panelists: Michael C Brodsky MD, Leah Levi MD, Nancy J Newman MD, Peter J Savino MD
1:10 PM “I see double and my eyelid is in the way.”
Paul H Phillips MD
15, 37
1:25 PM “I see two golf club heads when I putt.”
Courtney E Francis MD
15, 38
1:40 PM “Things are blurry and jumpy when I read.”
Janet C Rucker MD
16, 38
1:55 PM Mini-Talk: Orbital Pulleys and Muscles Made Ridiculously Simple
Joseph L Demer MD PhD*
2:05 PM “Words run together on the TV.”
Kimberly M Winges MD
21, 39
2:20 PM “I see double when I get tired.”
Marc J Dinkin MD
22, 39
2:35 PM “I see double after my cataract surgery.”
Stacy L Pineles MD
22, 41
2:50 PM Recap
Michael S Lee MD*
2:55 PM REFRESHMENT BREAK and AAO 2015 EXHIBITS
Section IV: Test Interpretation Made Ridiculously Simple
Moderators: Rudrani Banik MD*, Collin M McClelland MD
Panelists: Anthony C Arnold MD*, Valerie Biousse MD*, Randy H Kardon MD PhD*, Jeffrey G Odel MD
3:25 PM What Do I Do With This Visual Field?
Gregory P Van Stavern 23, 42
3:40 PM Should I Trust My Exam, or the OCT?
M Tariq Bhatti MD*
23, 43
3:55 PM I Can’t Get Cocaine Drops in My Office!
Heather Moss MD PhD*
24, 44
4:10 PM Mini-Talk: Bias in Testing—Do I Only Find What I Thought Was There?
Harold E Shaw Jr MD*
4:20 PM The MRI Is Abnormal—Now What?
Fiona E Costello MD*
26, 44
4:35 PM Everyone’s ESR Is High—Who Needs a Biopsy?
Todd A Goodglick MD
26, 45
4:50 PM Recap
Prem S Subramanian MD PhD*
4:55 PM Closing Remarks
Prem S Subramanian MD PhD*
4:57 PM
ADJOURN
* Indicates that the presenter has financial interest.
No asterisk indicates that the presenter has no financial interest.
17
25
Case Presentations
2
Section I: Vision Loss
2015 Subspecialty Day | Neuro-Ophthalmology
Section I: Vision Loss Made Ridiculously Simple
“I suddenly lost vision in one eye.”
Marie D Acierno MD
C ase
“I am slowly losing vision in one of
my eyes.”
Guy V Jirawuthiworavong MD
C ase
History and Exam
Twenty-year-old male reports left eye dull pain with eye movements and a constant headache that began one week ago. Several
days later he noticed blurred vision in his left eye. The vision
gradually worsened over 1 to 2 days. He denies any involvement
in the other eye. He has had no recent viral-like illness or associated symptoms. His past medical history is noncontributory. On
examination, his visual acuity is 20/20 O.D. and 20/80 O.S. with
a left relative afferent pupillary defect, dyschromatopsia, and a
left central scotoma. Slitlamp biomicroscopy and IOPs are normal. The funduscopic examination reveals a normal appearing
right optic disc and left optic disc edema. There are no macular
lesions, and the vitreous is clear in either eye.
Clinical Course and Outcome
The patient had an MRI brain/orbit imaging study with gadolinium with fat suppression revealing enhancement of the left optic
nerve. His pain with eye movements began to subside shortly
after clinical presentation. The patient received a 3-day course
of IV steroids at an outpatient infusion center. His vision in the
left eye was 20/30 at 4 weeks after the onset, with subjective
improvement in his color vision. At 6 months, his visual acuity
was 20/20 in each eye with no apparent relative afferent pupillary defect, subtle left optic disc atrophy, and retinal nerve fiber
layer thinning by OCT in the left eye compared to the right eye.
History and Exam
Fifty-three-year-old automotive saleswoman noted unilateral,
progressive loss of vision in the right eye over the last year. Her
visual problems began in her early 20s, when she experienced
flashes of lights and “blank spots” in her vision. Multiple eye
examinations were unrevealing. A few years ago, she noted more
flashes, numerous dots, and a kaleidoscope of colors in her vision
with more significant progression over the past 6 months. She
was discovered to have a visual field defect by a retina specialist
3 years ago, but records were not available. She was uncertain
about a prior diagnosis of retinitis pigmentosa. She complains of
difficulty seeing at night and photophobia.
Past medical history is significant for fibromyalgia, eczema,
anxiety, and depression. Family history is significant for glaucoma in her late father, who “died of arthritis,” and ulcerative
colitis in her son. She denies family members needing a white
cane or a seeing eye dog. Review of systems reveals an occasional
headache and Raynaud phenomenon that she describes as painfully cold fingers. She denies any numbness or weakness. Her
current medications are tramadol, alprazolam, and citalopram.
On examination, patient is a mild hyperope with BCVA of
20/25 O.D. and 20/20 O.S. Color vision is intact. She has a 2+
afferent pupillary defect on the right. Her confrontational fields
reveal a temporal defect in the right eye and a normal field in
the left eye. Her slitlamp exam is unremarkable. On fundus
exam, peripapillary atrophy with a cup-to-disc ratio of 0.3 O.U.
is noted. In the peripheral retina, sectoral pigmentary epithelial
atrophy, contiguous with the optic nerve, is seen, but there is
minimal, if any, bone spiculing. Amsler grid shows an incomplete
pericentral visual field defect temporal to fixation O.D. and no
defects O.S. A visual field is completed.
Clinical Course and Outcome
OCT macula and nerve fiber layer and an ultrawide-field fluorescein angiogram were completed. Extensive blood testing for
infectious and inflammatory conditions did not reveal any contributory findings. Old records were still in the process of being
obtained. The visual field in the right eye showed a “c-shaped”
pericentral zonal visual field defect that corresponded to the
perifoveal zonal retinal thinning of the macula on the OCT.
The left eye visual field had a mild enlarged blind spot, with the
OCT of the NFL being thinner on the left than on the right. The
fluorescein angiogram of the right eye showed zonal / sectoral
window defects extending superiorly and in continuity from the
optic disc, and the left eye had peripapillary retinal pigment epithelial abnormalities with window defects. Watchful observation
2015 Subspecialty Day | Neuro-Ophthalmology
was recommended. Patient’s visual field remained stable, with
sequential 30-2 and 10-2 visual field testing. Patient was assured
of the findings and that her condition was consistent with acute
zonal occult outer retinopathy (AZOOR). She used tinted glasses
to help decrease her photophobia.
“My doctor says I might have
glaucoma.”
Jacinthe Rouleau MD
C ase
History and Exam
A 75-year-old male is referred by his family doctor for suspicion
of glaucoma because his confrontation visual field is abnormal
inferiorly in the left eye.
The patient complains about a vague sensation of blurry
vision in the left eye present for 4 months. He can’t remember
exact onset but it’s now stable. He has no headache or other neurological symptoms. Three months ago an optometrist told him
that his visual field was abnormal O.S. and recommended that
he see an ophthalmologist. However, the patient failed to do so,
discussing his eye problem with his general practitioner instead.
His past medical history is significant for hypertension, hyperlipidemia, and cardiac stenting 4 years ago. His medications
include lisinopril, metoprolol, atorvastatin, aspirin, and clopidogrel. He is a nonsmoker. His family history is positive for a sister
with glaucoma.
On examination, his BCVA is 20/25 O.D. and 20/40 O.S.
IOP is 21 in both eyes, with pachymetry of 537 and 542 microns
O.D. and O.S., respectively. A relative afferent pupillary defect
(RAPD) is present O.S. Ocular motility is full. Anterior segment
is normal except for moderate nuclear sclerosis in both eyes.
Gonioscopy reveals open angles in both eyes. Color vision is
decreased O.S. with HRR plates (O.D. 10/10 and 5.5/10 O.S).
Visual field testing is normal O.D., but there is an inferior altitudinal visual field defect O.S. Examination of the optic nerves
showed asymmetric cup-to-disc ratios. The maculae and peripheral retina are normal.
Clinical Course and Outcome
The cup-to-disc ratio of the right optic nerve is 0.2, while the left
is 0.5 with suspicion of pallor superiorly. OCT shows asymmetric cupping, and the left eye has thinning of the nerve fiber layer
superiorly and temporally.
MRI of brain and orbits with gadolinium, obtained by his
family doctor, was normal 2 weeks previously. Complete blood
count, erythrocytes sedimentation rate, and C-reactive protein
were also normal.
Various causes of optic neuropathy can be mistaken for glaucoma: compressive \ infiltrative, ischemic (both arteritic and nonarteritic), congenital, hereditary, post-traumatic, demyelinating,
or toxic. Some red flags should raise suspicion for nonglaucomatous cupping: pallor of the remaining rim, vertically aligned field
defects or other atypical visual fields defects for glaucoma (central or cecocentral), marked RAPD, asymmetrical loss of color
vision, asymmetric cupping, especially without history of asym-
Section I: Vision Loss
3
metric IOP, visual acuity less than 20/40, and patient younger
than 50 years of age.
Because compressive or infiltrative lesions of the optic nerve
can mimic visual field loss from glaucoma, neuroimaging, preferably MRI of the brain and orbits with gadolinium contrast and
fat suppression, is warranted in any suspicious or atypical presentation.
In our case, some clues raise the suspicion for a nonglaucomatous optic neuropathy. The optic disc demonstrates prominent pallor with less severe excavation and notching than in
glaucoma. Moreover, the severity of the visual field loss inferiorly does not match the degree of the notching superiorly. The
presence of the RAPD and the asymmetry in visual acuity also
raise suspicion, especially with the absence of asymmetric IOP
or pachymetry. In addition, the decreased color vision O.S. is
another red flag for a neuro-ophthalmologic pathology. Alternatively, the presence of structural crowding of the contralateral
disc (disc at risk of 0.2) and systemic hypertension and hyperlipidemia favor the diagnosis of an old nonarteritic anterior
ischemic optic neuropathy (NAION). Other potential risk factors are diabetes, sleep apnea, generalized hypoperfusion, severe
anemia, nocturnal hypotension, and medications (amiodarone
and erectile dysfunction agents). Previous ischemic optic neuropathy may present with nerve fiber bundle field loss and optic
atrophy if seen after the disc swelling resolves. Increased cupping
can be seen as sequelae of both arteritic and nonarteritic forms of
ischemic optic neuropathy, although it is more often seen in the
arteritic form.
In our case, the diagnosis of arteritic anterior ischemic optic
neuropathy is unlikely given the normal blood tests and the
absence of systemic symptoms of giant cell arteritis (headache,
jaw claudication, weight loss, anorexia, or fever). NAION constitutes 95% of all anterior ischemic optic neuropathies and is
the most common cause of acute optic neuropathy in people over
the age of 50. Because of its prevalence, previously undiagnosed
NAION with optic atrophy can be seen on routine ophthalmologic exam. When prior NAION is suspected, efforts should be
made to document the acute initial disc edema by reviewing prior
medical records. In our case, acute optic disc swelling is documented on the fundus pictures taken 3 months ago by his optometrist and confirms our diagnosis.
Other causes of nonglaucomatous cupping include methanol
poisoning or genetic optic neuropathies like Leber hereditary
optic neuropathy (LHON) or dominant optic atrophy (DOA).
These are bilateral optic neuropathies with typically central
visual fields defects with profound optic atrophy. LHON is more
likely to present as sequential bilateral optic neuropathy, while
the methanol poisoning and DOA have relatively symmetric
involvement of the optic nerves.
Additionally, in the presence of altitudinal field defect, a
complete examination of the posterior pole and peripheral retina
is always important to rule out retinal pathology such as retinal
detachment or branch retinal vein or artery occlusion.
4
Section I: Vision Loss
2015 Subspecialty Day | Neuro-Ophthalmology
Mini-Talk: How I Use Testing When I Suspect
Optic Neuropathy
Alfredo A Sadun MD PhD
Neuro-ophthalmology is about getting a good history. This is
not so much an exercise in compulsive detail as it is in asking the
right question, one that enables the clinician to test the hypothesis. Hence, the best history is obtained by knowing the general
principles of neuro-ophthalmology, applying critical aspects
of anatomy and physiology, and having a clear understanding
of the differential diagnosis. This differential diagnosis begins
broadly and proceeds to the narrow based on answers to the
questions posed. In particular, in cases of possible optic neuropathy, the following questions are most important:
1. Does the patient see things as dark (brown or grey) as
opposed to distorted or blurred?
Loss of retinal ganglion cells (RGCs) or their axons in the
optic nerve decreases the sense of brightness. In contradistinction, maculopathies often cause metamorphopsia,
problems with the ocular media, and refractive problems,
blurred vision.
2. Is there a subjective loss of color vision?
Dyschromatopsia is frequently the seminal symptom in
optic neuropathies.
If the answers to questions 1 and 2 are positive, assume that
there is an optic neuropathy. Most optic neuropathies are ischemic, inflammatory, compressive, or metabolic.
3. What is the tempo or cadence of visual loss?
Vascular injuries come on abruptly. Inflammatory conditions are more likely to be subacute. Metabolic and compressive injuries are likely to progress slowly.
4. Is the condition unilateral or bilateral?
Unless there is a chiasmal or postchiasmal lesion, bilateral
conditions speak to metabolic and, less likely, inflammatory conditions.
If the answers to 3 and 4 suggest slow progressive bilateral
loss of vision, then the main differential diagnosis is between
a parasellar mass and metabolic optic neuropathy. But nonorganic vision loss should always be considered, especially in the
absence of signs. Other useful questions include pain, pain with
eye movements, and other, especially neurological, associated
symptoms.
Let us assume, at this stage, that we have a 68-year-old
woman with painless loss of vision characterized first by loss of
color, who sees a dark splotch in the center of her vision bilaterally. The visual loss came on over a period of several months.
But the referring physician did not note abnormalities on fundus
examination. What should we concentrate on in the neuro-ophthalmological examination? Key elements in this examination
are the following:
1.Pupils
Are the reactions sluggish? Is there an afferent pupillary
defect (APD)? In this case, no.
2. Optic disc appearance on fundus examination
Is there subtle evidence of disc edema, papillitis, telangiectasis of peripapillary vessels, or arteriolar attenuation?
Is there nerve fiber layer thinning or thickening, and does
the macula have any abnormalities? None of these findings
were evident.
3. Visual field pattern
Are the central scotomas symmetrical? Is there respect for
the vertical or horizontal raphes? In this case, small central
scotomas crossing both the horizontal and vertical raphes
were seen.
In this patient, the pupils were normal; there was no APD.
And there was no optic disc edema or frank optic atrophy,
though there was perhaps slight temporal pallor. The macular
reflex was normal. These findings are consistent with the diagnoses of parasellar mass, metabolic and non-organic, considered
above. Non-organic vision loss should not be a diagnosis of
exclusion. There are several tests that can help us find deliberate
malingering and even evidence of conversion reaction hysteria.
An intracranial mass, especially in the area of the chiasm, can be
easily found by neuroimaging. However, the visual acuities were
worse than 20/400 O.U., unlikely to be seen in a chiasmal syndrome. Furthermore, the visual fields showed scotomas in both
the nasal and temporal side. Further questioning revealed that
the patient was not currently on any known toxic agent and had
a good diet. How then to proceed with specialized testing?
1. HVF 30-2 vs. 10-2
The 24-2 program is good for glaucoma screening and
monitoring. But the 30-2 gives a better indication of how
the defect may emerge from the blind spot, and the 10-2 is
good for detailing the shape of the central scotoma.
2. Red Amsler grid testing
This is especially useful when the central scotoma is small
and shallow. Testing for both red and contrast in the
central area can be very helpful in toxic / metabolic optic
neuropathies.
3. OCT for macula, retinal nerve fiber layer (RNFL) and
ganglion cell layer (GCL)
The RNFL showed slight thinning temporally O.U. But
OCT is not very sensitive for the RNFL in this quadrant.
More obvious was thickening of the RNFL in the inferior
temporal areas O.U. The macula was clear of any cysts,
edema, or structural alterations. However the GCL was
bilaterally and symmetrically thinned.
4. MRI, etc.
In this case, MRI or other neuroimaging would not be that
helpful. Getting unnecessary imaging opens the Pandora’s
box for both false positives and, more importantly, hyperdiagnosis (real lesions that have no bearing on the patient’s
symptoms). Unwarranted anxiety, additional testing,
biopsies, and even neurosurgical interventions have been
known to follow MRIs that should not have been ordered
insofar as an intracranial lesion was already outside the
confines of the differential diagnosis.
Section I: Vision Loss
2015 Subspecialty Day | Neuro-Ophthalmology
In the aggregate, bilateral GCL thinning, especially in the setting of RNFL thickening inferior temporally, is highly specific
for mitochondrial optic neuropathy. While the papillomacular
bundle is poorly visualized by OCT RNFL, GCL thickness in the
macula, especially in comparison to RNFL thickness, provides
very helpful information in the detection, characterization, and
monitoring of both genetic and acquired forms of mitochondrial
optic neuropathies. This was demonstrated in glaucoma and,
more recently, in both genetic and acquired forms of mitochondrial optic neuropathy.
In the case described above, the patient had been on ethambutol for about 6 months when she first noted vision loss. Her
dosing was too high, especially in light of her compromised
kidney function (creatinine of 1.4). Her ophthalmologist discontinued the ethambutol, but the vision loss progressed such that
by 8 months, her vision fell to counting fingers O.U. It was then
that she presented to us not “currently” on any toxic medication. However, persistent or even progressively worsening vision
after discontinuation of therapy is not uncommon in ethambutol
toxicity, which is an acquired form of mitochondrial optic neuropathy. The patient began to recover at 9 months and returned
to 20/25 after 1-2 years.
This case serves to remind us of the value of history taking in neuro-ophthalmology. History related to her past MAC
infection, ethambutol dosing, and her kidney functions were all
obtained with the knowledge that slowly progressive bilaterally
symmetrical optic neuropathies are often metabolic. Genetic and
acquired causes can often be determined by a judicious history,
aided by the limited use of precise testing modalities. Choosing
to obtain OCT for RNFL and GCL measures proved much more
useful than neuro-imaging.
5
Selected Readings
1. Sadun AA. Mitochondrial optic neuropathies. J Neurol Neurosurg
Psychiatry. 2002; 72(4):423-425.
2. Sadun A. Acquired mitochondrial impairment as a cause of optic
nerve disease. Trans Am Ophthalmol Soc. 1998; 96:881-923.
3. Zoumalan CI, Sadun AA. Optical coherence tomography can monitor reversible nerve-fibre layer changes in a patient with ethambutol-induced optic neuropathy. Br J Ophthalmol. 2007; 91(6):839840.
4. Barboni P, Savini G, Valentino ML, Montagna P, Cortelli P, De
Negri AM, Sadun F, Bianchi S, Longanesi L, Zanini M, de Vivo A,
Carelli V. Retinal nerve fiber layer evaluation by optical coherence
tomography in Leber’s hereditary optic neuropathy. Ophthalmology 2005; 112(1):120-126.
5. Barboni P, Savini G, Cascavilla ML, Caporali L, Milesi J, Borrelli
E, La Morgia C, Valentino ML, Triolo G, Lembo A, Carta A, De
Negri A, Sadun F, Rizzo G, Parisi V, Pierro L, Bianchi Marzoli S,
Zeviani M, Sadun AA, Bandello F, Carelli V. Early macular retinal
ganglion cell loss in dominant optic atrophy: genotype-phenotype
correlation. Am J Ophthalmol. 2014; 158(3):628-636.
6
Section I: Vision Loss
“I lose my vision and then it comes
back.”
Jeffrey Bennett MD PhD
2015 Subspecialty Day | Neuro-Ophthalmology
“I’m losing vision in both eyes.”
John J Chen MD
C ase
C ase
History and Exam
History and Exam
A 57-year-old woman presented with a 1-month history of transient, painless vision loss in the right eye. The episodes lasted
an average of 5-10 minutes, but some lasted longer. The events
typically began with flashing lights and progressed to light perception vision. The episodes occurred sporadically and in rare
instances happened more than once in a single day. There were
no concurrent ophthalmologic or neurologic symptoms. Between
events, the vision in her right eye remained slightly blurred. The
patient’s past medical history was notable for hypertension, and
her current medications included enalapril and estrogen. Family and social history were unremarkable. Review of systems
revealed no fevers, chills, night sweats, temporal artery tenderness, polymyalgia, or jaw claudication.
On examination, visual acuities were 20/30 right eye and
20/20 left eye, with subjective color desaturation in the right eye.
Visual fields were full, and pupils were briskly reactive to light
without an afferent pupillary defect. Motility was normal and
dilated exam showed no evidence of optic disc or retinal abnormalities.
Clinical Course and Outcome
Following the fundus examination, the patient experienced an
episode of vision loss in the right eye. Visual acuity dropped
to light perception in the right eye, and the right pupil dilated
slightly and became sluggishly reactive to light with an associated
right afferent defect. Fundus exam showed constriction of retinal
arterioles with sluggish flow in retinal bed and segmental constriction of retinal veins with “box-carring.”
Subsequent laboratory studies showed a normal complete
blood count, erythrocyte sedimentation rate, C-reactive protein,
and comprehensive metabolic profile. Serology was negative for
antinuclear antigen and anti-neutrophil cytoplasmic antibody.
MRI brain and MR angiography of the neck were normal. Echocardiography showed normal function, and bubble study was
negative.
The patient was treated with verapamil and had no further
events.
A 54-year-old female with a 2-year history of normal-tension
glaucoma was referred for bilateral progressive visual loss. One
year prior to presentation, she developed painless, progressive
vision loss in the left eye that kept progressing despite cataract
surgery. This prompted bilateral trabeculectomies with singledigit IOP outcomes. However, she continued to develop painless,
progressive vision loss in the right eye 6 months later, which led
to a referral to neuro-ophthalmology for further evaluation.
The patient was no longer able to work as a nurse or drive.
Other than the vision loss, she was asymptomatic. Her past
medical history is significant for hyperlipidemia and depression.
Medications include simvastatin and sertraline.
On examination, BCVA was CF O.D. and HM O.S. There
was a 2.1 log unit relative afferent pupillary defect O.S. IOPs
were 1 O.D. and 6 O.S. (mmHg). Slitlamp examination showed
a large elevated bleb superiorly O.U., mild cataract O.D., and
pseudophakia O.S. Fundus examination revealed a cup-to-disc
ratio of 0.9 O.D. and 0.95 O.S.
Clinical Course and Outcome
Goldmann visual fields showed dense central scotoma and nasal
loss, bilaterally. OCT showed severe thinning of the retinal nerve
fiber layer O.U. with mild chorioretinal folds O.D. Detailed fundus examination revealed pallor of the remaining rim O.S.
The patient underwent an MRI that showed a large sellar mass with compression of the chiasm and optic nerves.
She underwent bifrontal craniotomy 1 week later with gross
resection revealing a WHO grade II chordoid meningioma.
She noticed instant improvement in the vision O.D. following
the surgery. Examination 2 months later showed her vision
improved from CF to 20/60 O.D. O.S. remained HM. Visual
fields showed a prominent nasal step consistent with underlying
glaucoma.
2015 Subspecialty Day | Neuro-Ophthalmology
“Can my child see?”
Gena Heidary MD PhD
C ase
History and Exam
A 2-month-old baby girl presented to the neuro-ophthalmology
clinic for unusual eye movements and poor visual behavior. The
parents have noted that she does not look at her mother while
nursing.
Birth history
She was born full term at 38 weeks to a 16-year-old G2P0
mother. The pregnancy was uncomplicated.
Maternal medical history
No known medical illnesses. Prenatal vitamins were used
throughout the pregnancy.
Family history
Noncontributory.
Social history
Mother and father are from El Salvador and currently reside in
the United States. There is no history of consanguinity.
Section I: Vision Loss
7
Exam
Vision: Light averse in each eye. No response to 20/2400 card
on preferential looking testing and unable to elicit response to
the optokinetic nystagmus drum. Cycloplegic refraction was
+1.50 sphere in each eye. Sensorimotor exam was notable for
conjugate, roving eye movements with full ductions. A moderate
angle esotropia of 30 PD was noted. Pupillary examination was
normal, without evidence of a relative afferent pupillary defect.
Anterior segment examination was normal in each eye. A dilated
fundus examination revealed bilateral abnormalities involving
the optic nerves. Foveae were normal.
Clinical Course and Outcome
In the setting of bilateral optic nerve hypoplasia, neuroimaging and a complete endocrinologic workup were pursued. MRI
revealed absence of the septum pellucidum and thinning of the
optic nerves more prominently involving the right optic nerve.
The pituitary appeared normal radiographically. A full endocrinologic evaluation was normal.
The infant began working with a teacher for visual impairment through Early Intervention, with gradual improvement of
vision. Patching of the dominant left eye was initiated to facilitate visual development in the right eye. She was followed at regular intervals to monitor visual development. At the most recent
follow-up at age 4, visual acuity is 20/300 right eye and 20/200
left eye. Strabismus surgery was performed to improve the ocular
alignment, with a marked improvement of her esotropia. She has
been registered with the appropriate social services for children
with legal blindness.
8
Advocating for Patients
2015 Subspecialty Day | Neuro-Ophthalmology
2015 Advocating for Patients
Lynn K Gordon MD PhD
Ophthalmology’s goal in protecting quality patient eye care
remains a key priority for the Academy. All ophthalmologists
should consider their contributions to the following three funds
as (a) part of their costs of doing business and (b) their individual
responsibility in advocating for patients and their profession:
• Surgical Scope Fund (SSF)
• OPHTHPAC® Fund
• State Eye PAC
Your ophthalmologist colleagues serving on Academy committees—the Surgical Scope Fund Committee, the Secretariat for
State Affairs, and the OPHTHPAC Committee—are committing
many hours on your behalf. The Secretariat for State Affairs is
collaborating closely with state ophthalmology society leaders
to protect Surgery by Surgeons at the state level. Meanwhile, the
OPHTHPAC Committee is hard at work identifying congressional advocates in each state to maintain close relationships with
federal legislators in order to advance ophthalmology and patient
causes. Both groups’ ultimate goals are to ensure robust funds
for both the SSF and the OPHTHPAC Fund so that they are able
to (a) protect quality patient eye care, (b) protect ophthalmology
practices from payment cuts, (c) reduce burdensome regulations,
and (d) advance the profession by promoting funding for vision
research and expanded inclusion of ophthalmology in public and
private programs.
These committed ophthalmologists serving on your behalf
have a simple message to convey:
“We also need you”!
• We need you to contribute to each of these three funds.
• We need you to establish relationships with state and federal legislators.
• We need you to help us protect quality patient eye care
and the profession.
Surgical Scope Fund
The Surgical Scope Fund (SSF) provides grants to state ophthalmology societies to support their legislative, regulatory, and
public education efforts to derail optometric surgery proposals
that pose a threat to patient safety, quality of surgical care, and
surgical standards. Since its inception, the Surgery by Surgeons
campaign—in partnership with state ophthalmology societies
and with support from the SSF—has helped 32 state/territorial
ophthalmology societies reject optometric surgery proposals.
As of July 1, 2015, the Secretariat for State Affairs, in collaboration with the California Academy of Eye Physicians and
Surgeons (CAEPS) and the California Medical Society, continues
to battle an onerous optometric surgery scope of practice bill
(SB 622) in the Golden State. The Secretariat has reached out
to all ophthalmology subspecialty society partners to help in
this effort, and several have stepped up to the plate. In addition,
ophthalmology leaders at California academic institutions have
played a critical role by voicing their concerns about the California surgery bill and the impact it would have on quality eye
care for patients. A June 24 op-ed in the San Francisco Examiner
aptly focused on these leaders’ concerns with its headline “Quality surgical eye care ensured through training.” CAEPS has benefitted from contributions to the SSF, having received significant
support from the fund.
Other state ophthalmology societies have also benefitted from
SSF distributions in 2015 and were able to successfully implement patient safety advocacy campaigns to defeat attempts by
optometry to expand its scope of practice to include surgery. The
Texas Ophthalmological Association was successful in its patient
advocacy and public education efforts to defeat three different
optometric-backed surgical scope expansion bills in the Texas
state legislature.
In addition, the Academy supported the Alaska Society of Eye
Physicians and Surgeons in opposing optometric surgery scope
legislation that posed a threat to patient surgical care. If enacted,
the optometric surgery bill would have authorized optometrists
in Alaska to perform surgery with lasers, scalpels, and needles,
and to perform other surgical procedures. The legislation would
also have allowed optometrists to perform all injections except
intravitreal and to prescribe any controlled substances. Thanks
to an effective Surgery by Surgeons advocacy campaign, with
support from the SSF, this legislation died in committee. The
Alaska state legislature adjourned for the year on April 27.
The Academy relies not only on the financial contributions
to the SSF from individual ophthalmologists and their business
practices, but also on the contributions made by ophthalmic
state, subspecialty, and specialized interest societies. Several
subspecialty societies contributed to the Surgical Scope Fund in
2014, and the Academy counts on their contributions in 2015.
OPHTHPAC® Fund
OPHTHPAC is a crucial part of the Academy’s strategy to
protect and advance ophthalmology’s interests in key areas
including physician payments from Medicare as well as protecting ophthalmology from federal scope of practice threats.
Established in 1985, OPHTHPAC is one of the oldest, largest,
and most successful political action committees in the physician
community and is very successful in representing your profession to the U.S. Congress. As one election cycle ends, a new one
starts. OPHTHPAC is always under financial pressure to support
our incumbent friends as well as to make new friends with candidates. These relationships allow us to have a seat at the table
and legislators willing to work on issues important to us and our
patients. Among the significant achievements of OPHTHPAC
are the following:
• Repealed the flawed Sustainable Growth Rate (SGR) formula
• Blocked the unbundling of the Medicare global surgery fee
period
• Removed a provision in fraud and abuse legislation that
targeted eyelid surgery
• Protected your ability to perform in-office ancillary services
Advocating for Patients
2015 Subspecialty Day | Neuro-Ophthalmology
Surgical Scope Fund
OPHTHPAC® Fund
State Eye PAC
To derail optometric surgical scope of practice
initiatives that threaten patient eye safety and
quality of surgical care
Ophthalmology’s interests at the federal level
Support for candidates for State House and
Senate
Political grassroots activities, lobbyists and
media
Campaign contributions, legislative education
Campaign contributions, legislative education
Contributions: Limited to $5,000
Contribution limits vary based on state regulations.
Contributions above $200 are on the public
record.
Contributions are on the public record depending upon state statutes.
Support for candidates for U.S. Congress
No funds may be used for candidates or PACs.
Contributions: Unlimited
Individual, practice, and organization
Contributions are 100% confidential.
• Working to reduce the burdens from Medicare’s existing quality improvement programs such as the Electronic
Health Record Meaningful Use program
• Working in collaboration with subspecialty societies to
preserve access to compounded and repackaged drugs
such as bevacizumab
Leaders of the North American Neuro-Ophthalmology Society (NANOS) are part of the American Academy of Ophthalmology’s Ophthalmic Advocacy Leadership Group (OALG), which
has met every January for the past eight years in the Washington
D.C. area to provide critical input and to discuss and collaborate
on the Academy’s advocacy agenda. The topics discussed at
the 2015 OALG meeting included collaborative efforts on the
IRIS Registry and quality reporting under Medicare. As a 2015
Congressional Advocacy Day (CAD) partner, NANOS ensured
a strong presence of neuro-ophthalmology specialists to support
ophthalmology’s priorities as nearly 400 Eye M.D.s had scheduled CAD visits to members of Congress in conjunction with
the Academy’s 2015 Mid-Year Forum in Washington. NANOS
remains a crucial partner with the Academy in its ongoing federal and state advocacy initiatives.
State Eye PAC
It is also important for all ophthalmologists to support our
respective State Eye PACs because state ophthalmology societies cannot count on the Academy’s SSF alone. The presence of a
strong State Eye PAC providing financial support for campaign
contributions and legislative education to elect ophthalmologyfriendly candidates to the state legislature is also critical. The
Secretariat for State Affairs strategizes with state ophthalmology
societies on target goals for State Eye PAC levels.
ACTION REQUESTED: ADVOCATE FOR YOUR
PATIENTS!!
Academy Surgical Scope Fund contributions are used to support
the infrastructure necessary in state legislative / regulatory battles
and for public education. PAC contributions are necessary at
the state and federal level to help elect officials who will support
the interests of our patients. Contributions to each of these three
funds are necessary and should be considered the costs of doing
business. SSF contributions are completely confidential and may
be made with corporate checks or credit cards, unlike PAC contributions, which must be made by individuals and are subject to
reporting requirements.
Please respond to your Academy colleagues who are volunteering their time on your behalf to serve on the OPHTHPAC*
and Surgical Scope Fund** Committees, as well as your state
ophthalmology society leaders, when they call on you and your
subspecialty society to contribute. Advocate for your patients
now!
*OPHTHPAC Committee
Donald J Cinotti MD (NJ) – Chair
Janet A Betchkal MD (FL)
William S Clifford MD (KS)
Robert A Copeland Jr MD (Washington DC)
Anna Luisa Di Lorenzo MD (MI)
Sidney K Gicheru MD (TX)
Michael L Gilbert MD (WA)
Gary S Hirshfield MD (NY)
Jeff S Maltzman MD (AZ)
Thomas J McPhee MD (AZ)
Lisa Nijm MD JD (IL)
Andrew J Packer MD (CT)
Diana R Shiba MD (CA)
Woodford S Van Meter MD (KY)
John (“Jack”) A Wells III MD (SC)
Ex Officio Members
Daniel J Briceland MD (AZ)
Michael X Repka MD (MD)
Russell Van Gelder MD PhD (WA)
George A Williams MD (MI)
**Surgical Scope Fund Committee
Thomas A Graul MD (NE) – Chair
Arezo Amirikia MD (MI)
Matthew F Appenzeller MD (NC)
Ronald A Braswell MD (MS)
John P Holds MD (MO)
Cecily A Lesko MD FACS (NJ)
William (“Chip”) W Richardson II MD (KY)
David E Vollman MD MBA (MO)
Ex Officio Members
Daniel J Briceland MD (AZ)
Kurt F Heitman MD (SC)
9
10
Section II: Eye Pain and Headache
2015 Subspecialty Day | Neuro-Ophthalmology
Section II: Your “What” Hurts? Eye Pain and
Headache Made Ridiculously Simple
“Worst Headache of My Life!”
“My head hurts for days on end…”
S Tonya Stefko MD
John H Pula MD
C ase
C ase
History and Exam
History and Exam
A 44-year-old real estate professional noticed that his glasses did
not seem to be working well for his left eye. Everything up and
to the left was blurred and lighter in color, he complained, for
about a month. He visited the optometrist, where his refraction
had not changed. He was seen later that week by a retina specialist, who noted 20/32 vision with correction O.U. and an otherwise entirely normal exam. Two weeks later, he had a visual field
test that showed a temporal and superior defect in the left eye,
with nonspecific change in the right. He was referred to neuroophthalmology and given an appointment with a VEP, about 6
weeks in the future.
MRI was ordered by the optometrist and was pending insurance review and approval. Over the next 2 weeks, the patient
felt increasing tired and achy and attributed these symptoms to
stress. He also experienced a dull, daily headache with mild light
sensitivity. The headache initially responded to over-the-counter
ibuprofen treatment. Two days prior to his scheduled MRI, he
experienced sudden, excruciating holocranial headache with
photophobia. He had never before had such a severe headache.
This occurred at work, and he was taken by his colleagues to the
local emergency department for evaluation.
A 37-year-old man with no significant past medical history presents with a chief complaint of “My head hurts for days on end.”
The headache was strictly left-sided with a retrobulbar component. For the last 5 days, multiple headaches have occurred at the
same times of day, each one lasting ~70 minutes, the first always
waking him up at 4:30 a.m. With his headaches, the patient gets
conjunctival injection and lacrimation of the affected side. When
drinking alcohol or smoking a cigarette, the headache is exacerbated. The headaches are so severe that he is unable to sit still
during one and paces the room, or even bangs his head against
a wall to try and alleviate them. Evaluation reveals an anxious
man, with normal acuity and color vision, and normal slitlamp
and fundus exams. Motility and alternate cover test are normal.
There is a 2-mm relative ptosis on the left. He has anisocoria,
with right pupil 8 mm in the dark and 2 mm in the light; left eye
3 mm in dark, 1 mm in light, without afferent pupillary defect.
There is dilation lag of the left eye with no vermiform movements of the pupil.
Clinical Course and Outcome
MRI brain with contrast showed a pituitary mass with hemorrhage. He was admitted for emergent neurosurgical intervention.
He was found to have central hypoadrenalism, hypothyroidism,
and possible diabetes insipidus, and his prolactin was mildly
elevated. He awoke from surgery with markedly improved visual
fields and 20/20 BCVA O.U., 11/11 Ishihara color plates, normal extraocular movement, and small relative afferent pupillary
defect O.S.
Clinical Course and Outcome
Based on exam, it was felt that the patient’s anisocoria was due
to autonomic dysfunction at the affected pupil. Two drops of
1.0% apraclonidine were administered to each eye. After 45
minutes, the right pupil remained unchanged, with constriction
from 8 mm to 2 mm, but the left pupil size changed to 9 mm in
the dark and 8 mm in the light. In addition, after apraclonidine,
the left eyelid palpebral fissure increased and was now 1 mm
more open than the right.
At this point, further history was obtained, with the patient
denying any recent cervical neck manipulation, trauma, or neck
pain. Carotid imaging was urgently obtained nevertheless, but
was negative for arterial dissection or other abnormality. The
following day, 1% hydroxyamphetamine drops were instilled
into each eye, and there was minimal change in either pupil size
after 1 hour. MRI of the brain and orbits, MRI of the neck, and
CT scan of the chest were obtained, and all were unremarkable.
While getting his brain MRI, the patient developed a typical
severe headache and was taken to the emergency room. He was
given 10L of 100% oxygen for 20 minutes, and his headache
resolved. He was then put on a course of oral steroids and verapamil, with sumatriptan as needed. Within 1 week, his headaches improved significantly, although there was no change in
the anisocoria or ptosis.
2015 Subspecialty Day | Neuro-Ophthalmology
Section II: Eye Pain and Headache
11
“My eye just aches all the time.”
Rod Foroozan MD
C ase
History and Exam
Clinical Course and Outcome
A 49-year-old woman noted episodic headaches on the right side
for the past 3 months. She described the headache, which could
involve the top of her head, as a stabbing pain lasting seconds
and occasionally associated with a milder headache in between
these episodes. On occasion she noted the pain over the right eye
as well. She was in good health apart from the headaches. She
had a motor vehicle accident without head trauma 1 year before
the onset of symptoms.
Ophthalmic examination revealed UCVA 20/20 in each eye
with normal slitlamp examination (including gonioscopy) and
funduscopy. MRI of the brain and orbits with contrast and
magnetic resonance angiography of the head and neck without
contrast showed no abnormalities. She was diagnosed with
migraine and started on topiramate. Four weeks later she noted
no improvement in the episodic pain.
Palpation over the right trochlear region revealed no tenderness.
Palpation over the occiput reproduced pain similar to her prior
symptoms that radiated to the top of her head on the right side.
Topiramate was discontinued and trigeminal neuralgia was
considered. She started carbamazepine but had no relief of symptoms after 3 weeks.
An injection of 0.5% bupivacaine in the region of the right
greater occipital nerve, lateral and inferior to the external occipital protuberance, was performed. She reported relief of pain
within minutes of the injection. The pain relief lasted 4 weeks
before a mild discomfort in the same distribution recurred. Two
months later a second injection largely relieved the symptoms,
with some intermittent mild pain occurring once every few
weeks.
12
Section II: Eye Pain and Headache
2015 Subspecialty Day | Neuro-Ophthalmology
Mini-Talk: Where Do I Begin? Headache History and
Exam Made Ridiculously Simple
Lynn K Gordon MD PhD
When patients present to the ophthalmologist with a chief complaint of headache, there are often immediate concerns for the
physician. Is the pain attributable to eye disease? Does the pain
arise from a headache syndrome? Is the symptom of pain evidence for some disease entity that could cause serious morbidity
or even mortality? After all, the eye and periocular structures
have sensory innervation from the trigeminal nerve; therefore
pain in the head can arise from the eye, periorbita, or primary
or secondary headache disorders. How can one quickly but
thoroughly evaluate the patient and form a strategic plan, when
appropriate, for additional diagnostic testing or therapeutic
interventions? This presentation concentrates on simplifying the
headache history and examination.
History
The goal of the history is to narrow the differential diagnosis and
determine whether and what additional testing is indicated. The
following are the typical questions that you should ask when a
patient presents with a headache:
1. Do you have a history of headaches? If yes, then is this
headache similar to those you have had in the past?
It is not typical for a patient to see their ophthalmologist during
their first or worst headache of their life. However, there might
be a circumstance when you would be consulted on this type of
patient in an emergency room. “First or worst” are red flags that
may require urgent evaluation. If the headache is similar to prior
headaches, without a change in frequency or severity, additional
questions will help to define the differential diagnosis.
2. Is there a pattern to the headaches: are they associated with
specific triggers or do they occur at a specific time or times
during the day? Is there a periodicity to the headaches? For
example, have you had weeks or months of regular headaches
in the past? Does the headache interfere with your usual
activities? What is the location and character of the headache?
Does eye movement aggravate the pain?
Migraine headaches are often associated with triggers that can
include menses, food, alcohol, stress, and fatigue. Cluster headaches tend to occur recurrently at specific times of the day and
may occur for one to many weeks followed by a headache-free
interval. Eye movement often increases trochlear headache.
3. Do you have any associated symptoms with the headache?
Specifically ask about nausea or vomiting, phonophobia, photophobia, and/or worsening with activity. These are all symptoms
associated with migraine. Cluster headaches often are associated with nasal congestion or rhinorrhea, unilateral ptosis or lid
edema, unilateral conjunctival injection, miosis, or facial sweating. In addition, patients often are restless during the headaches
and prefer to keep moving, not still. The autonomic symptoms
can also be seen with other classifications of the trigeminal autonomic cephalgias. Is there double vision? Microvascular ocular
motor cranial mononeuropathies are often initially associated
with pain or headache. Does the patient have a fever, rash, stiff
neck, or discomfort with eye movements in association with
the headache? Any infectious or meningeal signs such as fever,
sweats, and/or neck pain worse with flexion must be promptly
evaluated.
4. What is your past and current medical history? Specifically,
is there a history of any cancer, autoimmune disease, or
immunosuppression (medication induced or HIV)? Is it possible
that the patient is pregnant? Is there a history of seizures?
These would all be red flags that may require additional testing.
5. Do you have associated symptoms of weight loss, fevers,
scalp pain, jaw or tongue claudication, and/or episodes of
transient vision loss?
Always remember giant cell arteritis in the differential diagnosis of individuals over the age of 50 with new-onset headache.
Remember, if your suspicion is high for giant cell arteritis you
must start high-dose steroids pending the full evaluation of the
patient.
6. Are there any associated focal neurologic symptoms?
These would all be red flags in the history that would require
additional testing.
7. What medications are you taking?
If the patient is using a migraine prophylactic agent such as topiramate, then failure to respond to the medicine may be the cause
of the headache. Don’t forget, angle-closure glaucoma may occur
in such patients and be misdiagnosed as headache! Overuse of
over-the-counter analgesics may be associated with medication
overuse headache or rebound headache.
Clinical Exam
The history has already helped you decide on the focus of the
examination. However there are still a few remaining details that
will help in determining the next steps.
Diseases of the eye may cause pain that is interpreted by the
patient as a headache. Common diseases that may have a subacute presentation include ocular surface disease such as dry eye
or chronic exposure keratitis. Thyroid eye disease may present
with ocular or orbital discomfort. Severe, acute, intermittent, or
chronic pain may result also from keratitis, intraocular inflammation, angle-closure glaucoma, and orbital inflammatory diseases.
Perform a thorough ocular and orbital examination. Specifically, examine the lids and orbit for signs of ptosis, proptosis,
displacement of the globe, or lid retraction. Remember to palpate
the trochlea to help identify cases of trochlear headache.
Identify the BCVA and perform color vision screening and
visual field testing. Examine the pupil carefully for anisocoria or
for a relative afferent pupillary defect. Perform a motility evalu-
2015 Subspecialty Day | Neuro-Ophthalmology
ation in order to detect subtle signs of cranial nerve involvement.
The slit lamp and dilated evaluation of the fundus will identify
any inflammatory or infectious involvement of the eye or ocular
adnexa and, if present, will identify papilledema.
Next Steps
The careful history and physical examination determine the next
steps. Patients with atypical symptoms, red flags in the history
or examination, and autonomic symptoms generally will require
neuroimaging. The choice of neuroimaging studies is dictated
by the clinical suspicion, the availability of testing modality, and
the expertise of the imaging facility. A noncontrast CT scan is
generally not recommended as an initial study for patients with
isolated headache. Laboratory evaluations may be required
when considering systemic inflammatory diseases. After neuroimaging studies are performed, evaluation of the cerebral spinal
fluid is required when papilledema is observed, when there are
new-onset seizures, or when suspicion exists for meningitis or
encephalitis.
Section II: Eye Pain and Headache
13
14
Section II: Eye Pain and Headache
“The light!! It hurts my eyes!!”
Bradley J Katz MD
C ase
History and Exam
A 45-year-old female presented with a chief complaint of light
sensitivity.
Her past medical history was unremarkable except for occasional “sinus headaches.” She reported no drug allergies and
took no medications other than over-the-counter nonsteroidal
anti-inflammatory drugs. Her symptoms had been present for the
past 9 months and she’d consulted with two other eye doctors,
who’d told her that her eyes were completely normal. Although
she’d been “light sensitive” for most of her life, her symptoms
were now to the point where she often wore sunglasses indoors.
Her light sensitivity was beginning to interfere with her ability
to work at her occupation in data entry. Her boss had refused to
allow her to remove the fluorescent lights above her cubicle.
Her eye examination was entirely normal.
2015 Subspecialty Day | Neuro-Ophthalmology
Past medical history reveals controlled hypertension, benign
prostatic hyperplasia, and hypothyroidism. Of note, he attributes
his ptosis and facial asymmetry to childhood head injury sustained when kicked in the head by a cow. Current medications
include doxazosin, lisinopril, levothyroxine, and finasteride. He
has no known medication allergies. Social history and family history are unrevealing.
On exam, Snellen visual acuity was 20/20 in each eye, without afferent pupillary defect (APD). Color perception by Ishihara plates was 8/11 in the right eye and 11/11 in the left eye.
On dilated ophthalmoscopy, the right disc margin was slightly
blurred, with cup-to-disc of 0.2 in each eye with otherwise normal fundi. External exam revealed 1-mm ptosis on the right and
2 mm of left ptosis, asymmetric facies, and a large scar on the left
cheek. Ocular motility revealed decreased adduction of the right
eye and the alignment measurements below:
6 XT
1 RHT
1 XT
2 XT
30 XT
<1 RHT
6 RHT
6 RHT
Clinical Course and Outcome
Further questioning revealed that this patient had frequent
headaches and that these headaches were often unilateral with a
throbbing quality, increased light sensitivity, occasional sound
sensitivity (“phonophobia”) and occasional nausea. She reported
more than 15 headache days per month. This constellation of
symptoms is consistent with a diagnosis of chronic migraine. She
was using ibuprofen 800 mg once or twice a day, nearly every
day, indicating that she may also be suffering from rebound
headache.
She was prescribed FL-41 tinted spectacles for indoor use and
was scheduled to see a neurologist for prophylactic and acute
treatment of migraine.
4 XT
10 RHT
What is the most likely diagnosis?
1.
2.
3.
4.
5.
Post-concussive headache with decompensated exophoria
Myasthenia gravis
Pupil-sparing aneurysmal third nerve palsy
Temporal arteritis
Complicated migraine
Clinical Course and Outcome
“It hurts when I talk, and this cough
won’t go away.”
Gabrielle R Bonhomme MD
CASE
History and Exam
An 82-year-old male farmer presents with 2 weeks of intractable dry cough, intermittent blurred vision in his right eye, and
transient, binocular, horizontal diplopia only on left gaze. He
also reports dull right neck, jaw, and mouth pain while eating or
yawning. Other than longstanding, asymmetric ptosis, he was in
his usual state of good health prior to symptom onset and denies
illness or upper respiratory infection. Prior to presenting to the
ER for worsening pain symptoms, he saw his dentist and was
told his exam was normal. He was evaluated by his primary care
provider with chest X-ray, which was negative.
Given his ptosis, transient binocular diplopia, blurred vision, and
facial pain, the patient was admitted to the neurology service.
Neuroimaging including brain MRI and MRA revealed normal
intracranial vasculature and mild, diffuse small vessel ischemic
changes attributable to age, and excluded stroke. Orbits were
unremarkable. MRA of the neck excluded carotid dissection
or stenosis. Acetylcholine receptor antibody panel was sent to
evaluate for myasthenia and was normal. ESR was 21, Hb was
11, and CRP was elevated at 1.674 mg/dL. Right temporal artery
biopsy was performed and was positive for temporal arteritis.
The patient was treated with IV methylprednisolone during inpatient stay, with 1 mg/kg prednisone taper directed by rheumatology. His cough, headaches, mouth pain, jaw claudication, and
scalp tenderness quickly resolved on steroids.
2015 Subspecialty Day | Neuro-Ophthalmology
Section III: Double Vision and Nystagmus
15
Section III: Double Vision and Nystagmus Made
Ridiculously Simple
“I see double, and my eyelid is in the
way.”
“I see two golf club heads when I
putt.”
Paul H Phillips MD
Courtney E Francis MD
C ase
C ase
History and Exam
History and Exam
A 60-year-old man had binocular, horizontal, diplopia worse in
right gaze that first occurred 1 week prior to presentation. He
denied decreased vision and ocular pain. His past medical history was remarkable for diabetes and hypertension. Medications
included insulin and lisinopril. He consulted his local eye doctor,
who diagnosed him with an “ischemic cranial nerve palsy.”
Ophthalmologic examination: Visual acuity was 20/20 O.U.
Confrontation visual fields were full O.U. External and slitlamp
examination were normal. Ocular motility examination showed
a complete abduction deficit O.D. Pupils were equal, reactive,
without a relative afferent pupillary defect. Funduscopic examination was normal, with a flat optic disc in both eyes.
Further diagnostic testing and treatment?
A 57-year-old African American man presents to his ophthalmologist with a complaint of progressively worsening diplopia
for the past 6 months. He reports that when he golfs, he gets
double vision as he follows the ball in the air after he drives the
ball. On exam, his acuity is 20/20 in both eyes. Slitlamp exam
and funduscopic exam show mild cataracts but are otherwise
unremarkable. He is noted to have a chin-up position. Exophthalmometry reveals 1 mm of proptosis on the left side. Motility
exam shows a mild supraduction deficit in the left eye, otherwise
full. On alternate cover testing, he has a 12 PD right hypertropia
in primary gaze, increasing to 25 in upgaze and decreasing to 6
in downgaze. The deviation is fairly similar in right and left gazes
and right and left head tilts.
Clinical Course
Clinical Course and Outcome
Two weeks after presentation, the patient complained of right
upper lid ptosis and anisocoria. Physical examination showed
the previous complete right abduction deficit as well as right
upper lid ptosis and mild limitations of adduction, elevation,
and depression. Pupil examination showed anisocoria with a
3-mm pupil O.D. and a 4-mm pupil O.S. with no relative afferent pupillary defect. There was a right pupil dilation lag, and the
amount of anisocoria increased with dim illumination.
A diagnostic test was performed.
The pupil examination showed Horner syndrome O.D. The
combination of CN VI palsy, CN III palsy, and a Horner syndrome O.D. localized the pathology to the right cavernous sinus
and mandated neuroimaging.
An MRI of the brain with gadolinium showed a right cavernous sinus mass consistent with a meningioma.
The patient did not elect to pursue further treatment.
In summary, this is a case of binocular vertical diplopia with
evidence of limited supraduction in one eye with associated
mild proptosis. The differential diagnosis for this case includes
mechanical and paretic causes of limited extraocular movements.
A detailed history and review of systems is important to narrow
down potential etiologies. The patient should be asked about a
history of thyroid dysfunction, prior head or facial trauma, and
prior eye surgery (particularly retinal detachment repair, scleral
buckle placement, or previous retrobulbar block). A history of
prior malignancy is also important to note.
Congenital or traumatic fourth nerve palsies are the most
common cause of acquired vertical deviations. Patients typically
present with a hyperdeviation that worsens in the contralateral
gaze and ipsilateral head tilt; however, longstanding deviations
may show some spread of comitance. Symptoms are typically
worst in activities requiring downgaze, such as reading or walking downstairs. On exam, a contralateral head tilt is often present, with fairly full extraocular movements and varying amounts
of ipsilateral inferior oblique overaction. Vertical fusional amplitudes are often higher than normal with congenital palsies.
Thyroid eye disease (TED) is another frequent cause of
vertical diplopia, as the inferior rectus muscle is the most commonly involved muscle, followed by the medial and superior
rectus muscles. Enlargement of the muscle leads to a restrictive
strabismus and a hypotropia on the affected side. Patients have
decreased symptoms in downgaze and so may adopt a chin-up
position in order to reduce symptoms of diplopia (as opposed to
the head tilt frequently seen in fourth nerve palsies). TED most
commonly occurs in the setting of Graves disease, but patients
may also present with a history of hypothyroidism or no prior
16
Section III: Double Vision and Nystagmus
thyroid dysfunction. Other findings consistent with TED can
be helpful in confirming the diagnosis, which is typically a clinical one. Lid retraction, lid lag, temporal flare, lagophthalmos,
limited abduction and resultant esotropia (due to medial rectus
restriction), and proptosis can all be seen in TED, with lid lag
being particularly specific. Elevation in IOP in upgaze and positive forced ductions can be signs of a restrictive strabismus, but
are not specific for TED. For patients without a clear history of
hyperthyroidism or lack of other findings on exam, orbital imaging can be helpful in ruling out other causes, including orbital
masses and inflammatory diseases such as orbital myositis. A
noncontrast orbital CT shows enlargement of the affected muscles, with sparing of the muscle tendons, with rare involvement
of the lateral rectus muscle. Patients presenting with an exotropia
and a diagnosis of TED should be evaluated for possible concurrent ocular myasthenia gravis, due to the increased association of
the two diseases.
Selected Readings
1. Harrad R. Management of strabismus in thyroid eye disease. Eye
2015; 29:234-237.
2. Peragallo JH, Velez FG, Demer JL, Pineles SL. Postoperative drift in
patients with thyroid ophthalmopathy undergoing unilateral inferior rectus muscle recession. Strabismus. 2013; 21(1):23-28.
3. Volpe NJ, Mirza-George N, Binenbaum G. Surgical management of
vertical ocular misalignment in thyroid eye disease using an adjustable suture technique. J AAPOS. 2012; 16(6):518-522.
4. Bothun ED, Scheurer RA, Harrison AR, Lee MS. Update on thyroid
eye disease and management. Clin Ophthalmol. 2009; 3:543-551.
5. Chen VM, Dagi LR. Ocular misalignment in Graves disease may
mimic that of superior oblique palsy. J Neuroophthalmol. 2008;
28(4):302-304.
“Things are blurry and jumpy when I
read.”
Janet C Rucker MD
C ase
History and Exam
A 29-year-old woman presented to her ophthalmologist reporting difficulty reading due to blurry vision. She explained that
words seemed to “jump up and down” when she tried to read a
book. She had first noticed this several months earlier. She occasionally noticed the jumping while watching television, but it was
more bothersome with reading. She also reported that she felt
less steady while walking than usual. She otherwise felt well and
had no other new visual or neurologic symptoms. She specifically
denied headaches or double vision.
2015 Subspecialty Day | Neuro-Ophthalmology
Her medical history was significant for ovarian cancer that
had been diagnosed and treated 3 years prior. She was statuspost hysterectomy, bilateral salpingo-oopherectomy, and chemotherapy with cisplatin and etoposide. Post-treatment, she
developed gait difficulty with sensory loss in her feet that was
attributed to chemotherapy-induced peripheral neuropathy.
On exam, acuity was 20/30 O.U. Color vision, pupils, visual
fields, and fundus were normal. Eye movement range was full.
Vertical oscillations were seen in primary gaze that increased in
lateral and downgaze and decreased in upgaze. It was characterized as a slow drift upward with a fast corrective jerk movement
downwards.
What would be the next step?
Clinical Course and Outcome
The vertical oscillations seen on examination represented
downbeat nystagmus (DBN). The immediate consideration in
this patient would be to determine if there is any relationship
between the nystagmus and her ovarian cancer history. The
cancer history was several years ago, and she was closely followed by her gynecologic oncologist, who reported the patient’s
cancer to be in remission. Brain metastasis from ovarian cancer is
uncommon but must be considered. Thus, brain MRI with contrast would be the most appropriate initial diagnostic test. Other
structural lesions that might be found on brain MRI and should
also be in the differential diagnosis in a young woman with newonset DBN include a Chiari I malformation (a common cause of
DBN), leptomeningeal enhancement (suggesting carcinoma or
infection in the cerebrospinal fluid), and cerebellar or medullary
demyelination or stroke.
Brain MRI with contrast was normal.
Patient was also sent by the ophthalmologist for a neurologic
consultation for her walking difficulties. In addition to the DBN,
the neurologist reported two other findings: (1) poor sensation
in her feet with reduced ankle reflexes attributed to her known
chemotherapy-induced neuropathy and (2) a wide-based gait
suggestive of ataxia attributed to a cerebellar problem.
Given the lack of an answer for a cerebellar process with
DBN and ataxia, paraneoplastic cerebellar degeneration was
highly suspect. Though isolated idiopathic DBN is quite common, the presence of ataxia and her history of ovarian cancer
were quite concerning for a link between her exam findings and
her cancer history.
Lumbar puncture revealed normal cerebrospinal fluid, including negative cytology. Chest, abdomen, and pelvic CT scans were
normal. Anti-GAD antibodies were negative, and thiamine level
was normal. A paraneoplastic panel revealed anti-Yo antibodies in the serum. She was treated with a course of IVIg. Pelvic
PET CT scan revealed lymph node enlargement suggestive of
recurrent cancer. CT-guided biopsy confirmed recurrent ovarian
cancer.
Section III: Double Vision and Nystagmus
2015 Subspecialty Day | Neuro-Ophthalmology
17
Mini-Talk: Orbital Pulleys and Muscles Made
Ridiculously Simple
Joseph L Demer MD PhD
determined by the relationship between its pulley
and its scleral insertion.
This mini-talk attempts humor in a valiant but probably futile
effort to convince neuro-ophthalmologists that they should
care about other things in the orbit besides nerves. All other
approaches have so far failed, anyway.
FDA Disclosure: Surface coils not approved by FDA were
employed for some imaging studies.
E. Peer pressure: Normal people have stereotypic pulley locations. But not everyone conforms.
F. Orbital layers of the oblique EOMs translate the
rectus pulleys in the coronal plane.
Grant Support: USPHS National Eye Institute EY08313, and
Research to Prevent Blindness. No financial conflict of interest
exists.
I. “I am shocked! Shocked!” Many of our 19th-century
beliefs about the extraocular muscles (EOMs) are
incorrect!
A. EOMs are homogeneous structures that rotate the
globe.
D. EOMs follow straight paths from orbital apex to
scleral insertion.
C. Everyone has the same arrangement of EOMs.
D. Abnormalities of EOMs are limited to underaction
(paralysis or paresis), overaction, innervational
miswiring, or stiffness.
E. All important pathophysiology of EOMs can be
diagnosed by clinical motility examination.
F. Orbital connective tissues and EOM sleeves are not
clinically very important.
1. The orbital layer of the inferior oblique inserts
on the inferior rectus and lateral rectus pulleys.
2. The orbital layer of the superior oblique inserts
on the superior oblique sheath posterior to the
trochlea; the sheath travels through the trochlea
and inserts on the superior rectus pulley.
3. In convergence, rectus pulley array excyclo
rotates, due to orbital layers of oblique EOMs.
G. Coordinated movements of the pulley’s ocular rotations account for Listing’s law of torsion and previously mysterious aspects of ocular kinematics.
H. Pulley composition and structure are highly stereotypic. Pulleys are composed of dense, woven collagen, stiffened by elastin and incorporating smooth
muscles having autonomic innervation.
I. Pulleys reconfigure in situations not conforming to
Listing’s law of ocular torsion.
1. Rectus pulley shirts torsionally with the eye during ocular torsion evoked by head tilting.
2. Rectus pulley array extorts during convergence,
required for stereopsis.
II. Amazing Revelations From 21st-Century Science
A. Surface coil MRI of living human orbits has near
microscopic resolution.
B. Digital imaging of immuno- and histo-chemically
stained whole human orbits permits 3-D reconstruction and correlation with in vivo MRI.
IV. Duplicity of EOMs: Different Parts Do Different
Things
C. Classical physiologic data was equivocal and can be
more satisfactorily reinterpreted in light of recent
anatomical findings.
III. Foolish EOMs don’t know their names. They just pull
along paths defined by connective tissue pulleys.
A. Rectus EOMs do not follow shortest, straight-line
paths from orbital apex to scleral insertion.
B. Connective tissue rings called pulleys are located
between the globe equator and posterior pole that
constrain EOM paths.
C. Pulleys serve as the functional mechanical origins of
the EOMs.
D. Any rectus EOM can have horizontal, vertical, and
torsional actions. Pulling direction of any EOM is
A. Orbital layer
1. Contains 40%-50% of total muscle fibers
2. Inserts on the connective tissue pulley, not on the
globe
3. Translates the pulley along EOM axis
4.Does not rotate the globe
B. Global layer
1. Contains 50%-60% of total muscle fibers
2. Contiguous with the insertional tendon
3. Rotates the globe
C. Global layer compartments
1. Separately controllable neuromuscular units
2. Apply force to different scleral insertion points
along broad EOM tendons
18
Section III: Double Vision and Nystagmus
3. Differential function in inferior rectus, lateral
rectus, medial rectus, and superior oblique (vertical vs. torsional)
2015 Subspecialty Day | Neuro-Ophthalmology
VI. Nothing can protect pulleys from surgical exploitation
by ophthalmologists who are in the know.
A. Conventional strabismus surgical approaches
expose the anterior parts of the pulleys.
B. Suggestion: Handle pulley tissues conservatively,
unless aim is to alter pulleys.
C. Lost muscles retract into their pulleys, where they
can generally be found.
D. Efficacious pulley surgeries have been studied by
MRI.
V. Rebels With a Strabismus Cause
Heterotopy (malpositioning) of rectus pulleys causes
incomitant strabismus.
A. Bad pulleys masquerade as cranial nerve palsies. A
pulley out of place will cause rectus EOMs to pull in
an oblique or torsional direction.
B. Heterotopic pulleys can be diagnosed by appropriate orbital MRI or CT.
C. Some pulleys are born for trouble.
1. Retroequatorial myopexy: Mechanism shown
by MRI; interference of myopexy suture with the
pulley. Stretching of the pulley suspension creates
a mechanical restriction in the field of action of
the operated EOM.
2. Augmented rectus transposition (Foster)
1. Congenital A pattern pulley heterotopy
a. Superior location of lateral relative to medial
rectus pulley in one or both orbits, so that the
medial acts as a relative depression in adduction
b. Or lateral location of inferior relative to superior rectus pulley in one or both orbits, so
that the inferior rectus acts as an abductor in
depression
c. Both kinds of heterotopy will manifest overdepression in adduction and be confused
clinically with “superior oblique overaction”
2. Congenital V pattern pulley heterotopy
b. Produces large shifts in pulley position into
the direction of the paralyzed EOM
c. Benefits from extensive pulley dissection
a. Inferior location of lateral relative to medial
rectus pulley in one or both orbits, so that
the medial rectus acts as a relative elevator in
adduction
1. No scleral suturing is required. Just sew the pulley to the muscle belly.
b. Or medial location of inferior relative to
superior rectus pulley in one or both orbits, so
that the inferior rectus acts as an adductor in
depression
2. Modified approach works as well as conventional fadenoperation for acquired esotropia
with high accommodative convergence-toaccommodation ratio.
3. Modified approach avoids risk of scleral perforation.
c. Both kinds of heterotopy will manifest overelevation in adduction and be confused clinically with “inferior oblique overaction”
1. Sagging eye syndrome: Failure of the LR-SR
band ligament may cause 15%-20% of adult
acquired strabismus in U.S. practices.
a. More effective than full tendon width rectus
transfer for paralytic strabismus
E. Posterior fixation (“fadenoperation”) works by
creating hindrance to posterior pulley shift during
EOM contraction.
F. Conventional surgery works in sagging eye syndrome.
D. Some pulleys have trouble thrust upon them.
a. Age-related distance esotropia (ARDE): Bilaterally symmetrical failure causes symmetrical
lateral rectus sag, converting some lateral rectus force from abduction to infraduction.
b. Asymmetrical lateral rectus sag causes
hypotropia and excyclotropia ipsilateral to
the greater sag. This may resemble superior
oblique palsy, except that excyclotropia is in
the lower eye.
2. Heavy eye syndrome: Large angle esotropia and
ipsilateral hypotropia associated with high axial
myopia. The lateral rectus pulley slips under
the globe, converting the lateral rectus from an
abductor to a depressor.
1. “All-in” medial rectus recession works well for
ARDE.
Hint: Do not be timid. Recess both medial rectus
muscle in nearly every patient, and recess for a
target angle twice the largest ET measured in
any distance gaze position. Check convergence
amplitudes first, consider topical anesthesia.
2. Partial rectus tenotomy under topical anesthesia
works well for cyclovertical strabismus.
Selected Readings (Hidden in Plain Sight All This
Time!)
1. Demer JL, Miller JM, Poukens V, Vinters HV, Glasgow B. Evidence
for fibromuscular pulleys of the recti extraocular muscles. Invest
Ophthalmol Vis Sci. 1995; 36:1125-1136.
2. Porter JD, Poukens V, Baker RS, Demer JL. Structure-function
correlations in the human medial rectus extraocular muscle pulley.
Invest Ophthalmol Vis Sci. 1996; 37:468-472.
2015 Subspecialty Day | Neuro-Ophthalmology
Section III: Double Vision and Nystagmus
19
3. Clark RA, Miller JM, Demer JL. Location and stability of rectus
muscle pulleys: muscle paths as a function of gaze. Invest Ophthalmol Vis Sci. 1997; 38:227-240.
22. Demer JL, Kono R, Wright W. Magnetic resonance imaging of
human extraocular muscles in convergence. J Neurophysiol .2003;
89:2072-2085.
4. Demer JL, Poukens V, Miller JM, Micevych P. Innervation of extraocular pulley smooth muscle in monkey and human. Invest Ophthalmol Vis Sci. 1997; 38:1774-1785.
23. Demer JL, Oh SY, Clark RA, Poukens V. Evidence for a pulley
of the inferior oblique muscle. Invest Ophthalmol Vis Sci. 2003;
44:3856-3865.
5. Clark RA, Miller JM, Rosenbaum AL, Demer JL. Heterotopic rectus muscle pulleys or oblique muscle dysfunction? J AAPOS. 1998;
2:17-25.
24. Demer JL. Ocular kinematics, vergence, and orbital mechanics.
Strabismus 2003; 11:49-57.
6. Demer JL, Clark RA, Miller JM. Role of orbital connective tissue in
the pathogenesis of strabismus. Am Orthoptic J. 1998; 48:56-64.
25. Demer JL. Pivotal role of orbital connective tissues in binocular
alignment and strabismus. The Friedenwald Lecture. Invest Ophthalmol Vis Sci. 2004; 45:729-738.
7. Clark RA, Isenberg SJ, Rosenbaum AL, Demer JL. Posterior fixation sutures: a revised mechanical explanation for the fadenoperation based on rectus extraocular muscle pulleys. Am J Ophthalmol.
1999; 128:702-714.
26. Clark RA, Ariyasu R, Demer JL. Medial rectus pulley posterior
fixation is as effective as scleral posterior fixation for acquired esotropia with a high AC/A ratio. Am J Ophthalmol. 2004; 137:10261033.
8. Demer JL, Miller JM. Orbital imaging in strabismus surgery. In:
Rosenbaum AL and Santiago P., eds. Advanced Strabismus Surgery.
New York: Mosby; 1999:84-98.
27. Pirouzian A, Goldberg RA, Demer JL. Inferior rectus pulley hindrance: orbital imaging mechanism of restrictive hypertropia following lower lid surgery. J AAPOS. 2004; 8:338-344.
9. Demer JL, Clark RA, Miller JM. Heterotopy of extraocular muscle
pulleys causes incomitant strabismus. In: Lennerstrand G, ed.
Advances in Strabismology. Amsterdam: Swets; 1999: 91-94.
28. Clark RA, Ariyasu R, Demer JL. Medial rectus pulley posterior
fixation: a novel technique to augment recession. J AAPOS. 2004;
8:451-456.
10. Demer JL, Oh SY, Poukens V. Evidence for active control of rectus extraocular muscle pulleys. Invest Ophthalmol Vis Sci. 2000;
41(6):1280-1290.
29. Kono R, Poukens V, Demer JL. Superior oblique muscle layers in
monkeys and humans. Invest Ophthalmol Vis Sci. 2004; 46:27902799.
11. Demer JL. Extraocular muscles. In: Jaeger EA and Tasman PR., eds.
Clinical Ophthalmology. Philadelphia: Lippincott Williams and
Wilkins; 2000, vol. 1, ch. 1.
30. Demer JL, Clark RA. Magnetic resonance imaging of human extraocular muscles during static ocular counter-rolling. J Neurophysiol.
2005; 94:3292-3302.
12. Clark RA, Miller JM, Demer JL. Three-dimensional location of
human rectus pulleys by path inflections in secondary gaze positions. Invest Ophthalmol Vis Sci. 2000; 41:3787-3797.
31. Demer JL. Current concepts of mechanical and neural factors in
ocular motility. Curr Opin Neurol. 2006; 19:4-13.
13. Oh SY, Poukens V, Demer JL. Quantitative analysis of extraocular
muscle layers in monkey and human. Invest Ophthalmol Vis Sci.
2001; 42:10-16.
14. Clark RA, Demer JL. Rectus extraocular muscle pulley displacement after surgical transposition and posterior fixation for treatment of paralytic strabismus. Am J Ophthalmol. 2002; 133:119128.
15. Demer JL. The orbital pulley system: a revolution in concepts of
orbital anatomy. Ann NY Acad Sci. 2002; 956:17-32.
16. Oh SY, Clark RA, Velez F, Rosenbaum AL, Demer JL. Incomitant
strabismus associated with instability of rectus pulleys. Invest Ophthalmol Vis Sci. 2002; 43:2169-2178.
17. Kono R, Clark RA, Demer JL. Active pulleys: magnetic resonance
imaging of rectus muscle paths in tertiary gazes. Invest Ophthalmol
Vis Sci. 2002; 43:2179-2188.
18. Kono R, Poukens V, Demer JL. Quantitative analysis of the structure of the human extraocular muscle pulley system. Invest Ophthalmol Vis Sci. 2002; 43:2923-2932.
19. Clark RA, Demer JL. Effect of aging on human rectus extraocular
muscle paths demonstrated by magnetic resonance imaging. Am J
Ophthalmol. 2002; 134:872-878.
20. Demer JL. A 12 year, prospective study of extraocular muscle imaging in complex strabismus. J AAPOS. 2003; 6: 337-47.
21. Miller JM, Demer JL, Poukens V, Pavlowski DS, Nguyen HN,
Rossi EA. Extraocular connective tissue architecture. J Vision.
2003; 2:12-23.
32. Demer JL. Gilles Lecture: Ocular motility in a time of paradigm
shift. Clin Experiment Ophthalmol. 2006; 34:822-826.
33. Demer JL. Mechanics of the orbita. Dev Ophthalmol. 2007;
40:132-157.
34. Lim KH, Poukens V, Demer JL. Fascicular specialization in human
and monkey rectus muscles: evidence for structural independence
of global and orbital layers. Invest Ophthalmol Vis Sci. 2007;
48:3089-3097.
35. Kono R, Okanobu H, Ohtsuki H, Demer JL. Displacement of the
rectus muscle pulleys simulating superior oblique palsy. Japn J
Ophthalmol. 2008; 52:36-43.
36. Demer JL. Inflection in inactive lateral rectus muscle: evidence suggesting focal mechanical effects of connective tissues. Invest Ophthalmol Vis Sci. 2008; 49:4858-4864.
37. Demer JL, Clark RA, Crane BT, Tian JR, Narasimhan A, Karim
S. Functional anatomy of the extraocular muscles during vergence.
Prog Brain Res. 2008; 171:21-28.
38. Clark RA, Demer JL. Posterior inflection of weakened lateral rectus
path: connective tissue factors reduce response to lateral rectus
recession. Am J Ophthalmol. 2009; 147:127-133.
39. Rutar T, Demer JL. “Heavy eye syndrome” in the absence of high
myopia: a connective tissue degeneration in elderly strabismic
patients. J AAPOS. 2009; 13:36-44.
40. da Silva Costa RM, Kung J, Poukens V, Yoo L, Tychsen L, Demer
JL. Intramuscular innervation of primate extraocular muscles:
unique compartmentalization in horizontal recti. Invest Ophthalmol Vis Sci. 2011; 52:2830-2836.
41. Demer JL, Dusyanth A. T2 fast spin echo magnetic resonance imaging of extraocular muscles. J AAPOS. 2011; 15:17-23.
20
Section III: Double Vision and Nystagmus
2015 Subspecialty Day | Neuro-Ophthalmology
42. Demer JL, Clark RA, Kung J. Functional imaging of human extraocular muscles in head tilt dependent hypertropia. Invest Ophthalmol Vis Sci. 2011; 52:3023-3031.
49. Clark RA, Demer JL. Differential lateral rectus compartmental contraction during ocular counter-rolling. Invest Ophthalmol Vis Sci.
2012; 53:2887-2896.
43. Demer JL, Clark RA, da Silva Costa RM, Clark RA, Kung J, Yoo
L. Expanding repertoire in the oculomotor periphery: selective compartmental function in rectus extraocular muscles. Ann NY Acad
Sci. 2011; 1233:8-16.
50. Chaudhuri Z, Demer JL. Sagging eye syndrome: connective tissue involution causes horizontal and vertical strabismus in older
patients. JAMA Ophthalmol. 2013; 131:619-625.
44. Wabulembo G, Demer JL. Long term outcome of medial rectus
recession and pulley posterior fixation in esotropia with high AC/A
ratio. Strabismus 2012; 20:115-120.
45. Chaudhuri Z, Demer JL. Medial rectus recession is as effective as
lateral rectus resection in divergence paralysis esotropia. Arch Ophthalmol. 2012; 130:1280-1284.
46. Pineles SL, Laursen J, Goldberg RA, Demer JL, Velez FG. Function
of transected or avulsed rectus muscles following recovery using an
anterior orbitotomy approach. J AAPOS. 2012; 16:336-341.
47. Clark RA, Demer JL. Functional morphometry of horizontal rectus
extraocular muscles during ocular duction. Invest Ophthalmol Vis
Sci. 2012; 53:7375-7379.
48. Shin A, Yoo L, Chaudhuri Z, Demer JL. Independent passive
mechanical behavior of bovine extraocular muscle compartments.
Invest Ophthalmol Vis Sci. 2012; 53:8414-8423.
51. Demer JL, Clark RA. Differential compartmental function of
medial rectus muscle during conjugate and converged ocular adduction. J Neurophysiol. 2014; 112(4):845-855.
52. Demer JL. The Apt Lecture: Connective tissues reflect different
mechanisms of strabismus over the life span. J AAPOS. 2014;
18:309-315.
53. Demer JL. Compartmentalization of extraocular muscle function.
Eye (Lond.) 2014; 29:157-162.
54. Shin A, Yoo LY, Demer JL. Independent active contraction of
extraocular muscle compartments. Invest Ophthalmol Vis Sci.
2015; 56:199-206.
55. Demer JL, Clark RA. Magnetic resonance imaging demonstrates
compartmental muscle mechanisms of human vertical fusional vergence. J Neurophysiol. 2015; 13:2150-2163.
56. Peragallo JH, Pineles SL, Demer JL. Recent advances clarifying the
etiologies of strabismus. J Neuroophthalmol. 2015; 35:185-193.
2015 Subspecialty Day | Neuro-Ophthalmology
“Words run together on the TV.”
Kimberly M Winges MD
C ase
Clinical History and Exam
A 68-year-old woman was referred by optometry for recurrent
episodes of binocular horizontal diplopia that started 3 years
ago and worsened over the last few months. She described words
running together in the captions on the television screen, resolved
by closing either eye. Diplopia was present only at distance,
never at near, and it was not associated with fatigue or change in
position. She denied new medications, lid drooping, trouble with
speech or ambulation, dizziness, headache, jaw claudication,
transient visual loss, or childhood strabismus. Her past medical
history was remarkable for pseudophakia O.U. and hypothyroid
disease.
Eye exam revealed 20/20 BCVA O.U., normal near stereoacuity (8/9 circles by Titmus test), full confrontation visual fields,
no relative afferent pupillary defect, and no proptosis or ptosis
of either eye. Slitlamp biomicroscopy and dilated exams were
healthy, showing centered IOLs O.U. Extraocular motility testing revealed 5 PD of esotropia in primary gaze, which increased
to 8 PD in right and left gaze. Maddox rod testing did not elicit a
vertical deviation or ocular torsion. She was orthophoric at near.
Ductions were full, with normal saccades, slightly choppy pursuit, and intact vestibulo-ocular reflex testing.
What would you do next?
1. Observe and place Fresnel prism on her glasses
2. Order ESR and CRP and start empiric prednisone
­treatment
3. Order MRI brain w/wo contrast
4. Order TSH and noncontrast CT orbit
Clinical Course and Outcome
In summary, this is a case of adult-onset intermittent, binocular,
horizontal diplopia, with exam revealing a relatively comitant
esotropia at distance, full ductions, and orthophoria at near.
With an otherwise normal eye exam apart from pseudophakia,
the main differential diagnosis includes divergence insufficiency
(DI) or subtle bilateral CN VI palsies. In this patient, the full
ductions and normal saccade velocity made 6th nerve palsy less
likely. Also, there were no other signs or symptoms of increased
intracranial pressure, which can cause bilateral CN VI palsy
due to downward displacement of the brain over the 6th nerves
as they course against the skull base. Furthermore, onset was
insidious and not sudden, unlike the typical CN VI palsy. Other
considerations are broken down esophoria or thyroid eye disease, which is usually accompanied by other suggestive ocular
findings.
While nomenclature may vary, the term “divergence insufficiency” (DI) denotes a comitant esotropia that is worse at
distance than at near, with full ductions. Patients have binocular
horizontal diplopia at distance but not when viewing near targets. True neurologic divergence insufficiency or paralysis is a
supranuclear phenomenon, possibly due to disturbance of an
ill-defined divergence center in the brainstem. It can be associ-
Section III: Double Vision and Nystagmus
21
ated with spinocerebellar ataxia, progressive supranuclear palsy,
brainstem stroke, cerebellar and skull-based lesions, or head
trauma. This phenomenon has also been reported in patients
with increased intracranial pressure, where the distinction
between DI and bilateral CN VI palsies may be difficult.
On follow-up testing, this patient complained of difficulty
reading and running, with oscillopsia and frequent falls. She was
found to have downbeat nystagmus and ataxic, wide-based gait.
Her mother had developed similar symptoms without diagnosis
years prior. Brain MRI showed cerebellar atrophy, and she was
eventually diagnosed with spinocerebellar atrophy.
Workup of DI should be based upon careful attention to any
signs of cranial nerve palsy or historical/exam details suggesting
a neurologic cause (such as ataxia or presence of nystagmus, for
example). When appropriate, neuroimaging can rule out brainstem or other skull base lesions. Lab testing should be tailored
to the history and exam findings that invoke suspicion of other
sources of diplopia, such as myasthenia gravis, thyroid disease,
or temporal arteritis.
When no neurologic cause is suspected or found, DI is considered primary or idiopathic. In some cases of age-related distance
esotropia, mechanical changes in the orbit can be shown on highresolution MRI that cause inferior displacement of the lateral
rectus and breaking of the lateral rectus-superior rectus band.
These involutional changes cause esotropia and hypotropia, or
the sagging eye syndrome, commonly associated with blepharoptosis and cyclovertical strabismus. This more recently recognized
syndrome may be a source of DI in the elderly.
Treatment of DI is similar to that of bilateral CN VI palsies,
including base-out prism correction and/or strabismus surgery. A
trial of 4 PD base-out Fresnel prism worked well for this patient
to alleviate her diplopia in distance spectacles.
References
1. Liu GT, Volpe NJ, Galetta SL. Eye movement disorders: third,
fourth, and sixth nerve palsies and other causes of diplopia and
ocular misalignment. In: Neuro-ophthalmology: Diagnosis and
Management, 2nd ed. New York: Saunders Elsevier; 2010:491-550.
2. Kline LB, Foroozan R. Supranuclear and internuclear gaze pathways. In: Neuro-Ophthalmology Review Manual, 7th ed. Thorofare, NJ: SLACK Inc.; 2013:45-72.
3. Jacobson DJ. Divergence insufficiency revisited: natural history of
idiopathic cases and neurologic associations. Arch Ophthalmol.
2000; 118:1237-1241.
4. Chaudhuri Z, Demer JL. Sagging eye syndrome: connective tissue
involution as a cause of horizontal and vertical strabismus in older
patients. JAMA Ophthalmol. 2013; 131(5):619-625.
5. Mittelman D. Age-related distance esotropia. J AAPOS. 2006;
10(3):212-213.
22
Section III: Double Vision and Nystagmus
“I see double when I get tired.”
Marc J Dinkin MD
C ase
2015 Subspecialty Day | Neuro-Ophthalmology
“I see double after my cataract
surgery.”
Stacy L Pineles MD
C ase
History and Exam
A 45-year-old female photo editor with a medical history of
hypertension and high cholesterol presented with double vision
when tired. Eleven months before presentation, her left eye began
deviating outward intermittently, especially when tired and at the
end of a long work day. This was soon followed by intermittent
binocular vertical diplopia, worse in downgaze and worse when
tired and after a glass of wine. An MRI of brain and orbits was
obtained by her primary medical doctor and was normal.
Patient was diagnosed by an ophthalmologist as experiencing a breakdown of congenital strabismus and offered corrective
surgery. She opted instead to patch one eye and seek another
opinion and presented for neuro-ophthalmic consultation.
She endorsed bouts of left upper eyelid ptosis, lasting a few
weeks intermittently over the last year.
Family history was significant for diabetes in both parents.
She was single and never smoked. She was on no medications
and had an allergy to penicillin.
On examination, visual acuity was 20/15 in right eye and
20/20 in left eye. Pupils were equal in the dark, but in the light,
the left pupil was 1 mm larger. Intraocular pressures, confrontational visual fields, and funduscopy were unremarkable. Extraocular movements appeared full, but cross-cover testing demonstrated an incomitant exotropia worse in upgaze and left gaze.
Fusional amplitude in the vertical plane was 3 PD. Maddox rod
testing indicated a right hypertropia that was worse in upgaze
and to the left, as well as in right head tilt. Saccades were slowed
in the horizontal direction.
There was left ptosis and Hertel exophthalmometry revealed
measurements of 23 cm on the right and 21 cm on the left.
Clinical Course
Cogan lid twitch sign was not detected, but there was significant fatigability with worsening of left ptosis after 1 minute of
upgaze. Enhanced ptosis (curtaining) was present bilaterally. The
remaining general neurological examination was normal, including testing for fatigability of the deltoids and ability to count to
30 in one breath.
After 20 minutes of rest, the palpebral fissure and marginal
reflex distance became equal at 10 and 5 mm, respectively, in
both eyes.
Laboratory testing revealed elevated binding, blocking, and
modulating acetylcholine receptor antibodies: 68.0 nmol/L (normal: 0.05 nmol/L), 45% and 83%, respectively.
MRI of the chest demonstrated a minimally enhancing soft
tissue mass of the thymus measuring 8.4x3.5 cm. Computed
tomography revealed calcification of the mass. A robotic thymectomy was performed, and the mediastinal mass contained both
thymic tissue and amyloid.
The patient was treated with oral steroids for 3 months and
experienced complete resolution of ptosis and diplopia.
History and Exam
A 66-year-old man presents with decreasing vision in both eyes
(left more than right) for about 3 years. He also notices glare at
night. His past ocular history is significant for a history of “lazy
eye” during childhood. He thinks he wore glasses and maybe
patched one eye during childhood to correct his lazy eye. He
does not notice any diplopia at this time. The remainder of his
past medical, surgical, social, and family history was noncontributory. On examination, his BCVA is 20/40 O.D. and 20/60
O.S. His refraction is +2.50 O.U. He has a small esotropia. His
slitlamp examination is normal except for 2+NS cataract O.D.
and 2+NS/2+PSC O.S. His dilated fundus examination is normal.
He underwent uncomplicated cataract surgery in the left eye
and on postoperative Day 1, he began to notice binocular diplopia. At that time, his BCVA was 20/40 O.D. and 20/25 O.S.
What is the most likely etiology of his double vision?
1.
2.
3.
4.
Extraocular muscle contracture after retrobulbar injection
Unmasked esotropia from thyroid eye disease
Fixation switch diplopia
Monocular diplopia due to induced astigmatism
Clinical Course and Outcome
On further examination, the patient was found to have refractive
errors of +2.50 O.D. and +0.50 O.S. His motility examination
demonstrated a comitant esotropia of 6 PD at distance and near.
His ocular rotations were normal. He was able to fuse with 6 PD
base out in front of his left eye. He was treated primarily with
prism glasses and then underwent cataract surgery in his right
eye. He continued to need the prism but could be weaned down
from 6 PD to 2 PD to no prism over a period of a few months. As
he began to fixate with his right eye, the diplopia resolved despite
a residual esotropia of 4 PD.
The differential diagnosis for binocular diplopia after cataract
surgery typically includes anesthetic myotoxicity from a retrobulbar injection, pre-existing strabismus masked by the cataract
(due to thyroid eye disease, myasthenia gravis, other forms of
strabismus), central disruption of fusion, optic aberrations, or
disorders unique to a history of amblyopia such as elimination
of suppression in the amblyopic eye due to a long-standing cataract, a change in the angle of strabismus leading to a drift outside
of the suppression scotoma, or fixation switch diplopia. This
patient was diagnosed with fixation switch diplopia because it
was clear that his diplopia began after his fixation was switched
from the right eye to the left eye. In this case, the patient’s dominant eye (O.D.) had poorer vision after the left eye underwent
cataract surgery and was corrected to 20/25. Once the right eye
was corrected to 20/20, the patient was able to regain fixation
with the dominant eye and his diplopia resolved as his left eye
vision was moved back into his appropriate facultative suppression scotoma.
Section IV: Test Interpretation
2015 Subspecialty Day | Neuro-Ophthalmology
23
Section IV: Test Interpretation Made
Ridiculously Simple
What Do I Do With This Visual Field?
Should I Trust My Exam, or the OCT?
Gregory P Van Stavern MD
M Tariq Bhatti MD
C ase
C ase
History and Exam
History and Exam
A 55-year-old woman presented to a general ophthalmologist
with a 1-year history of blurred vision in both eyes. She had a
history of hypertension, which was well controlled, and was
otherwise healthy. Family history was notable for a sister with
glaucoma but was otherwise unremarkable. She did not smoke
and only drank alcohol socially. She was using amlodipine for
blood pressure control and zolpidem for sleep as needed. She had
noticed more difficulty with reading and driving, particularly
with left turns and left lane changes.
On initial examination, BCVA was 20/30 O.D., 20/25 O.S.,
attributed to mild cataracts. Pupils were normal, with no relative afferent pupillary defect noted. IOP was 19 O.D., 20 O.S.
Fundus examination showed flat optic discs, cup-to-disc 0.5
O.D., 0.45 O.S., with some thinning of the temporal rim. The
remainder of the examination was unremarkable. Humphrey
visual fields were performed. These were interpreted as showing
an inferior altitudinal defect in the right eye and a superior arcuate defect in the left eye. The working diagnosis at that time was
“normal-tension glaucoma vs. previous nonarteritic ischemic
optic neuropathy (NAION).” The patient missed a follow-up
appointment and returned nearly 9 months later. Visual acuity
was now 20/40 O.U. Her fundus examination showed possible
neuroretinal rim pallor O.U. Repeat Humphrey visual fields
showed worsened field loss.
A previously healthy 24-year-old woman experienced sudden
loss of vision of the left eye while jogging on a treadmill. Within
45 minutes the vision improved but did not return to normal. She
denied eye pain, headache, or any other systemic or neurological symptoms. The only medication she was taking was ethinyl
estradiol/norgestrel (oral contraceptive pills). She did not smoke
and had only an occasional alcoholic beverage. Family history
was unremarkable.
The patient presented to a local emergency room and was
evaluated by an ophthalmologist and admitted to the hospital for
management of the visual loss. Patient reports that she had blood
work, cranial MRI, and transesophageal echocardiogram that
were all reported to be negative or normal. She was discharged
from the hospital after 3 days and referred to a local retina specialist. On her initial follow-up examination, visual acuity was
noted to be 20/20 O.D. and hand motions O.S. An intravenous
fluorescein angiogram was interpreted as showing choroidal
ischemia O.S. Approximately 3 months after the onset of visual
loss, vision was 5/200 O.S. with superior optic nerve pallor,
resulting in a referral to the neuro-ophthalmology service.
Clinical Course and Outcome
The patient was referred to neuro-ophthalmology. The examination was unchanged, but the field defect raised concern for a
chiasmal lesion. MRI of the brain and orbits with and without
gadolinium was performed. This demonstrated a large, homogeneously enhancing mass compressing the optic chiasm and the
right intracranial optic nerve, most compatible with suprasellar
meningioma. She underwent surgical resection of the tumor and
did well postoperatively, with partial recovery of field and visual
acuity.
Clinical Course and Outcome
On the day of the neuro-ophthalmic visit, the patient stated that
the visual changes of the left eye were stable and the vision in the
right was normal. On examination, visual acuity was 20/20 O.D.
and 20/30 O.S. Color vision with the Ishihara pseudochromatic
plates was 10/10 O.D. and 7.5/10 O.S. There was a left relative afferent pupillary defect. Orbital, ocular motility, IOP, and
slitlamp biomicroscopy examinations were all normal. Funduscopic examination of the right eye was normal. In the left eye,
the superior pole of the optic nerve was pale, with narrowing of
the superotemporal arcade vessel. Automated perimetry demonstrated a dense inferior altitudinal defect O.S. OCT retinal nerve
fiber layer thickness was normal O.D. (96 µm) and thin superotemporally O.S. (57 µm). Macular OCT demonstrated thinning
of the inner retinal layers in the superior retina with parafoveal
disruption within the ellipsoid zone.
24
Section IV: Test Interpretation
“I can’t get cocaine drops in my
office!”
Heather E Moss MD PhD
C ase
History and Exam
A 67-year-old man in excellent health presented for evaluation of
1 month of right eyelid drooping. He thought it got better in the
evening. He denied diplopia or change in vision. He had not had
headaches, dyspnea, dysarthria, or dysphagia.
On review of systems he noted right shoulder pain. He was
not taking any medications. He smoked 3 cigarettes each day. He
had a history of cocaine use, last 6 months prior to evaluation.
On examination, visual acuity was 20/20 with each eye. Ductions were full. The right pupil was 1 mm smaller than the left
pupil. There was no relative afferent pupillary defect. There was
2 mm of relative ptosis of the right eye. Anterior and posterior
ophthalmoscopic exams were unremarkable.
What would you do next?
•
•
•
•
MRI scan
Send to emergency department
Refer to neuro-ophthalmology
Do further in office testing/examination
Clinical Course and Outcome
Unilateral ptosis and miosis are concerning for a diagnosis of
Horner syndrome. Other diagnostic considerations include concurrent causes of isolated ptosis (eg, aponeurotic, myasthenia
gravis, third nerve palsy) and anisocoria (eg, physiologic, tonic
pupil, third nerve palsy). Additional clinical examination can
quickly refine the diagnosis.
Further diagnostic information regarding anisocoria can be
easily obtained by comparing the relative anisocoria in light and
dark environments. In this case, detailed pupil exam revealed
that the right pupil was 1 mm smaller than the left pupil with the
room lights on and 1.5 mm smaller than the left pupil with the
room lights off. This suggested that the right eye was not dilating
adequately due to sympathetic dysfunction.
Further diagnostic information regarding ptosis can be
obtained by measuring the position and function of the upper
eyelids. In this case, eyelid measurements included marginal
reflex distance 1 of 2 mm in the right eye and 4 mm in the left.
Levator function was normal and symmetric in both eyes without curtain, twitch, or fatigue. This could be consistent with
aponeurotic ptosis or Müller muscle weakness due to sympathetic dysfunction. Though not seen in this patient, another sign
sometimes seen in oculosympathetic injury is lower lid ptosis,
characterized by decreased marginal reflex distance 2.
2015 Subspecialty Day | Neuro-Ophthalmology
In any case of ptosis and/or anisocoria, further review of
systems and examination can help to refine the differential diagnosis. Though not seen in our patient, ocular misalignment could
point toward third nerve palsy, myasthenia gravis, or cavernous
sinus syndrome. Accompanying numbness, weakness, or coordination disturbance might suggest brainstem injury. Our patient
reported shoulder pain ipsilateral to his ophthalmic findings,
which was concerning for an associated process in the chest,
cervical spine, or neck. His cocaine history, though remote, suggested vascular etiologies.
In this case the anisocoria worse in the dark and ptosis with
normal levator function were sufficient to clinically diagnose
Horner syndrome, and workup was initiated to identify a causal
etiology. If the exam is equivocal or if there are contraindications
to imaging, eye drop testing can be used to confirm a Horner
syndrome diagnosis. Cocaine drops act presynaptically and cause
less dilation of a pupil with sympathetic dysfunction than of a
pupil without sympathetic dysfunction. Apraclonidine, which
is more readily available than cocaine, acts postsynaptically to
cause more dilation of a pupil with sympathetic dysfunction
than of a pupil without sympathetic dysfunction. Literature suggests that both tests have similar sensitivity and sensitivity for
Horner syndrome. Based on the mechanisms of action, cocaine is
theoretically more sensitive than apraclonidine in the hyperacute
setting, though reports suggest that this does not limit its use in
clinical practice. In our patient, apraclonidine testing had been
used by the referring provider, who reported that 30 minutes
following instillation of 1 gtt. of 0.5% apraclonidine in each eye,
the right pupil was 1 mm larger than the left. This confirmed a
diagnosis of right Horner syndrome.
If there is a known causal explanation (eg, recent carotid
instrumentation, known recent lateral medullary stroke), then
further evaluation may not be needed. However, this was not the
case with our patient, so imaging was necessary to screen for etiologies requiring treatment. Complete evaluation of the sympathetic chain includes brain imaging (MRI, not CT), neck imaging
(MRI), carotid angiographic imaging (CTA or MRA, not carotid
ultrasound), and chest imaging (CT or MRI, not chest x-ray),
which can be refined or staged based on accompanying signs
and symptoms. Important etiologies requiring treatment include
brainstem stroke or tumor, Pancoast tumor in the lung apex,
carotid pathology such as dissection, cavernous sinus pathology
such as tumor, or orbital process. In our case, MRI of the brain
and neck, MRA neck, and CT chest with contrast were obtained.
The chest CT showed a right lung apex mass. Biopsy was diagnostic of small cell lung cancer.
Section IV: Test Interpretation
2015 Subspecialty Day | Neuro-Ophthalmology
Mini-Talk: Bias in Testing—Do I Only Find What I
Thought Was There?
Harold E Shaw Jr MD
NOTES
25
26
Section IV: Test Interpretation
“The MRI is abnormal; now what?”
Fiona E Costello MD
C ase
2015 Subspecialty Day | Neuro-Ophthalmology
Everyone’s ESR Is High: Who Needs
a Biopsy?
Todd Alan Goodglick
C ase
History and Exam
History
A 30-year-old woman with recent headaches is admitted after
2 generalized tonic-clonic seizures. Her comorbidities include
Crohn disease, for which she recently initiated treatment with
infliximab. The patient stands 5 feet tall and weighs 260 lbs.
Over the past 4 weeks she has experienced a 30-lb. weight loss
secondary to bloody diarrhea from active inflammatory bowel
disease.
She describes a week-long history of blurred vision in both
eyes. She describes episodes of visual dimming in one or both
eyes that is often triggered by rapid position changes and hears a
pulsing “heartbeat in her ears.” She denies diplopia.
Examination
Visual acuity is 20/100 in the right eye and 20/80 in the left eye.
Pupils measure 4 mm in darkness and constrict to 3 mm in bright
light with no relative afferent pupillary defect. The patient is
able to read 3/6 Hardy-Rand-Rittler pseudoisochromatic plates
with each eye. Ocular motility and external ocular examination
are normal. The visual field testing and fundus examination are
depicted in Figures 1 and 2.
The patient undergoes a cranial MRI scan with venography,
which demonstrates a partially empty sella, bilateral flattening of
the globes, and tortuosity of the optic nerve sheaths. Initially, the
MR venogram is reported as normal.
History and Exam
Fifty-nine-year-old white woman referred for variable headache
on her left side for 3 months, without association to activity or
position, pain around the ears and in throat. She specifically
denies jaw claudication. She also notes 2 episodes, 2 and 3 weeks
ago, of graying out of part of her peripheral vision in her left eye,
but it only lasted about a minute so she didn’t seek help. Her
internist has previously obtained a noncontrast MRI of the head
and carotid Doppler study, which were normal.
Patient’s past medical history includes type 2 diabetes mellitus, hypertension, and generalized arthritis presently in a flareup, including the neck, which she attributes the headache to. She
was diagnosed with pneumonia 3 weeks ago, associated with
fevers and mild congestive heart failure.
Her visual acuities were 20/25 O.D., 20/30 O.S. The rest of
her examination was normal.
Clinical Course and Outcome
Below are possible laboratory results:
Sed rate
C-reactive
protein
1
2
3
4
22
35
60
60
0.5
5
0.5
5
The four options are as follows: 1 = observe, 2 = grab for the bottle of prednisone
on the shelf, 3 = grab for the surgical scheduling sheet on the shelf, 4 = grab for the
glass of single malt scotch on the shelf.
The patient in question, on the young side for a reflexive
consideration of giant cell arteritis (GCA) (diagnostic criteria 50
years old or greater but peak incidence at 70-80), presents with
common symptoms that are neither textbook nor dismissible for
this disease (ie, vague but bothersome temporal headache, transient visual obscurations, generalized myalgias, and equivocal lab
results with other potential reasons that might explain them. As
is often the case the exam doesn’t demonstrate classic findings
(catastrophic vision loss, focal or diffuse signs of inflammation
along the superficial temporal arteries, pale optic nerve head
swelling).
Answer and Teaching Points
28
Section I: Vision Loss
2015 Subspecialty Day | Neuro-Ophthalmology
Section I: Vision Loss Made Ridiculously Simple
“I suddenly lost vision in one eye.”
Marie D Acierno MD
Final Diagnosis
This young man has unilateral optic neuritis.
Teaching Points
Optic neuritis is an acute inflammatory disorder of the optic
nerve commonly associated with multiple sclerosis. It is manifest by acute, unilateral visual loss, eye pain, typically with eye
movement, decreased color and/or decreased contrast, visual
field defect, and a relative afferent pupillary defect. Ninety-two
percent of patients have accompanying pain. Sixty-five percent of
patients have a normal appearing optic nerve, reflecting retrobulbar disease, while 35% have optic disc edema.
Our patient presented with optic disc edema (rather diffuse /
severe without hemorrhages), a less common presentation for
acute, isolated optic neuritis. If a patient presents with optic disc
edema, it may be necessary to re-examine the patient in several
days to assess for development of macular exudates. This finding
establishes the diagnosis of neuroretinitis, which is not associated with demyelinating disease and therefore does not warrant
neuroimaging. Sarcoid can also present as a typical optic neuritis,
usually with optic disc involvement. Visual recovery varies for
sarcoid optic neuritis, but it often does not improve without
steroid therapy. One should definitely consider Leber hereditary
optic neuropathy (LHON) in such a clinical presentation. LHON
may occur in both men and women, but it is more common in
young adult males. Patients with LHON may present with a
normal-appearing disc in the affected eye or with mild disc swelling with peripapillary telangiectasia. LHON is a painless visual
loss resulting in a dense central scotoma in the affected eye. Only
those LHON patients with specific mitochondrial mutations may
experience partial or complete visual recovery after months or
years. Patients will usually acquire bilateral simultaneous visual
loss in weeks to months in the contralateral eye.
The risk of developing MS following an acute optic neuritis is
lower for those patients with severe optic disc swelling with hemorrhages or a macular star, absence of pain, vision reduced to no
light perception, and in males.
In a typical presentation of optic neuritis, blood laboratory
testing and lumbar puncture are not indicated. If the clinical
course is unusual in any way, such as prolonged pain, lack of
visual recovery within 4 to 6 weeks, recurrence within 2 months,
or manifestation of other clinical findings, the presentation is
considered atypical and a workup is pursued accordingly.
Optic neuritis may be the first clinical sign of MS or it may
occur in a patient with the established diagnosis of MS. Our
patient’s MRI orbit imaging study demonstrates left optic nerve
enhancement, which is helpful but not necessary for the diagnosis of optic neuritis. Contrast-enhanced imaging with fat suppression technique allows optimal views of the optic nerves. MRI
brain/orbit imaging study with gadolinium should be considered
in all patients with optic neuritis to assess the risk for MS.
MS demyelinating plaques can be found throughout the brain
but have a predilection for the periventricular white matter
region in an ovoid configuration perpendicular to the periventricular region and lateral borders of the corpus callosum. The
overall 15-year risk of developing MS after optic neuritis is
50% based on clinical criteria. However, the risk can be further
refined. In the absence of demyelinating lesions, the risk is low,
at 25%, while the risk is 72% when lesions are present.
If the clinical presentation has the typical features of an acute
optic neuritis, spontaneous visual recovery usually begins in the
first 4 to 6 weeks, with continued improvement and good visual
prognosis by 6 to 12 months. Ninety-four percent recover to
20/40 or better, and 3% of patients remain with 20/200 or worse
visual outcome after 5 years, based on the Optic Neuritis Treatment Trial (ONTT) results. However, those with visual recovery
still had residual deficits such as decreased contrast sensitivity.
The clinical decision to manage and treat a patient with
typical optic neuritis with steroids is often challenging. The
ONTT was a multicentered, randomized, controlled clinical trial
designed to evaluate the efficacy and safety of oral prednisone
(1 mg/kg daily for 2 weeks) vs. intravenous methylprednisolone
(250 mg 4 times daily for 3 days, followed by prednisone 1 mg/
kg daily for 11 days) compared with oral placebo for the treatment of acute optic neuritis. Primary outcome measures were
contrast sensitivity and visual field, and secondary measures
were vision and color. In summary, the ONTT revealed the IV
steroid-treated patients recovered visual function faster within
the first 4-6 weeks following onset of optic neuritis, but showed
no statistical difference in final visual outcome from the IV steroid-treated patients and those receiving placebo. Oral steroids
are contraindicated for acute optic neuritis because of a higher
rate of recurrence of optic neuritis.
Treatment varies among clinicians despite the ONTT. Some
clinicians treat patients with 1 gm per day infused over 3 to 5
days in an outpatient infusion center. Some use oral prednisone
thereafter, while others have abandoned it. The potential side
effects and complications should be discussed with the patient
prior to administration. Oral steroids alone are always contraindicated for acute optic neuritis because of an increased rate
of recurrences of optic neuritis. But all patients with abnormal
MRIs should be referred to the neurologist for further evaluation and management for MS. Institution of disease-modifying
therapy should be discussed.
OCT of the nerve fiber layer is a useful supplementary test
to measure optic nerve structure in patients with optic neuritis.
OCT can quantify the initial swelling and subsequent atrophy of
the retinal nerve fiber layer as disc pallor develops at 6-8 weeks
following the onset of an acute, isolated optic neuritis. Often, the
optic atrophy can be subtle on funduscopic examination. The
OCT provides a quantitative measure of the retinal nerve fiber
layer thickness that can document the degree of optic atrophy.
Section I: Vision Loss
2015 Subspecialty Day | Neuro-Ophthalmology
Selected Readings
C. Dry AMD
1.Basic and Clinical Science Course Section 5: Neuro-Ophthalmology, 2009-2010. San Francisco: AAO, 2009: 111-172.
D. Macular edema
2. Cleary PA, Beck RW, Anderson MM Jr, Kenny DJ, Backlund JY,
Gilbert PR. Design, methods, and conduct of the Optic Neuritis
Treatment Trial. Control Clin Trials. 1993; 14(2):123-142.
1. Postoperative cystoid macular edema (CME)
2. Diabetic macular edema
29
3. Uveitic CME
3. Comi G. Shifting the paradigm toward earlier treatment of multiple
sclerosis with interferon beta. Clin Ther. 2009; 31(6):1142-1157.
4. Fisher JB, Jacobs FA, Markowitz CE, et al. Relation of visual function to retinal nerve fiber layer thickness in multiple sclerosis. Ophthalmology 2006; 113:324-332.
4. Systemic medications (thiazolidinediones, fingolimid, tamoxifen, taxanes, niacin, interferon)
5. Ocular medications (prostaglandin analogs, epinephrine, timolol)
5. Gea Y. Multiple sclerosis: the role of MR imaging. AJNR Am J
Neuroradiol. 2006; 27:1165-1176.
E. Infectious (toxoplasmosis, presumed ocular histoplasmosis, diffuse unilateral subacute neuroretinitis)
6. Jirawuthiworavong GV, Miller AM, Fajardo D. Demyelinating
optic neuritis. 2015. EyeWiki website: eyewiki.org/demyelinating_optic_neuritis.
F. Inflammatory (sarcoidosis, Vogt-Koyanagi-Harada
syndrome, posterior scleritis, serpiginous chorioretinopathy, autoimmune retinopathy)
G. Retinitis pigmentosa and allied disorders
H. Toxicity (hydroxychloroquine, chloroquine, thioridazine, chlorpromazine, quinine)
I. Macular dystrophy (cone-rod dystrophy, Best disease, Stargardt disease)
J. Macular telangiectasia
7. Kappos L, Freedman MS, Polman CH, et al; BENEFIT Study
Group. Long-term effect of early treatment with interferon beta-1b
after a first clinical event suggestive of multiple sclerosis: 5-year
active treatment extension of the Phase 3 BENEFIT trial. Lancet
Neurol. 2009; 8(11):987-997.
8. McKinney AM, Lohman BD, SarikayaB, Benson M, Lee MS, Benson MT. Accuracy of routine fat-suppressed FLAIR and diffusionweighted images in detecting clinically evident acute optic neuritis.
Acta Radiol. 2013; 54:455-461.
9. Optic Neuritis Study Group. The 5-year risk of MS after optic neuritis: experience of the Optic Neuritis Treatment Trial. Neurology
1997; 49:1404-1413. K. White dot syndromes (AZOOR)
L. Nutritional vitamin A deficiency
10. Optic Neuritis Study Group. High- and low-risk profiles for the
development of multiple sclerosis within 10 years after optic neuritis. Arch Ophthalmol. 2003; 121:944-949. 11. Optic Neuritis Study Group. Multiple sclerosis risk after optic neuritis: final Optic Neuritis Treatment Trial follow-up. Arch Neurol.
2008; 65(6):727-732.
M. Paraneoplastic syndromes
1. Cancer-associated retinopathy (CAR)
2. Melanoma-associated retinopathy (MAR)
N. Tumor-ocular melanoma
O. Radiation-induced retinopathy
II. Optic Nerve
A.Glaucoma
B. Compressive lesions affecting the optic nerve anterior to the chiasm (cerebral meningioma, aneurysm)
Guy V Jirawuthiworavong MD
C. Optic nerve meningioma / optic nerve glioma
D. Toxic optic neuropathy (ethambutol, linezolid)
Final Diagnosis
E. Nutritional optic neuropathy (post-gastric bypassB12, folate, thiamine)
F. Hereditary optic neuropathy-dominant optic atrophy (Kjer’s)
“I am slowly losing vision in one of
my eyes.”
Acute zonal occult outer retinopathy (AZOOR)
Teaching Points
In summary, this patient presents with unilateral visual loss with
a relative afferent pupillary defect, pericentral scotoma, and sectoral retinal pigmented epithelial atrophy corresponding to visual
field, OCT, and fluorescein angiography findings, consistent
with the diagnosis of AZOOR.
Broad differential diagnosis of gradual, unilateral visual loss:
I.Retina
A. Vitreoretinal interface disorders (epiretinal membrane, vitreomacular traction, macular hole)
B. Central serous chorioretinopathy
The fundus findings of retinal pigmented epithelial atrophy
narrows the differential diagnosis to retinal causes. OCT of the
macula shows thinning of the outer retina, but there is no vitreomacular traction, intraretinal fluid, or subretinal fluid. The
patient’s history of photopsias, nyctalopia, scotoma, and gradual
progression is consistent with conditions such as retinitis pigmentosa, cone-rod dystrophy, white dot syndrome, autoimmune retinopathy, or a paraneoplastic syndrome. Blind spot enlargement
can be the first visual field defect seen in retinitis pigmentosa.
However, asymmetry of fundus findings and a relative afferent
pupillary defect (RAPD) are exceptions to the rule in patients
with retinitis pigmentosa, as well as in cone-rod dystrophy and
30
Section I: Vision Loss
autoimmune retinopathy (AIR). Color vision is not spared in
cone-rod dystrophy. CAR, MAR, and AIR patients have attenuated vasculature but otherwise have no other typical retinal findings. MAR patients usually present with a preceding diagnosis
of melanoma. Underlying cancer screening and the presence of
antiretinal antibodies against retinal proteins and retinal tissue
can help differentiate CAR, MAR, and AIR. This patient’s left
eye shows peripapillary changes seen on fluorescein angiography.
As a side note, Stargardt disease tends to spare the peripapillary
retina and the fovea, and the light-colored yellow flecks found in
Stargardt disease rarely become hyperpigmented.
AZOOR is an idiopathic inflammatory syndrome that affects
young women more than men. AZOOR patients may complain
of gradual loss of vision over time with constant photopsias.
They often present with an enlarged blind spot that later progresses into a temporal / zonal visual field defect. This condition
can affect one or both eyes and tends to be asymmetric in presentation. Central vision can be spared, and there is minimal RAPD
at onset. Patients present acutely without any retinal changes on
exam, and as a result, the diagnosis is often made many years
later as the RPE changes are noted in follow-up. Patient’s family
members can have an underlying autoimmune condition.
AZOOR is diagnosed by history and retinal exam and can
be confirmed by visual field testing, fluorescein angiogram, and
OCT. The retinal findings on OCT and fluorescein angiogram
correspond topographically to the locations of the visual field
defect. These field defects tend to be peripapillary or contiguous
with the optic nerve. The newest imaging modality of fundus
autofluorescence can show characteristic zones of hyperfluorescence at the leading edge of the AZOOR. It is postulated that
AZOOR is either an autoimmune dysregulatory condition or
a viral infectious disease entering the optic disc, but its etiology
remains to be determined.
Unlike AZOOR, retinitis pigmentosa (RP) can present at any
age and tends to follow the rules of mendelian genetics, except
in simplex RP. The common visual field defect is a ring scotoma
but, as mentioned earlier, can uncommonly present with an
enlarged blind spot on visual field testing. Fundus findings of
bone spicules are sine qua non for RP, whereas AZOOR may or
may not have any hyperpigmented spots.
Selected Readings
1. Ryan SJ. Retina, vols. 1 and 2. 5th ed. London: Saunders-Elsevier;
2013.
2. Yanuzzi LA. The Retina Atlas. London: Saunders-Elsevier; 2010.
3. Nussenblatt RB, Whitcup SM. Uveitis: Fundamentals and Clinical
Practice. 4th ed. London: Mosby-Elsevier; 2010.
4. Makri OE, Georgalas I, Georgakopoulos CD. Drug-induced macular edema. Drugs 2013; 73(8):789-802.
5. Mrejen S, Khan S, Gallego-Pinazo R, et al. Acute zonal occult outer
retinopathy: a classification based on multimodal imaging. JAMA
Ophthalmol. 2014; 132(9):1089-1098.
2015 Subspecialty Day | Neuro-Ophthalmology
“My doctor says I might have
glaucoma.”
Jacinthe Rouleau MD
Final Diagnosis
Prior nonarteritic anterior ischemic optic neuropathy
Teaching Points
1. Be aware that various optic neuropathies can mimic glaucomatous visual fields or cupping.
2. Clues that raise the suspicion of nonglaucomatous cupping
are pallor of the rim, reduced central acuity or color vision
loss out of proportion to disc cupping, atypical visual field
defects, discordance between visual fields and cupping,
marked relative afferent pupillary defect, and patient age
less than 50 years old.
3. When optic atrophy is suspected to be from an old nonarteritic anterior ischemic optic neuropathy (NAION) but
prior optic nerve edema cannot be documented, appropriate imaging of the orbits is recommended to eliminate a
compressive or infiltrative lesion.
Selected Readings
1. Greenfield DS, Siatkowski RM, Glaser JS, Schatz NJ, Parrish RK
2nd. The cupped disc: who needs neuroimaging? Ophthalmology
1998; 105:1866-1874.
2. Fraser CL, White AJ, Plant GT, Martin KR. Optic nerve cupping
and the neuro-ophthalmologist. J Neuroophthalmol. 2013; 33:377389.
3. Golnik K. Nonglaucomatous optic atrophy. Neurol Clin. 2010;
28:631-640.
4. Atkins EJ, Bruce BB, Newman NJ, Biousse V. Treatment of nonarteritic anterior ischemic optic neuropathy. Surv Ophthalmol. 2010;
55:47-63.
5. Lee AG, Chau FY, Golnik KC, Kardon RH, Wall M. The diagnostic yield of the evaluation for isolated unexplained optic atrophy.
Ophthalmology 2005; 112:757-759.
Section I: Vision Loss
2015 Subspecialty Day | Neuro-Ophthalmology
“I lose my vision and then it comes
back.”
31
“I’m losing vision in both eyes.”
John J Chen MD
Jeffrey Bennett MD PhD
Final Diagnosis
Final Diagnosis
Vasospastic amaurosis fugax causing transient monocular vision
loss
This patient had both normal-tension glaucoma and compressive
optic neuropathy, with the former leading to a delay in the diagnosis of the compressive element.
Teaching Points
Teaching Points
Retinal vasospasm is a rare cause of amaurosis fugax. During an
episode, funduscopic findings include arterial and venous narrowing due to reduced arterial blood flow, “box-carring” of the
blood column, and collapse of the retinal veins. Between events,
the fundus exam is normal. Rarely, events may result in vascular
retinopathy or optic neuropathy.
As vasospastic amaurosis is rare, the diagnosis should be
made only after excluding visual and life-threatening vascular
conditions in susceptible individuals. Critical conditions that are
important to consider in the patient with transient monocular
vision loss include carotid stenosis, cardiac emboli (valvular disease and paradoxical emboli), hypercoagulable states (antiphospholipid antibody syndrome), and vasculitis. In many, but not
all instances, fundus findings may provide important clues such
as cholesterol, calcific, or platelet-fibrin emboli, or choroidal
changes (Elschnig spots). Carotid Doppler ultrasound, erythrocyte sedimentation rate, C-reactive protein, and echocardiography should be performed in all patients with vascular risk factors
who have transient monocular vision loss. MR angiography
should be added if there is any concern for carotid dissection.
Since vasospastic amaurosis has been associated with polyarteritis nodosa and eosinophilic vasculitis, additional autoimmune
serologies (ANA and ANCA) should be considered in suspicious
cases. Also, given the reported association between vasospastic
amaurosis and cluster / migraine headaches, a careful headache
history may be helpful in making a diagnosis when other causes
have been excluded.
Vasospastic amaurosis is exquisitely responsive to calcium
channel blockade. Verapamil 240-360 mg daily is often sufficient
to resolve the recurrent events.
1. Compressive optic neuropathy causes a painless, progressive decline in vision, often affecting central vision first.
With compressive optic neuropathy, the optic nerves
initially appear normal without edema unless the lesion
is anterior or causes obstructive hydrocephalus. Gradual
pallor of the optic nerve with thinning of the retinal nerve
fiber layer and ganglion cell layer is expected if not treated.
Because compressive optic neuropathy is generally reversible if found early, progressive painless vision loss is a sign
of retrobulbar compression until proven otherwise.
2. Patients can have two pathologies. This patient had
normal-tension glaucoma with advanced cupping, which
delayed the diagnosis of compressive optic neuropathy.
While Occam’s razor of diagnostic parsimony typically
holds true in medicine, we have to keep in mind Hickam’s
dictum: “Patients can have as many diseases as they damn
well please.”1
3. Compressive lesions can cause cupping of the nerve in
some instances.2,3 Therefore, neuroimaging is required to
evaluate for nonglaucomatous optic atrophy in patients
with cupping from presumed normal-tension glaucoma
who have atypical features, such as an age younger than
50 years old, decreased central visual acuity, pallor of the
residual neuroretinal rim, visual field defects respecting
the vertical midline, color vision deficits, or symptoms of
hypothalamic-pituitary dysfunction.4
4. A central or cecocentral scotoma on visual field testing is
almost always pathologic. Thompson and colleagues studied the ability to volitionally create functional visual fields
and a cecocentral scotoma was the hardest to create.5
Selected Readings
References
1. Bernard GA, Bennett JL. Vasospastic amaurosis fugax. Arch Ophthalmol. 1999; 117(11):1568-1569.
1. Hilliard AA, Weinberger SE, Tierney LM Jr., Midthun DE, Saint
S. Clinical problem-solving: Occam’s razor versus Saint’s Triad. N
Engl J Med. 2004; 350(6):599-603.
2. Petzold A, Islam N, Plant GT. Video reconstruction of vasospastic
transient monocular blindness. N Engl J Med. 2003; 348(16):16091610.
3. Hill DL, Daroff RB, Ducros A, Newman NJ, Biousse V. Most cases
labeled as “retinal migraine” are not migraine [review]. J Neuroophthalmol. 2007; 27(1):3-8.
2. Bianchi-Marzoli S, Rizzo JF 3rd, Brancato R, Lessell S. Quantitative analysis of optic disc cupping in compressive optic neuropathy.
Ophthalmology 1995; 102(3):436-440.
3. Kupersmith MJ, Krohn D. Cupping of the optic disc with compressive lesions of the anterior visual pathway. Ann Ophthalmol
(Skokie). 1984; 16(10):948-953.
4. Piette SD, Sergott RC. Pathological optic-disc cupping. Curr Opin
Ophthalmol. 2006; 17(1):1-6.
5. Thompson JC, Kosmorsky GS, Ellis BD. Field of dreamers and
dreamed-up fields: functional and fake perimetry. Ophthalmology
1996; 103(1):117-125.
32
Section I: Vision Loss
“Can my child see?”
2015 Subspecialty Day | Neuro-Ophthalmology
Final Diagnosis
Teaching Point 2
Once the diagnosis of optic nerve hypoplasia is made, whether
bilateral or unilateral, further testing including neuroimaging
and an endocrinologic evaluation should be performed to assess
for features that would support the diagnosis of septo-optic
dysplasia. These include absent septum pellucidum, dysgenesis /
agenesis of the corpus callosum, and pituitary abnormalities. In
the setting of septo-optic dysplasia, endocrine dysfunction may
occur in spite of a normal radiographic appearance of the pituitary. Most commonly, defects in growth hormone are identified;
however, panhypopituitarism may occur.
Visual impairment secondary to bilateral optic nerve hypoplasia
in the setting of septo-optic dysplasia
Selected Readings
Teaching Points
1. Borchert M. Reappraisal of the optic nerve hypoplasia syndrome. J
Neuroophthalmol. 2012; 32:58-67.
Gena Heidary MD PhD
The presentation will include fundus photos, MRI findings suggestive of septo-optic dysplasia, and endocrine workup. In addition, I will provide recent follow-up details including VA and
treatment of strabismus to document the natural history of optic
nerve hypoplasia.
Teaching Point 1
In an infant with subnormal vision for age, a framework for
approaching the exam is to consider several general categories: a
structural basis for visual impairment (eg, optic nerve hypoplasia, congenital cataracts), an inherited retinal dystrophy / degenerative process, cortical visual impairment, or delayed visual
maturation. In this case, the clinical findings on the examination
revealed abnormalities of the optic nerves, providing the etiology
for the poor visual behavior. Because of the bilateral involvement, the baby presented with nystagmus; in contrast, children
who harbor unilateral optic nerve hypoplasia will commonly
present with a sensory strabismus.
2. Mohney BG, Young RC, Diehl N. Incidence and associated endocrine and neurologic abnormalities of optic nerve hypoplasia.
JAMA Ophthalmol. 2013; 131:898-902.
3. Ahmad T, Borchert M, Geffner M. Optic nerve hypoplasia and
hypopituitarism. Pediatr Endocrinol Rev. 2008; 5:772-777.
4. Ahmad T, Garcia-Filion P, Borchert M, Kaufman F, Burkett L, Geffner M. Endocrinological and auxological abnormalities in young
children with optic nerve hypoplasia: a prospective study. J Pediatr.
2006; 148:78-84.
5. Garcia-Filion P, Fink C, Geffner ME, Borchert M. Optic nerve
hypoplasia in North America: a re-appraisal of perinatal risk factors. Acta Ophthalmol. 2010; 88:527-534.
2015 Subspecialty Day | Neuro-Ophthalmology
Section II: Eye Pain and Headache
33
Section II: Your “What” Hurts? Eye Pain and
Headache Made Ridiculously Simple
“Worst headache of my life!”
“My head hurts for days on end.”
S Tonya Stefko MD
John H Pula MD
Final Diagnosis
Final Diagnosis
Pituitary apoplexy
Horner syndrome in a patient with cluster headaches
Teaching Points
Teaching Points
Because the pituitary gland sits directly inferior to the optic
chiasm, any enlargement of the gland can lead to a chiasmal syndrome. As in any situation, a slowly growing mass will produce
fewer symptoms than a rapidly expanding one. Depending on
the patient’s specific anatomy and the anatomy of the tumor,
however, patients may have a myriad of other effects involving the optic nerves, upper cranial nerves, and optic tracts. In
pituitary apoplexy, the cells of the tumor infarct and/or bleed
and very quickly put pressure upward on the diaphragma and
laterally toward the cavernous sinuses. This is thought to be due
to intrinsic marginal perfusion combined with high metabolic
demands. It can cause the classic “thunderclap” headache or a
more insidious onset of diplopia, ptosis, decreased vision, and
light sensitivity evolving over hours to days. Headache is present
> 95% of the time, and aberration of extraocular movements
occurs in up to 75% of patients. Decrease in visual fields and,
slightly less commonly, visual acuities are found in roughly half
of patients.
The incidence of apoplexy in pituitary tumors is 2%-12%
(most commonly in nonfunctioning adenomas). It is the initial
presentation of the tumor in about 80% of patients. There are
several known associations with apoplexy, including cardiac surgery, insulin infusion, transient increased intracranial pressure,
initiation of anticoagulation, and hypo- or hyperperfusion states.
Surgical decompression of the optic apparatus, preferably
from a ventral approach (trans-sphenoidal or expanded endonasal), results in improvement of visual deficits in more than 75%
of patients whose symptoms have been present for less than a
week. If the main ophthalmic sequela is ophthalmoplegia, most
patients improve even with nonsurgical management. Surgical
treatment of strabismus should be delayed by 6-12 months to
ensure that natural recovery is complete.
There are several different causes of painful Horner syndrome.
Our patient’s most pertinent finding was a Horner syndrome.
When working up Horner syndrome, particularly when painful, we recall that the sympathetic pathway to the eye consists
of three “orders.” Wallenberg syndrome (lateral medullary syndrome) due to vertebral dissection can cause a painful first-order
Horner syndrome but would be expected to have other features
(eg, acute vestibular syndrome or crossed sensory loss). A painful
Horner syndrome will usually be postganglionic (third order).
The postganglionic sympathetic pathway traverses the carotid
artery, skull base, cavernous sinus, and orbital apex. Painful
postganglionic, nonisolated Horner syndrome etiologies include
Tolosa-Hunt syndrome (orbital pseudotumor), skull-based
nasopharyngeal carcinoma or inflammatory lesions, or orbital
apex syndrome. These syndromes’ other clinical features will be
specific to their location. An isolated, painful Horner syndrome
mainly either involves the region of the superior cervical ganglion
and internal carotid artery or presents without imaging abnormalities (as in the primary headache disorders).
Considering this, the differential diagnosis for isolated painful
Horner syndrome includes:
Selected Readings
1. Oldfield EH, Merrill MJ. Apoplexy of pituitary adenomas: the
perfect storm. J Neurosurg. 2015; 122(6):1444-1449.
2. Fraser CL, Biousse V, Newman NJ. Visual outcomes after treatment
of pituitary adenomas. Neurosurg Clin N Am. 2012; 23(4):607619.
3. Briet C, Salenave S, Chanson P. Pituitary apoplexy. Endocrinol
Metab Clin North Am. 2015; 44(1):199-209.
• The trigeminal autonomic cephalgias (TACs)
– Hemicrania continua
– Paroxysma hemicrania
– SUNA/SUNCT (short-lasting unilateral neuralgiform
headaches with autonomic symptoms / short-lasting
unilateral neuralgiform headaches with conjunctival
injection and tearing)
– Cluster headache
• Internal carotid artery dissection
• Raeder syndrome
The trigeminal autonomic cephalgias are primary headache
disorders. Raeder syndrome is also called oculopupillary sympathetic paralysis and is caused by lesion in the middle cranial
fossa. A carotid dissection may be otherwise nonfocal. Distinguishing between these causes of an isolated painful Horner
syndrome relies on both clinical and paraclinical investigation,
which are described next.
Cluster headache has certain features, which distinguish it from
other similar syndromes.
Perhaps the first description of cluster headache was by the British neurologist Dr. Wilfred Harris. The seminal case described
by Harris highlighted “a man, age 47... pain struck him suddenly
34
Section II: Eye Pain and Headache
across the left temple and forehead, and lasted for three-quarters
of an hour, like an ‘electric battery,’ while his face became
flushed and he felt faint. The neuralgia recurred thrice daily, at
about eight-hour intervals, for six weeks, and then disappeared
entirely for two years. Ever since his first attack he had left cervical sympathetic paralysis, there being slight ptosis, with a small
pupil.”1
The International Classification of Headache Disorders2
defines cluster headaches as having “a sense of restlessness or
agitation,” as well as conjunctival injection / lacrimation, nasal
congestion / rhinorrhea, eyelid edema, forehead and facial flushing and sweating, ear fullness, or Horner syndrome. Headaches
are severe, unilateral, orbital or supraorbital / temporal, and
occur between 8/day to every other day and last 15-180 minutes
each. The timing of the pain distinguishes it from the other trigeminal autonomic cephalgias. SUNCT/SUNA neuralgia lasts
only seconds. Paroxysma hemicrania usually lasts 10-20 minutes,
and hemicrania continua may last for days or more.3
During the typical 6-12 week cluster period, the headaches
occur with clockwork regularity. Once a cluster period has completed, it may recur over a variable amount of time. On the other
hand, the Horner syndrome may not resolve after the cluster,
and in fact may never resolve in some cases. Response to treatments also somewhat distinguishes cluster headache from other
headache syndromes. Both alcohol and smoking can be headache
triggers. Abortive treatments include inhaled high-flow oxygen
for 10-15 minutes, which can completely relieve pain. Triptans
(eg, sumatriptan, zolmitriptan) provide acute pain relief during
a cluster attack. Steroids (oral prednisone for 10-12 days) provide a bridge to prophylaxis, which often consists of verapamil,
~240 mg daily in divided doses.
A painful Horner syndrome is a carotid dissection until proven
otherwise.
Although this patient ended up having cluster headaches, one
clinical pearl to remember is that in the correct clinical context, a
painful Horner syndrome should be considered a carotid dissection until proven otherwise. There may be no other neurologic
signs, and in these cases the isolated presentation especially
mimics a cluster headache. Up to 20% of spontaneous carotid
dissections have a Horner syndrome,4 and 91% of carotid artery
dissections that have a Horner syndrome are painful. Diagnosis
is especially important because without treatment to prevent
thromboembolism, the risk of stroke nears 20%.5
References
1. Harris W. Neuritis and Neuralgia. London: Humphrey Milford,
Oxford University Press; 1926.
2. Headache Classification Committee of the International Headache
Society. The International Classification of Headache Disorders,
2nd ed. Cephalalgia 2004; 24:1-160.
3. May A. Diagnosis and clinical features of trigemino-autonomic
headaches. Headache 2013; 53:1470-1478.
4. Glaser JS. Neuro-ophthalmology. Philadelphia: Lippincott Williams
& Wilkins, 1999.
5. Nautiyal A, Singh S, DiSalle M, O’Sullivan J. Painful Horner syndrome as a harbinger of silent carotid dissection. PLoS Med. 2005;
2(1):e19.
2015 Subspecialty Day | Neuro-Ophthalmology
“My eye just aches all the time.”
Rod Foroozan MD
Final Diagnosis
Greater occipital neuralgia
Teaching Points
1. Highlights of the International Headache Society diagnostic
criteria for greater occipital neuralgia include pain that is
recurrent, severe, and shooting or stabbing in quality. Other
criteria include dysesthesia or tenderness of the scalp.1
Symptoms from greater occipital neuralgia overlap other headache syndromes. The pain is thought to be caused by irritation
of the greater occipital nerve with multiple potential sites from
surrounding tissue.2,3 The distinction from other headache syndromes can be difficult but is aided by criteria outlined by the
International Headache Society, 3rd edition.
The anatomy of the greater occipital nerve and its relationship to the surrounding soft tissue is variable, but the nerve commonly originates from the medial branch of the dorsal ramus of
C2. The greater occipital nerve then ascends between the inferior
oblique capitis muscle and semispinalis capitis and pierces the
semispinalis muscle. It then runs rostrolaterally and deep to the
trapezius muscle and pierces the aponeurosis of the trapezius
slightly inferior to the superior nuchal ridge, where it becomes
subcutaneous and lies medial to the occipital artery. Branches
then supply cutaneous sensation to the posterior scalp from the
external occipital protuberance to the vertex. Please see Figures
1 and 2 from reference #3 listed below for the pertinent anatomy
and potential sites of compression of the greater occipital nerve.3
2. The eye examination should not reveal other causes for pain,
and other primary (such as migraine) and secondary (giant cell
arteritis, cervical spine disease) headache syndromes should be
excluded by history, physical examination, and ancillary testing.
Irritation of the greater, lesser, and third occipital nerves can all
lead to symptoms categorized as occipital neuralgia. Pain may
occur in the frontal area or the orbit through trigeminocervical
interneuronal connections in the trigeminal spinal nuclei. When
greater occipital neuralgia involves the orbit, the eye may be the
presumed site of pathology; however, the eye examination does
not reveal any abnormalities that would cause pain.4 Degenerative conditions involving the cervical spine can cause pain in a
similar pattern to occipital neuralgia, and in these patients, imaging of the skull base and cervical spine should reveal an abnormality.
3. Relief of pain along the course of the greater occipital nerve
with an injection of local anesthetic helps suggest the diagnosis
but is not pathognomonic, as other headache syndromes such
as migraine may be relieved by these injections.
The pain from occipital neuralgia may respond to oral agents
such as gabapentin, carbamazepine, and tricyclic antidepressants. However, injection of local anesthetic (typically lidocaine
or bupivacaine) has been thought to be more likely to relieve the
pain from occipital neuralgia, but may also relieve pain from
migraine and other headache syndromes. The course of the
greater occipital nerve can be estimated by finding the external
occipital protuberance and moving about 2 cm laterally and
Section II: Eye Pain and Headache
2015 Subspecialty Day | Neuro-Ophthalmology
2 cm inferiorly. Palpation for pulsation and aspiration prior to
injection helps avoid involvement of the occipital artery. Some
authors have suggested mixing corticosteroids with local anesthetic, although there is no proof that this adds additional benefit. Injection of botulinum toxin in the same anatomic area has
also been reported to limit the pain from greater occipital neuralgia in a small group of patients.
Some patients may not respond to oral agents or local anesthetic injections. Neuromodulation using high-voltage radio
frequency has been reported to improve pain, and occipital nerve
stimulation using an implanted stimulator has also been noted to
be effective for some refractory patients. Surgical decompression
has been noted to be curative in some patients.
The International Headache Society diagnostic criteria for
occipital neuralgia1
A. Unilateral or bilateral pain fulfilling criteria B-E
B. Pain is located in the distribution of the greater,
lesser and/or third occipital nerves.
C. Pain has two of the following three characteristics:
1. Recurring in paroxysmal attacks lasting from a
few seconds to minutes
2. Severe intensity
3. Shooting, stabbing, or sharp in quality
D. Pain is associated with both of the following:
1. Dysesthesia and/or allodynia apparent during
innocuous stimulation of the scalp and/or hair
2. Either or both of the following:
a. Tenderness over the affected nerve branches
b. Trigger points at the emergence of the greater
occipital nerve or in the area of distribution of
C2
E. Pain is eased temporarily by local anesthetic block
of the affected nerve.
F. Not better accounted for by another ICHD-3 diagnosis
35
“The light!! It hurts my eyes!!”
Bradley J Katz MD
Discussion
Photophobia, an abnormal intolerance to light, is associated with
a number of ophthalmic and neurologic conditions. However,
in the presence of a normal neuro-ophthalmic examination,
the most common conditions associated with photophobia are
migraine, blepharospasm, and traumatic brain injury. Recent
evidence indicates that the intrinsically photosensitive retinal
ganglion cells play a key role in the pathophysiology of photophobia. Although pharmacologic manipulation of intrinsically
photosensitive retinal ganglion cells may be possible in the
future, current therapies are directed at optical modulation of
these cells.
Teaching Points
1. Most patients with light sensitivity do not have an ocular
problem (eg, iritis).
2. Most patients with light sensitivity have migraine,
although it may be undiagnosed or misdiagnosed.
3. The purpose of the ophthalmic exam is to rule out other
light sensitive conditions, such as dry eye and blepharospasm.
4. Management includes optical treatments for indoor and
outdoor light sensitivity and referral to a headache specialist or neurologist.
Selected Readings
1. Blackburn MA, Lamb R, Digre KB, et al. FL-41 tint improves blink
frequency, light sensitivity, and functional limitations in patients
with benign essential blepharospasm. Ophthalmology 2009;
116:997-1001.
2. Digre KB, Brennan KC. Shedding light on photophobia. J Neuroophthalmol. 2012; 32:68-81.
3. Güler AD, Ecker JL, Lall GS, et al. Melanopsin cells are the principal conduits for rod-cone input to non-image-forming vision.
Nature 2008; 453:102-105.
References
4. Hattar S, Liao HW, Takao M, Berson DM, Yau KW. Melanopsincontaining retinal ganglion cells: architecture, projections, and
intrinsic photosensitivity. Science 2002; 295:1065-1070.
1. The International Classification of Headache Disorders, 3rd ed.
(beta version). Cephalalgia 2013; 33:629-808.
5. Kawasaki A, Kardon RH. Intrinsically photosensitive retinal ganglion cells. J Neuroophthalmol. 2007; 27:195-204.
2. Dougherty C. Occipital neuralgia. Curr Pain Headache Rep. 2014;
18:411.
6. Noseda R, Kainz V, Jakubowski M, et al. A neural mechanism for
exacerbation of headache by light. Nat Neurosci. 2010; 13:239245.
3. Cesmebasi A, Muhleman MA, Hulsberg P, et al. Occipital neuralgia: anatomic considerations. Clin Anat. 2015; 28:101-108.
4. Lee AG, Brazis PW. The evaluation of eye pain with a normal ocular exam. Semin Ophthalmol. 2003; 18:190-199.
7. Noseda R, Burstein R. Advances in understanding the mechanisms
of migraine-type photophobia. Curr Opin Neurol. 2011; 24:197202.
36
Section II: Eye Pain and Headache
“It hurts when I talk, and this cough
won’t go away.”
Gabrielle R. Bonhomme MD
Final Diagnosis
Temporal arteritis presenting with persistent cough, trismus, and
transient diplopia.
Discussion
Temporal arteritis (GCA) is a medium to large vessel vasculitis
that commonly presents with visual symptoms due to anterior
ischemic optic neuropathy (AION) (94%) and classic constitutional symptoms of new-onset headache, jaw claudication, scalp
tenderness, and polymyalgia rheumatica (PMR). The incidence
of GCA increases in patients older than 50 years of age, ranging
from 18 to 27 cases per 100,000 in people over 50 years of age.
Genetic associations with HLA-DR4 and HLA-DRB1 exist.1
The American College of Rheumatology (ACR) requires 3 of
the following 5 criteria for diagnosis:
1. Age of 50 year or older
2. New headache
3. Temporal artery abnormality (tenderness to palpation or
decreased pulsation)
4. ESR > 50 mm/h
5. Abnormal findings on temporal artery biopsy (evidence of
vasculitis)1
However, patients may present without classic systemic
symptoms or with a variety of atypical, nonspecific constitutional symptoms, particularly early in the course of the disease.
Further, occult GCA may present with ocular involvement in
the absence of systemic signs or symptomatology in 21% of
patients.2 Given these atypical cases, a recent study estimates that
the ACR criteria used in isolation may miss up to 25% of cases
of GCA.3 While studies have quoted the specificity of ESR and
CRP in combination to be 97%,4 GCA may occur in the absence
of abnormally elevated inflammatory markers. Therefore, it
behooves the ophthalmologist both to identify expected ocular
signs of GCA and to recognize atypical systemic symptoms
indicative of evaluation for temporal arteritis to properly direct
patient management. In addition to large vessel involvement
such as aortitis and aortic aneurysm, otolaryngeal and respiratory disorders such as intractable, nonproductive cough, trismus,
and tongue infarction resulting from ischemia of affected tissues
may be the first presenting symptoms of GCA.6 A recent study
reported dry cough as a presenting symptom of GCA in 13.6%
2015 Subspecialty Day | Neuro-Ophthalmology
of patients with biopsy-proven GCA, often in association with
other systemic symptoms or elevated CRP, as seen in our patient.
Early recognition of dry cough in an elderly patient as a potential
symptom of GCA, particularly when in association with elevated
inflammatory markers or ocular findings such as AION, and
prompt temporal artery biopsy, may prevent delay of diagnosis
of GCA, and resultant vision loss.7
Teaching Points
1. Temporal arteritis (GCA) should be considered in the differential diagnosis when an elderly patient presents with
new-onset headache.
2. Recognition of less common, non-ocular signs of ischemia,
such as cough, trismus, and jaw claudication, is crucial to
the early diagnosis and therefore timely treatment of GCA.
3. Occult GCA should be suspected in patients over the age
of 50 years with AION and the above symptoms, even in
the absence of abnormal inflammatory markers such as
ESR and CRP.
References
1. Hunder GG, Bloch DA, Michel BA, et al. The American College of
Rheumatology 1990 criteria for the classification of giant cell arteritis. Arthritis Rheum. 1990; 33(8):1122-1128.
2. Hayreh SS, Podhajsky PA, Zimmerman B. Occult giant cell arteritis:
ocular manifestations. Am J Ophthalmol. 1998; 125(6):893.
3. Murchison AP, Gilbert ME, Bikyk JR, et al. Validity of the American College of Rheumatology criteria for the diagnosis of giant cell
arteritis. Am J Ophthalmol. 2012; 154:617-619.
4. Hayreh SS, Podhajsky PA, Raman R, et al. Giant cell arteritis: validity and reliability of various diagnostic criteria. Am J Ophthalmol.
1997; 123(3):285-296.
5. Parikh M, Miller NR, Lee AG, et al. Prevalence of a normal
C-reactive protein with an elevated erythrocyte sedimentation
rate in biopsy-proven giant cell arteritis. Ophthalmology 2006;
113(10):1842-1845.
6. Imran TF, Helfgott S. Respiratory and otolaryngologic manifestations of giant cell arteritis. Clin Exp Rheumatol. 2015; 89(2):164170.
7. Zenone T, Puget M. Dry cough is a frequent manifestation of giant
cell arteritis. Rheumatol Int. 2013; 33:2165-2168.
8. El-Dairi MA, Chang L, Bhatti T, et al. Diagnostic algorithm for
patients with suspected giant cell arteritis. J Neuro-Ophthalmol.
Epub ahead of print 2015 Mar 23.
2015 Subspecialty Day | Neuro-Ophthalmology
Section III: Double Vision and Nystagmus
37
Section III: Double Vision and Nystagmus Made
Ridiculously Simple
“I see double, and my eyelid is in the
way.”
Paul H Phillips MD
Final Diagnosis
Right CN III palsy, CN VI palsy, and Horner syndrome from a
right cavernous sinus meningioma
Cranial nerve VI innervates the lateral rectus muscle, and therefore a CN VI palsy is characterized by an abduction deficit that
increases with gaze toward the affected eye and at distance.1
Injury may occur at multiple locations, including the nucleus,
fascicle, subarachnoid space, cavernous sinus, and orbit.
Multiple etiologies may cause CN VI dysfunction, including
ischemia, compression, inflammation (multiple sclerosis, sarcoidosis), infection, and trauma. An increase or decrease in intracranial pressure may shift the brainstem and stretch CN VI, resulting in a unilateral or bilateral CN VI palsy. Thus, it is important
to evaluate the disc for papilledema in patients that present with
CN VI palsy.
Microvascular ischemia to the subarachnoid segment of the
nerve is a common cause of an isolated sixth-nerve palsy. The
following characteristics support an ischemic etiology:2
• Age greater than 50
• Arteriosclerotic risk factors such as hypertension, diabetes
mellitus, hypercholesterolemia, tobacco use
• No history of cancer
• The abduction deficit remains isolated during follow-up
examination.
• The abduction deficit stabilizes after 1-2 weeks and
improves in 3-4 months.
If patients fulfill these criteria, many investigators will presume a microvascular etiology and not obtain neuroimaging at
onset. Associated risk factors such as diabetes and hypertension
should be evaluated, and the patient should be followed to confirm improvement of the abduction deficit within 3-4 months.
Progressive or unresolved palsies or additional neurological
deficits mandate further neuroimaging. Murchison et al2 showed
that following this algorithm is cost-effective and unlikely to miss
an alternative, treatable etiology. However, Chou et al3 showed
that 4 of 19 patients (21%) with CN VI palsy that fulfilled these
criteria had nonischemic etiologies, including neoplasm, brainstem infarct, multiple sclerosis, and pituitary apoplexy, and recommend neuroimaging at presentation of all patients with CN
VI palsy.
Additional neurological deficits allow more refined localization and mandate neuroimaging. Further diagnostic testing
should be considered, including lumbar puncture, chest imaging,
and hematologic studies for etiologies such as syphilis, sarcoidosis, and collagen vascular disease.
A brainstem lesion affecting the sixth cranial nerve nucleus
will cause a gaze palsy (neither eye can rotate horizontally to
the side of the CN VI nuclear lesion), often accompanied by CN
VII palsy as cranial nerve VII fibers curve around the sixth nerve
nucleus. Brainstem lesions that affect the fascicle of the sixth
nerve may also damage the seventh nerve fascicle, tractus solitarius, and the descending tract of the trigeminal nerve, causing an
ipsilateral abduction deficit, facial paresis, loss of taste over the
anterior 2/3 of the tongue, and facial hypoesthesia (Foville syndrome). Lesions located in the ventral pons may affect the sixth
cranial nerve, seventh cranial nerve, and the corticospinal tract,
resulting in an ipsilateral abduction deficit, facial weakness, and
contralateral hemiplegia (Millard-Gubler syndrome). Lesions
of the cerebellopontine angle may affect CN V (decreased facial
sensation), CN VI (abduction deficit), CN VII (facial weakness),
and CN VIII (decreased hearing and vestibular dysfunction).
Inflammation in the petrous bone may cause an ipsilateral CN
VI palsy and facial pain (Gradenigo syndrome). An abduction
deficit accompanied by proptosis suggests an orbital process
affecting CN VI or the extraocular muscles (thyroid orbitopathy,
orbital pseudotumor).
The cavernous sinus contains cranial nerves III, IV, V1, V2,
and VI, as well as the third-order, postganglionic ocular sympathetic fibers. Any combination of ipsilateral dysfunction of these
cranial nerves suggests cavernous sinus involvement.
The third-order postganglionic sympathetic fibers ascend with
the carotid artery into the cavernous sinus and join CN VI in the
posterior portion of the cavernous sinus.4,5 The fibers then join
the ophthalmic branch of the trigeminal nerve and enter the orbit
through the superior orbital fissure. Therefore, the combination
of a CN VI palsy and an ipsilateral postganglionic Horner syndrome localizes the pathology to the posterior cavernous sinus.
Our patient presented with an isolated CN VI palsy that was
consistent with an ischemic etiology. However, he subsequently
developed an ipsilateral Horner syndrome as well as CN III paresis, suggesting cavernous sinus pathology. Neuroimaging confirmed a cavernous sinus lesion consistent with a meningioma.
Teaching Points
1. Ischemic cranial nerve palsies do not typically affect multiple cranial nerves simultaneously.
2. Ischemic cranial nerve palsies should improve over 3-4
months.
3. Unilateral dysfunction of multiple cranial nerves (III, IV,
V1, V2, and VI) suggests cavernous sinus localization.
4. The sympathetic fibers travel with CN VI in the posterior
cavernous sinus. Therefore, the combination of CN VI
palsy and Horner syndrome suggests cavernous sinus
localization.
5. Controversy exists over whether to image an isolated CN
VI palsy in the vasculopathic older patient after the initial
visit, or to monitor closely.
38
Section III: Double Vision and Nystagmus
References
1. “The patient with diplopia.” In: Neuro-Ophthalmology: Basic and
Clinical Science Course. San Francisco: American Academy of Ophthalmolgy 2014-2015: section 5, chapter 8, 212-221.
2. Murchison AP, Gilbert ME, Savino PJ. Neuroimaging and acute
ocular motor mononeuropathies: a prospective study. Arch Ophthalmol. 2011; 129(3):301-305.
3. Chou KL, Galetta SL, Liu GT, et al. Acute ocular motor mononeuropathies: prospective study of the roles of neuroimaging and clinical assessment. J Neurol Sci. 2004; 219:35-39.
4. Gutman I, Levartovski S, Goldhammer Y, Tadmor R, Findler G.
Sixth nerve palsy and unilateral Horner’s syndrome. Ophthalmology 1986; 93:913-916.
5. Tsuda H, Ishikawa H, Kishiro M, Koga N, Kashima Y. Abducens
nerve palsy and postganglionic Horner syndrome with or without
severe headache. Intern Med. 2006; 45(14):851-855.
“I see two golf club heads when I
putt.”
Courtney E Francis MD
Final Diagnosis
Thyroid eye disease with left inferior rectus restriction
Teaching Points
On further review, the patient had a remote history of Graves
disease, treated with methimazole and currently in remission. He
was followed over time to ensure stability of his deviation for
at least 6 months. He underwent left inferior rectus recession of
6 mm, with resolution of his symptoms. Small deviations may be
managed conservatively with prism glasses.
As alignment may change following surgery, orbital decompressions should always be performed first if clinically indicated
and lid surgery should be reserved until after strabismus surgery.
Care should be taken to avoid overcorrection in cases of inferior
rectus recession, which could lead to diplopia in downgaze.
Additionally, some patients with restrictive strabismus from thyroid eye disease can have late overcorrection after surgery.
Selected Readings
1. Harrad R. Management of strabismus in thyroid eye disease. Eye
2015; 29:234-237.
2. Peragallo JH, Velez FG, Demer JL, Pineles SL. Postoperative drift in
patients with thyroid ophthalmopathy undergoing unilateral inferior rectus muscle recession. Strabismus 2013; 21(1):23-28.
3. Volpe NJ, Mirza-George N, Binenbaum G. Surgical management of
vertical ocular misalignment in thyroid eye disease using an adjustable suture technique. J AAPOS. 2012; 16(6):518-522.
4. Bothun ED, Scheurer RA, Harrison AR, Lee MS. Update on thyroid
eye disease and management. Clin Ophthalmol. 2009; 3:543-551.
5. Chen VM, Dagi LR. Ocular misalignment in Graves disease may
mimic that of superior oblique palsy. J Neuroophthalmol. 2008;
28(4):302-304.
2015 Subspecialty Day | Neuro-Ophthalmology
“Things are blurry and jumpy when I
read.”
Janet C Rucker MD
Final Diagnosis
Paraneoplastic cerebellar degeneration with downbeat nystagmus and gait ataxia and positive paraneoplastic antibodies due
to recurrent ovarian cancer
Teaching Points
Teaching Point 1: Nystagmus
Nystagmus is a spontaneous, repetitive movement of the eyes
that is initiated by slow eye drifts away from desired eye position. It may be jerk (slow drifts followed by corrective fast movements in the opposite direction) or pendular (to-and-fro slow
oscillations without fast corrective movements). Jerk nystagmus
is named by the direction of its fast movements (eg, “right beating,” “up beating,” etc.). Both jerk and pendular nystagmus may
have horizontal, vertical, and/or torsional components. Oscillopsia, a subjective sensation of visual motion, is often experienced
by the patient with nystagmus and may be described as “jumping
vision” or simply as “blurred vision.”
Three main types of nystagmus are commonly seen in primary
gaze with motion in the vertical direction. The first two are jerk
nystagmus: upbeat nystagmus (UBN) and downbeat nystagmus
(DBN). The third is acquired pendular nystagmus (APN). UBN
consists of slow drifts of the eyes downward with corrective fast
movements upwards. It is, by definition, present in primary gaze
and is typically more prominent in upgaze. UBN is much less
common than DBN and APN and is most typically seen in multiple sclerosis and Wernicke encephalopathy.
DBN consists of slow drifts of the eyes upward with corrective fast movements downward. It is, by definition, present
in primary gaze, but it may be of such low amplitude that it is
invisible to the naked eye. It is much more prominent in lateral,
downward gaze (and is thus sometimes called “side-pocket nystagmus”). It is often absent or converted to upbeat nystagmus in
upgaze. A chin-down resting head position may be adopted by
the patient to minimize oscillopsia. Strategies to optimize visualization of DBN in primary gaze include viewing the eye at the slit
lamp with careful attention to scleral vessels for fast-downbeat
motion and assessing for DBN during dilated ophthalmoscopy.
DBN is idiopathic in a large percentage of cases, especially when
it is an isolated finding. Other common causes include Chiari
I malformation, other cervicomedullary junction pathology,
and substance toxicities (eg, anti-convulsants, lithium, alcohol).
DBN is also commonly seen in association with cerebellar disease from structural abnormalities on MRI (eg, stroke, tumor),
genetic spinocerebellar degenerations, or with neurologically
active antibodies in the serum (eg, anti-GAD antibodies, paraneoplastic antibodies). The best-studied treatments for DBN are
the potassium channel blockers 4-aminopyridine (4-AP) and
3,4-diaminopyridine (3,4-DAP), in addition to chlorzoxazone
and gabapentin.
APN consists of slow pendular oscillations of the eyes without
fast corrective movements. It is commonly seen in two scenarios:
multiple sclerosis and oculopalatal tremor (OPT). In OPT, it
accompanies tremor of the palate and is typically seen weeks to
months following a stroke in the brainstem. MRI in OPT reveals
2015 Subspecialty Day | Neuro-Ophthalmology
increased T2 signal in and hypertrophy of the inferior olives in
the medulla. APN in MS often has a horizontal or elliptical trajectory. In OPT, it is often vertical. The two most effective treatments for APN are gabapentin and memantine.
Teaching Point 2: Paraneoplastic syndromes
Paraneoplastic syndromes cause cancer-related, immunemediated, acute to subacute decline in nervous system function
remote from a cancer site. They are most commonly associated
with small-cell cancers such as lung cancer but are also often
seen with ovarian and breast carcinoma and testicular germ cell
tumors. Typical paraneoplastic syndromes that involve vision
include optic neuropathy, retinopathy, cerebellar degeneration
with downbeat nystagmus, and opsoclonus. Paraneoplastic
antibodies are commonly present in serum. It is very important
to be aware that the cancer is often occult and undiagnosed at
time of onset of the visual or neurological symptoms, and intensive, exhaustive search for an underlying malignancy is critical.
If no malignancy is identified and a paraneoplastic syndrome is
strongly suspect, periodic surveillance for malignancy should be
performed annually for at least 5 years.
Section III: Double Vision and Nystagmus
39
signs or symptoms, or history of onset after trauma are
important. In this setting, neurologic workup with imaging and/or lumbar puncture may be helpful.
Selected Readings
1. Liu GT, Volpe NJ, Galetta SL. Eye movement disorders: third,
fourth, and sixth nerve palsies and other causes of diplopia and
ocular misalignment. In: Neuro-ophthalmology: Diagnosis and
Management, 2nd ed. New York: Saunders Elsevier; 2010:491-550.
2. Kline LB, Foroozan R. Supranuclear and internuclear gaze pathways. In: Neuro-Ophthalmology Review Manual, 7th ed. Thorofare, NJ: SLACK Inc.; 2013:45-72.
3. Jacobson DJ. Divergence insufficiency revisited: natural history of
idiopathic cases and neurologic associations. Arch Ophthalmol.
2000; 118:1237-1241.
4. Chaudhuri Z, Demer JL. Sagging eye syndrome: connective tissue
involution as a cause of horizontal and vertical strabismus in older
patients. JAMA Ophthalmol. 2013; 131(5):619-625.
5. Mittelman D. Age-related distance esotropia. J AAPOS. 2006;
10(3):212-213.
Selected Readings
1. Mehta AR, Kennard C. The pharmacological treatment of acquired
nystagmus. Pract Neurol. 2012; 12:147-153.
2. Thurtell MJ, Leigh RJ. Nystagmus and saccadic intrusions. Handb
Clin Neurol. 2011; 102:333-378.
3. Rucker JC. An update on acquired nystagmus. Semin Ophthalmol.
2008; 23:91-97.
4. Ko MW, Dalmau J, Galetta SL. Neuro-ophthalmologic manifestations of paraneoplastic syndromes. J Neuroophthalmol. 2008;
28:58-68.
“Words run together on the TV.”
Kimberly M Winges MD
Final Diagnosis
Divergence insufficiency, secondary to spinocerebellar ataxia
Teaching Points
1. Divergence insufficiency (DI) is defined as a comitant esophoria or esotropia worse at distance than near (or absent
at near), with full ductions and no evidence of cranial
nerve palsy.
2. DI can be secondary to neurologic causes, or it can be primary (idiopathic). A poorly defined divergence center in
the caudal pons is hypothesized to be responsible for neurologic divergence paralysis. More recently, involutional
changes within the orbital connective tissue and extraocular muscles have been recognized as non-neurologic causes
of distance esotropia in older individuals. Most patients
with DI in isolation do not develop future neurologic disease.
3. The main differential diagnosis of DI is bilateral CN VI
palsy, and careful attention to signs of increased intracranial pressure, other cranial nerve involvement, neurologic
“I see double when I get tired.”
Marc J Dinkin MD
Final Diagnosis
Myasthenia gravis
Teaching Points
1. Congenital strabismus may become symptomatic when the
patient is fatigued, such as at the end of the day.
2. Myasthenia can mimic any eye movement pattern.
3. Other strabismus patterns that suggest a congenital origin
include an intermittent V-pattern esotropia, pure inferior
oblique overaction, a limitation of elevation of an eye
when adducted (suggestive of a Brown syndrome), or
limitation of abduction of an eye (suggestive of a Duane
syndrome).
4. Specific exam features that help confirm a congenital origin are:
• High fusional amplitude: When increasing prism is
added in the vertical plane, for example, above and
beyond what is needed to remove the phoria, the
patient with congenital strabismus will often continue
to be able to fuse the two images, without diplopia,
over a large number of additional prism diopters. In
contrast, the myasthenic patient, who has not had a
lifetime to cortically adapt to the strabismus, will typically not have a high fusional amplitude.
• The finding of a compensatory head turn or tilt on old
photos
• In the case of a limitation of abduction, the presence of
retraction on adduction is highly suggestive of Duane’s
syndrome.1
• In the case of limited elevation on adduction, a positive
forced duction test suggests a restriction of the superior
oblique tendon consistent with Brown syndrome.
40
Section III: Double Vision and Nystagmus
5. Fatigable ptosis on sustained upgaze suggests ocular myasthenia but may also be present in up to 38% of patients
who do not have the disease.2
6. The onset of a new fatigable ptosis over a background of
fatigable diplopia is highly suggestive of myasthenia gravis, making decompensation of a congenital strabismus as
the cause of the diplopia much less likely. However, some
patients presenting with a decompensation of a congenital
strabismus may also have ptosis from other causes, most
often mechanical. And in fact, such a lid may also be more
difficult to keep up when the patient is tired. Thus, the
combination of ptosis and fatigable diplopia should not be
looked at as pathognomonic for myasthenia.
7. Lack of systemic fatigable weakness is consistent with a
diagnosis of ocular myasthenia, although in a recent study,
20.9% of patients will convert to generalized myasthenia
over the next two years.3
8. The presence of enhanced ptosis (curtaining) is not specific to ocular myasthenia, as it reveals a weakness of the
apparently uninvolved levator palpebrae muscle which is
brought out by passive lifting of the ptotic lid,4 although
it has been described in other conditions as well.5 This is
an example of Herring’s Law: that the force put into one
extraocular muscle is equal in both eyes. Cogan lid twitch
is another finding classic for ocular myasthenia6 but is not
ubiquitously observed.7
9. The Tensilon test has a high specificity for myasthenia but
may cause syncope or, rarely, cardiac arrhythmias.8
10. The rest test is a sensitive and specific assessment for
ocular myasthenia but requires rigorous documentation
before and after a 20-minute session of eye closure.9
11. A decrement on repetitive nerve stimulation (RNS) testing is highly specific for myasthenia,10 but the test has a
sensitivity as low as 24%2 so that a lack of decrement does
not rule out myasthenia. Single fiber EMG may reveal
increased jitter in cases of OMG, and has a higher sensitivity (90%-95%) than RNS,11 but it requires arduous and
time-consuming testing and is only performed by select
experts.
12. Acetylcholine receptor antibodies may be elevated in pure
ocular myasthenia, but sensitivity is low, ranging from
14.1%12 to 50%.13
13. Some patients with ocular myasthenia harbor a thoracic
thymoma, and in rare cases the tumor may be malignant,14
or even more rarely, as in this case, be composed of amyloid.15,16
14. Other entities that may mimic ocular myasthenia because
of a diurnal variability include Miller Fisher syndrome17
and silent sinus syndrome.18 The latter is associated with
hypoglobus, or inferior positioning of the globe, due to
an inferior bowing of the orbital flow into a low pressure
maxillary sinus, thus producing a vertical diplopia.19 Such
“sinking” may worsen later in the day due to the effects of
gravity, thus mimicking the fatigability of myasthenia.
References
1. Hotchkiss MG, Miller NR, Clark AW, Green WR. Bilateral
Duane’s retraction syndrome: a clinical-pathologic case report.
Arch Ophthalmol. 1980; 98(5):870-874.
2. Mittal MK, Barohn RJ, Pasnoor M, et al. Ocular myasthenia gravis
in an academic neuro-ophthalmology clinic: clinical features and
therapeutic response. J Clin Neuromuscul Dis. 2011; 13(1):46-52.
2015 Subspecialty Day | Neuro-Ophthalmology
3. Nagia L, Lemos J, Abusamra K, Cornblath WT, Eggenberger ER.
Prognosis of ocular myasthenia gravis: retrospective multicenter
analysis. Ophthalmology. Epub ahead of print 2015 Apr 16. doi:
10.1016/j.ophtha.2015.03.010.
4. Gorelick PB, Rosenberg M, Pagano RJ. Enhanced ptosis in myasthenia gravis. Arch Neurol. 1981; 38(8):531.
5. Ishikawa H, Wakakura M, Ishikawa S. Enhanced ptosis in Fisher’s
syndrome after Epstein-Barr virus infection. J Clin Neuroophthalmol. 1990; 10(3):197-200.
6. Cogan DG. Myasthenia gravis: a review of the disease and a
description of lid twitch as a characteristic sign. Arch Ophthalmol.
1965; 74:217-221.
7. Van Stavern GP, Bhatt A, Haviland J, Black EH. A prospective
study assessing the utility of Cogan’s lid twitch sign in patients
with isolated unilateral or bilateral ptosis. J Neurol Sci. 2007;
256(1Y2):84-85.
8. Osserman KE, Kaplan LI. Rapid diagnostic test for myasthenia
gravis: increased muscle strength, without fasciculations, after intravenous administration of edrophonium (Tensilon) chloride. J Am
Med Assoc. 1952; 150(4):265Y268.
9. Odel JG, Winterkorn JM, Behrens MM. The sleep test for myasthenia gravis: a safe alternative to Tensilon. J Clin Neuroophthalmol.
1991; 11(4):288.
10. Benatar M. A systematic review of diagnostic studies in myasthenia
gravis. Neuromuscul Disord. 2006; 16:459-467.
11. Padua L, Stalberg E, LoMonaco M, et al. SFEMG in ocular myasthenia gravis diagnosis. Clin Neurophysiol. 2000; 111:1203-1207.
12. Lee JJ, Koh KM, Kim US. The anti-acetylcholine receptor antibody
test in suspected ocular myasthenia gravis. J Ophthalmol. 2014;
2014:689792.
13. Roh HS, Lee SY, Yoon JS. Comparison of clinical manifestations
between patients with ocular myasthenia gravis and generalized
myasthenia gravis. Korean J Ophthalmol. 2011; 25(1):1-7.
14. Tung CI, Chao D, Al-zubidi N, et al. Invasive thymoma in ocular
myasthenia gravis: diagnostic and prognostic implications. J Neuroophthalmol. 2013; 33(3):307-308.
15. Son SM, Lee YM, Kim SW, Lee OJ. Localized thymic amyloidosis
presenting with myasthenia gravis: case report. J Korean Med Sci.
2014; 29(1):145-148.
16. Chapman KO, Beneck DM, Dinkin MJ. Ocular myasthenia gravis
associated with thymic amyloidosis. J Neuroophthalmol. Epub
ahead of print 2015 Mar 27.
17. Anthony SA, Thurtell MJ, Leigh RJ. Miller Fisher syndrome
mimicking ocular myasthenia gravis. Optom Vis Sci. 2012;
89(12):e118-123.
18. Coombs PG, Mitchell J, Lelli G, Dinkin MJ. A case of silent sinus
syndrome caused by a dacryocystorhinostomy presenting as myasthenia gravis. North American Neuro-ophthalmology Society.
2014; Abstract 91.
19. Saffra N, Rakhamimov A, Saint-Louis LA, Wolintz RJ. Acute diplopia as the presenting sign of silent sinus syndrome. Ophthal Plast
Reconstr Surg. 2013; 29(5):e130Y-131.
2015 Subspecialty Day | Neuro-Ophthalmology
“I see double after my cataract
surgery.”
Stacy L Pineles MD
Final Diagnosis
The presence of diplopia combined with a history of childhood
amblyopia and strabismus raises the suspicion for fixation switch
diplopia.
Teaching Points
Fixation switch diplopia should be suspected in patients with a
history of childhood strabismus or amblyopia who become diplopic after an intervention forces them to switch fixation to their
previously amblyopic eye. Treatment typically involves restoration of fixation with the previously dominant eye using optical or
surgical means. Prisms can also be used as a short-term solution
if necessary in cases where there is coexistent strabismus.
Section III: Double Vision and Nystagmus
41
42
Section IV: Test Interpretation
2015 Subspecialty Day | Neuro-Ophthalmology
Section IV: Test Interpretation Made
Ridiculously Simple
What Do I Do With This Visual Field?
Gregory P Van Stavern MD
Final Diagnosis
Suprasellar meningioma compressing the intracranial optic nerve
and optic chiasm, causing a junctional scotoma
Teaching Point 1
The visual field is one of the most important tools in the ophthalmologist’s armamentarium, as it allows precise localization
of pathology within the afferent visual pathways. The retinal
nerve fiber layer is highly topographically organized, and that
topography is preserved throughout the visual pathway: a lesion
anywhere from the retina to the primary visual cortex will cause
predictable visual field defects, which can then narrow the differential diagnosis of the patient’s visual complaints and guide
diagnostic workup.
Visual field testing by perimetry should be performed in
any patient with unexplained visual loss and is a key element
in monitoring patients with known disease such as glaucoma
and idiopathic intracranial hypertension. All perimetry methods
are prone to inter-test variability, interoperator variability, and
performance failures, and should be interpreted in the context of
the entire examination and the results of confrontation testing.
Indeed, careful confrontation perimetry can be a great ally, as a
completely normal confrontation field (particularly when tested
using a small red target) is inconsistent with extensive visual field
loss seen on automated or kinetic perimetry, and should raise
suspicion of artifact or performance issues.
Patterns of visual field loss are highly localizing, so accurate
interpretation relies upon basic knowledge of visual pathway
neuroanatomy. In particular, the vertical meridian should be
carefully examined. The optic chiasm is the anatomic substrate
for the vertical meridian, and any visual field defect that respects
the vertical midline raises concern for a chiasmal or retrochiasmal lesion. Automated static perimetry is probability-based, and
the defects seen are not always as clean or “pretty” as those seen
with kinetic perimetry, so it is important to review the field carefully to see if there is any portion of the defect that lines up along
the vertical midline, even if there is some apparent crossover on
the probability plots. When interpreting any visual field, it is
helpful to ask the following questions:
1.
2.
3.
4.
5.
Is the test reliable?
Is the test normal?
Does the visual field defect involve one eye or both eyes?
If both eyes, does the defect respect the vertical meridian?
If the defects respects the vertical meridian, is it bitemporal
or junctional (optic chiasm) or homonymous (retrochiasmal)?
Sellar and suprasellar tumors may cause intracranial optic
nerve as well as chiasmal compression. Therefore, with such
tumors it is not rare to see visual field defects that reflect optic
nerve, chiasmal, and optic tract compression, and interpreting
these defects can be challenging. Lesions that affect the anterior
angle of the chiasm and the distal optic nerve produce a junctional scotoma, a distinct syndrome characterized by an optic
nerve-related defect in the eye ipsilateral to the tumor, and a
superotemporal defect in the eye contralateral to the mass. The
contralateral temporal visual field defect is usually superotemporal (reflecting the inferonasal crossing fibers) but may be a complete temporal hemianopia.
The origin of the superotemporal, contralateral visual field
defect is of both historical and clinical interest. Wilbrand proposed that crossed fibers originating from ganglion cells inferior
and nasal to the fovea in the contralateral eye extend anteriorly
(< 2 mm) into the involved optic nerve and are thus subject to
compression. This anatomic configuration is known as “Wilbrand’s knee.” Although some believe that Wilbrand’s knee is
simply a pathologic artifact rather than a true anatomic structure, its clinical relevance remains unquestioned, and the finding
of a “junctional” visual field defect provides strong evidence of
a lesion at the anterior angle of the chiasm, mandating neuro­
imaging.
Neuroimaging is indicated for all patients suspected of having a compressive optic neuropathy. The ideal modality is MRI
of the brain, with and without gadolinium, with fat-saturated
orbital views. This offers superb views of the globe, optic nerve
head, optic nerve sheath, extraocular muscles, and orbital apex.
It is also superior to CT for the evaluation of the intracanalicular
and intracranial optic nerve, the pituitary fossa, and the cavernous sinuses.
Treatment is dependent upon the specific type of tumor and
the location, size, and degree of optic nerve dysfunction. The
prognosis for visual outcome is related in part to duration of
compression, as well as retinal ganglion cell and retinal nerve
fiber layer integrity. Patients with severe optic atrophy at presentation (indicating loss of neurons and axons) have a worse
prognosis for recovery, although some of these patients may have
partial improvement after decompression.
Teaching Point 2
The diagnosis of “previous NAION” should be used with caution unless the patient had documented optic nerve swelling
at presentation and a history and subsequent clinical course
compatible with that diagnosis. It is important to remember that
NAION is ultimately a clinical diagnosis, supported by features
such as acute onset, optic disc swelling, presence of a small,
crowded optic disc, and subsequent stabilization with spontaneous resolution of optic disc edema. There are several “red flags”
in the history and examination that suggest a diagnosis other
than a previous episode of NAION:
1.
2.
3.
4.
5.
No documentation of a swollen optic disc
Gradual rather than acute onset
Lack of a small, crowded optic disc (cup-to-disc < 0.2)
Visual field defect that respects the vertical meridian
Progression of vision loss in the absence of disc swelling
Section IV: Test Interpretation
2015 Subspecialty Day | Neuro-Ophthalmology
The ophthalmologist should have a high index of suspicion
for a compressive optic neuropathy in such patients, for the following reasons:
1. Early detection of a mass may result in better prognosis for
vision and survival (if neoplastic).
2. In many cases, the mass lesion may pose a risk to the unaffected eye.
3. Visual loss may recover once the compressive lesion is
treated.
Indeed, compressive optic neuropathies are among the
most treatable forms of optic nerve dysfunction, and dramatic
recovery may occur with decompression of the visual pathways.
Therefore, early detection is critical, as the visual loss may be
reversible. The presence of a junctional scotoma in this case
strongly indicated that a compressive lesion was present.
Selected Readings
1. Volpe NJ. Compressive and infiltrative optic neuropathies. In:
Walsh and Hoyt’s Clinical Neuro-Ophthalmology, 6th ed. Philadelphia: Lippincott Williams and Wilkins; 2005:385-430.
2. Foroozan R. Chiasmal syndromes. Curr Opin Ophthalmol. 2003;
14(6):325-331.
3. Horton J. Wilbrand’s knee of the primate optic chiasm is an artefact
of monocular enucleation. Trans Am Ophthalmol Soc. 1997; 95:131.
4. Lee AG, Chau FY, Golnik KC, et al. The diagnostic yield for the
evaluation of isolated unexplained optic atrophy. Ophthalmology
2005;112:757-759.
Should I Trust My Exam, or the OCT?
2. Retinal nerve fiber layer and macular OCT (segmentation)
When interpreting the OCT, it is important to review not only
the retinal nerve fiber layer but the macular thickness maps and
the individual cross-sectional images. The thickness of the retinal
nerve fiber layer is influenced by not only the integrity of the
axons of the ganglion cells but glial cells and blood vessels.2 The
macular thickness map and retinal segmentation can provide
valuable information regarding the anatomical / structural status
of the different layers of the retina. Certain disease processes
have a predilection for particular layers of the retina. For example, retinal artery occlusion causes thinning of the inner layers of
the retina, and acute macular neuroretinopathy (AMNR) affects
the outer retinal layers.3 Furthermore, it has been shown that
macular OCT findings can aid in differentiating postacute retinal
artery occlusion from nonacute optic neuropathies. Ghazi et al
performed a retrospective OCT study of 17 eyes with postacute
retinal artery occlusion and 32 eyes with nonacute optic neuro­
pathy.4 They found 3 main features that distinguished postacute
retinal artery occlusion from nonacute optic neuropathy:
1. Complete inner retinal atrophy with loss of the normal
stratification of the inner retinal layers
2. Loss of the normal foveal depression
3. Marked thinning of the involved retina compared to nonacute optic neuropathy
Furthermore, Dolan et al demonstrated that macular thinning
was more profound in eyes with retinal artery occlusion than
in eyes with nonarteritic anterior ischemic optic neuropathy.5
Improved resolution with currently available spectral domain
OCT allows for the segmentation of the various layers of the retina, thereby distinguishing localized retinal nerve fiber layer and
ganglion cell layer thinning associated with optic neuropathies
from the more diffuse inner retinal layer thinning associated with
retinal artery occlusion.
M Tariq Bhatti MD
References
Final Diagnosis
1. Newman NJ. Optic disc pallor: a false localizing sign. Surv Ophthalmol. 1993; 37(4):273-282.
Branch retinal artery occlusion with secondary optic nerve
­atrophy.
2. Kardon RH. Role of the macular optical coherence tomography scan in neuro-ophthalmology. J Neuroophthalmol. 2011;
31(4):353-361.
Teaching Points
3. Fawzi AA, Pappuru RR, Sarraf D, et al. Acute macular neuroretinopathy: long-term insights revealed by multimodal imaging.
Retina 2012; 32(8):1500-1513.
1. Primary optic nerve pallor vs. secondary optic nerve pallor
Optic nerve pallor can be a false localizing sign because not all
cases of optic nerve pallor are due to primary optic nerve pathology.1 Retinal dystrophies (in particular cone dystrophies), retinal
degenerations (ie, retinitis pigmentosa), and retinal artery occlusions can all result in secondary optic nerve pallor. In some cases,
especially if nonacute, it can be clinically challenging to differentiate primary optic nerve pallor from secondary optic nerve
pallor. Prior to the development of OCT, intravenous fluorescein
angiography and electrophysiology, in addition to the clinical
examination, were required. The addition of OCT to the armamentarium of paraclinical tests allows for quick identification of
retinal pathology that otherwise might be below the threshold of
funduscopic examination.2
43
4. Ghazi NG, Tilton EP, Patel B, Knape RM, Newman SA. Comparison of macular optical coherence tomography findings between
postacute retinal artery occlusion and nonacute optic neuropathy.
Retina 2010; 30(4):578-585.
5. Dotan G, Goldenberg D, Kesler A, Naftaliev E, Loewenstein A,
Goldstein M. The use of spectral-domain optical coherence tomography for differentiating long-standing central retinal artery occlusion and nonarteritic anterior ischemic optic neuropathy. Ophthalmic Surg Lasers Imaging Retina. 2014; 45(1):38-44.
44
Section IV: Test Interpretation
“I can’t get cocaine drops in my
office!”
Heather E Moss MD PhD
Final Diagnosis
Small cell lung cancer of the right lung apex causing right Horner
syndrome
Teaching Points
1. Physical exam findings are free and quick tests that guide
the differential diagnosis of ptosis and anisocoria and can
be diagnostic of Horner syndrome.
2. Eye drop testing is helpful to confirm an equivocal diagnosis of Horner syndrome or in a patient where neuroimaging evaluation poses risks.
3. The most important reason to diagnose Horner syndrome
is to prompt workup for treatable etiologies:
a. If recent medical history provides an etiology, no additional testing may be needed.
b. Imaging of the sympathetic chain will screen for lesions
causing Horner syndrome.
Selected Readings
1. Almog Y, Gepstein R, Kesler A. Diagnostic value of imaging in
Horner syndrome in adults. J Neuroophthalmol. 2010; 30:7-11.
2. Davagnanam I, Fraser CL, Miszkiel K, Daniel CS, Plant GT. Adult
Horner’s syndrome: a combined clinical, pharmacological, and
imaging algorithm. Eye 2013; 27:291-298.
3. Trobe J. The evaluation of Horner syndrome. J Neuroophthalmol.
2010; 30:1-2.
“The MRI is abnormal; now what?”
Fiona E Costello MD
Diagnosis
Repeat review of the MR-venogram reveals the diagnosis of cerebral venous sinus thrombosis as the cause of the patient’s seizures
and manifestations of raised intracranial pressure.
Teaching Points
Cerebral venous sinus thrombosis (CVST) is a form of stroke
whereby thrombosis occurs in the cerebral venous sinuses or
veins.1 The incidence of CVST has been estimated at 3 to 5 cases
per million population per year, representing 0.5%- 1% of all
strokes.1 This condition tends to affect individuals aged less than
50 years, and is 3 times more common in women than men.1
CVST frequently affects patients with underlying risk factors for
venous clot formation, including pregnancy, oral contraceptive
pill use, systemic disease (eg, Crohn disease), infection (eg, mastoid sinus disease), and malignancy.1-2
The clinical features of CVST may vary and evolve over time.
Thrombosis or stenosis of cerebral venous sinuses results in
intracranial venous hypertension, which causes raised intracranial pressure by reducing passive cerebrospinal fluid resorption
2015 Subspecialty Day | Neuro-Ophthalmology
through the arachnoid villi.3 Venous hypertension is common in
CVST patients, who often manifest symptoms and signs of raised
intracranial pressure including headaches, papilledema, and
altered mental status.3 Additional features on examination may
include cranial nerve palsies, focal neurological deficits, seizures,
encephalopathy, and coma.2
Radiographic features of CSVT depend largely on the imaging modality utilized. Noncontrast computed tomography (CT)
imaging of the head may show venous hemorrhage or infarction.2 Occasionally hyperdense signal changes within the affected
sinus may be identified.2 With contrast administration and CT
venography, a filling defect in a sinus may be seen. MRI may
show venous thrombosis that is isointense on T1- and hypointense on T2-weighted imaging, thus mimicking a “flow void”;
subacute venous blot may become hyperintense on T1-weighted
imaging. Notably, 2-D time-of-flight MR-venography is routinely performed in suspected cases, albeit contrast MR venography is generally more sensitive in detecting CVST. Although
digital subtraction angiography has historically been the gold
standard, the relative lack of experienced angiographers and the
invasive nature of the examination has led to a dramatic decline
in its use.2 Despite major improvement in MRI techniques over
the past decade, misinterpretation of brain MRI resulting in
delayed diagnosis of CVST remains common.3 Imaging of the
intracranial venous system with MR-venography is not systematically requested by many clinicians evaluating patients with a
syndrome of isolated raised intracranial pressure, and radiologists are often asked to “rule-out CVST” on an isolated brain
MRI.3 This may lead to false negative results. For the general
ophthalmologist, CT or MR venography techniques are useful in
the evaluation of patients with suspected raised intracranial pressure, particularly for those with atypical clinical features. In the
case presented, the patient was obese, but her weight loss, underlying comorbidities, and seizures argued against the diagnosis of
idiopathic intracranial hypertension and in favor of CVST.
The mainstay of treatment for CVST is anticoagulation, even
in the setting of hemorrhagic venous infarction. The natural
history of CVST is highly variable, with some patients having
minimal or no symptoms and an uneventful recovery (~65%),
whereas others have a fulminant course culminating in extensive
venous infarction, blindness, dependency, or death (~20%).2
Interventional neuroradiologists can perform catheter-directed
thrombolysis by using targeted thrombolytics in the affected
sinuses. As expected, hemorrhagic venous infarcts and coexisting
malignancy correlate with poor outcome.2 Deep cerebral venous
thrombosis also has a negative impact on prognosis due to what
is often bilateral involvement of the thalami.2 Dural arteriovenous
fistula and increased cerebrospinal fluid pressure can also lead to
long-term complications after cerebral venous thrombosis.2
References
1. Amoozegar F, Ronksley PE, Sauve R, Menon B. Hormonal contraceptives and cerebral venous thrombosis risk: a systematic review
and meta-analysis. Front Neurol. Published online 2015 Feb 2. doi:
10.3389/fneur.2015.00007.
2. Di Muzio B, Gaillard F, et al. Cerebral venous thrombosis. Radiopaedia. N.d. Available at: http://radiopaedia.org/articles/cerebralvenous-thrombosis.
3. Ridha MA, Saindane AM, Bruce BB, et al. MRI findings of elevated
intracranial pressure in cerebral venous thrombosis versus idiopathic intracranial hypertension with transverse sinus stenosis.
Neuroophthalmol. 2013; 37(1):1-6.
2015 Subspecialty Day | Neuro-Ophthalmology
Everyone’s ESR Is High: Who Needs
a Biopsy?
Todd Alan Goodglick MD
Final Diagnosis
Giant cell arteritis
Teaching Points
The diagnosis of giant cell arteritis (GCA) is straightforward in
textbook cases where an elderly patient presents with temporal
headache, scalp tenderness, classic jaw claudication (ie, crescendo pain with chewing), and sudden loss of vision. However,
this diagnosis comes up far more frequently in the differential
diagnosis for variations of these symptoms such as atypical headache, transient vision loss, double vision, or even the incidental
finding of an elevated sedimentation rate, platelet count, or
C-reactive protein. This of course is particularly true when trying to make the diagnosis early to prevent the dreaded outcome
of permanent vision loss (incidence in GCA is 15%-30% of
cases). Eye doctors in particular need to maintain a high level of
vigilance, as 20% of GCA patients present with only ophthalmologic complaints.
What is known is that GCA is an inflammatory vasculopathy
involving medium and large arteries including branches of the
external carotids; ophthalmic, vertebral, and distal subclavian
arteries; and thoracic aorta. There is an association with the disease polymyalgia rheumatica (PMR), which may pre- or postdate
the diagnosis of GCA and cause diffuse myalgias of the neck
and proximal extremities. About 50% of GCA patients will also
attain a diagnosis of PMR at some point. PMR is 10 times as
common as GCA, so PMR cannot be a highly specific risk factor for GCA even though a diagnosis of PMR often raises such
a concern. Similarly, ethnicity can be used only as a relative risk
factor, with the incidence in northern European whites twice as
high as those of southern European descent and 20 times that
in patients of Asian and African descent.1 Because of the lack of
knowledge about the etiology of the disease or until there are
sensitive and specific biomarkers for disease activity, there will
always be ambiguous cases.
The sensitivity of an elevated age-corrected sed rate (Westergren erythrocyte sedimentation rate (ESR) (age + 10 for men
or age divided by 2 for women) in biopsy-confirmed GCA has
been estimated at 84%; and that for elevated C-reactive protein
(CRP), 86%. The specificity of these values was low, measuring
30% for either. Furthermore, only 4% of biopsy-proven patients
had a normal value for both.
Put another way, a high ESR is a nonspecific indicator of
inflammation, and a normal ESR is reported in up to 20% of
cases of GCA. The combination of elevated ESR and CRP has a
sensitivity of 98%. The odds ratio predicting a positive temporal
artery biopsy was 5 times for an elevated CRP, 4 times greater
for thrombocytosis, and 1.5 times greater for an elevated ESR.2
However, these numbers (and all such statistics in this disease)
are truly applicable only in a population with a proven diagnosis
as opposed to the population encountered in clinical settings
with variably suspicious symptoms. Clinicians are thus justifiably
confused when it comes to ruling in or out a diagnosis of GCA.
In all cases the diagnosis of GCA is best made, when possible,
histologically rather than clinically. Even in the most typical clin-
Section IV: Test Interpretation
45
ical scenarios the specificity of a pathology-supported diagnosis
carries significant weight when the risks of prolonged steroid
treatment are considered or encountered months later. A positive
biopsy heightens attention to atypical or vague symptoms that
might have been ignored otherwise. The sensitivity of a temporal
artery biopsy, even when done in a standardized manner, is not
known as this number would also depend on standardizing a
“pretest” population of patients with respect to suspicious symptoms and signs of the disease and an alternative gold standard for
the diagnosis. Without a better understanding of the pathogenesis of the disease this is not presently available. What is known
is that a specific size of biopsy specimen would be expected to
increase the sensitivity as skip lesions of foci of inflammation are
known to commonly occur. A length of a fixed biopsy > 1.5–2.0
cm has often been found to be associated with a higher positive
rate.
There is a trend to diminish the utility of a temporal artery
biopsy due to the perception of a (probably overestimated) false
negative rate.3 When properly done the false negative rate is,
although unknown, believed to be low. A properly done biopsy
involves obtaining a piece of artery, localized to the pain and
about 2 cm long given the possibility of skip lesions of inflammatory foci, which are known to occur frequently. Positive results
would include the presence of a mononuclear infiltrate throughout the wall of the muscular artery with characteristic but not
mandatory giant cells and destruction of the internal elastic
lamina.
The presence of giant cells distinguishes GCA from non-giant
cell arteritides such as ANCA-positive vasculitis. Findings suggestive of atherosclerotic changes would not be considered positive.
A biopsy of the superficial temporal artery is a straightforward outpatient procedure. The notion of a substantial false
negative rate for temporal artery biopsies thus needs to be interpreted with caution but (somewhat) limits the utility of such a
result in ruling out the disease. A negative result in the presence
of classic symptoms might be regarded as a false negative and
treatment continued, but otherwise a negative result can be used
as a strong indicator against this diagnosis and a positive result
makes the diagnosis unquestionable. A negative result suggests
more than an elimination of a diagnosis of GCA, and itself has a
differential diagnosis.4
Part of the confusion about the diagnosis has arisen from the
American College of Rheumatology diagnostic criteria for this
disease, in which 3 of 5 criteria need to be met for a diagnosis,
only 1 of which is a positive biopsy. These criteria, however,
were created in order to differentiate the various vasculitides and
are of limited use in typical clinical diagnostic situations.3
Simultaneous biopsy of both sides is not necessarily recommended routinely as it is thought to increase the yield of a positive diagnosis by 1%-4%. Biopsy of the second side should be
considered if there is a significant degree of suspicion for the
disease despite an initial negative biopsy, given the usefulness of
a tissue diagnosis. Frozen section of the artery has an 18% false
negative rate and cannot be relied upon.5
The approach should be “treat and then biopsy,” using
response to steroids as a significant and highly typical clinical
occurrence. The histologic findings are thought to persist for at
least 2-3 weeks after initiation of steroids, with evidence that
changes of healed arteritis may be found up to 1 year later.
In conclusion, the difficulties in making the diagnosis of GCA
stem from our present lack of understanding of the etiology and
the lack of specific markers of that process. There are presently
no imaging modalities (ultrasound, MRI, PET) that have reli-
46
Section IV: Test Interpretation
able diagnostic sensitivity or specificity.1 What remains is a high
level of suspicion and the temporal artery biopsy, including in
situations where systemic symptoms are being evaluated such as
fever of unknown origin, generalized myalgias, night sweats, and
weight loss, all primarily aimed at trying to prevent the devastating loss of vision that this disease can cause.
References
1. Weyand CM, Goronzy JJ. Giant cell arteritis and polymyalgia rheumatica. N Engl J Med. 2014; 371(17):50-57.
2. Walvick MD. Giant cell arteritis: laboratory predictors of a positive
temporal artery biopsy. Ophthalmology 2011; 118:1201-1204.
3. Danesh-Meyer H. Temporal artery biopsy: skip it at your patient’s
peril [editorial]. Am J Ophthalmol. 2012; 154 (4):617-619.
4. Roth AM, Milsow L, Keltner JL. The ultimate diagnoses of patients
undergoing temporal artery biopsies. Arch Ophthalmol. 1984;
102:901-903.
5. Taylor-Gjevre R, Vo M, Shukla D, Resch L. Temporal artery
biopsy for giant cell arteritis. J Rheumatol. 2005; 32:1279-1282.
2015 Subspecialty Day | Neuro-Ophthalmology
2015 Subspecialty Day | Neuro-Ophthalmology
47
Financial Disclosure
Transparency through disclosure of relationships with companies is one step in the Academy’s process of ensuring that all its
educational activities are fair, balanced, and not commercially
biased. The Academy’s Board of Trustees supports the position
that having a financial relationship should not restrict expert
scientific, clinical or non-clinical presentation or publication or
participation in Academy leadership or governance, provided
that appropriate disclosure of such relationship is made. Similarly, it should not restrict participation in AAO leadership or
governance, so long as appropriate disclosure is made. As an
ACCME accredited provider of CME, the Academy seeks to
ensure balance, independence, objectivity, and scientific rigor in
all individual or jointly sponsored CME activities.
All contributors to Academy educational and leadership
activities must disclose all financial relationships (defined below)
to the Academy annually. The ACCME requires the Academy to
disclose the following to participants prior to the activity:
• All financial relationships with Commercial Companies
that contributors and their immediate family have had
within the previous 12 months. A commercial company is
any entity producing, marketing, re-selling or distributing
health care goods or services consumed by, or used on,
patients.
• Meeting presenters, authors, contributors or reviewers
who report they have no known financial relationships to
disclose.
The Academy will request disclosure information from meeting presenters, authors, contributors or reviewers, committee
members, Board of Trustees, and others involved in Academy
leadership activities (“Contributors”) annually. Disclosure information will be kept on file and used during the calendar year in
which it was collected for all Academy activities. Updates to the
disclosure information file should be made whenever there is a
change. At the time of submission of a Journal article or materials for an educational activity or nomination to a leadership
position, each Contributor should specifically review his/her
statement on file and notify the Academy of any changes to his/
her financial disclosures. These requirements apply to relationships that are in place at the time of or were in place 12 months
preceding the presentation, publication submission, or nomination to a leadership position. Any financial relationship that may
constitute a conflict of interest will be resolved prior to the delivery of the activity.
Financial Relationship Disclosure
For purposes of this disclosure, a known financial relationship
is defined as any financial gain or expectancy of financial gain
brought to the Contributor or the Contributor’s immediate family (defined as spouse, domestic partner, parent, child or spouse
of child, or sibling or spouse of sibling of the Contributor) by:
• Direct or indirect compensation;
• Ownership of stock in the producing company;
• Stock options and/or warrants in the producing company,
even if they have not been exercised or they are not currently exercisable;
• Financial support or funding to the investigator, including
research support from government agencies (e.g., NIH),
device manufacturers, and/or pharmaceutical companies;
or
• Involvement with any for-profit corporation that is likely
to become involved in activities directly impacting the
Academy where the Contributor or the Contributor’s family is a director or recipient
Description of Financial Interests
Category Code Description
Consultant / Advisor
C
Consultant fee, paid advisory
boards or fees for attending a
meeting
Employee E
Employed by a commercial
company
Lecture Fees L
Lecture and speakers bureau
fees (honoraria), travel fees or
reimbursements when speaking
at the invitation of a commercial company
Equity Owner
O
Equity ownership/stock options
(publicly or privately traded
firms, excluding mutual funds).
Patents / Royalty
P
Patents and/or royalties that
might be viewed as creating a
potential conflict of interest
Grant Support
S
Grant support from all sources
48
2015 Subspecialty Day | Neuro-Ophthalmology
Faculty Financial Disclosures
Anne S Abel MD
Joseph L Demer MD PhD
Andrew G Lee MD
None
US Public Health Service, National Eye
Institute: S
CredentialProtection: O
Kathleen B Digre MD
None
Eli Lilly & Company: S
Neuro-ophthalmix: E,P
Marc J Dinkin MD
Leah Levi MD
Alphabiocom: C
None
Mays A El-Dairi MD
Grant T Liu MD
Prana Pharmaceuticals: C
None
AbbVie: C
Alnaylam: C
Apsara Therapeutics: C,O
Aquaporumab: P
Chugai Pharmaceuticals: C
EMD Serono: C
Genzyme: C
Mallinckrodt: S,C
MedImmune: C
Novartis Pharmaceuticals
Corporation: S,C
Teva Neuroscience: C
Rod Foroozan MD
Collin M McClelland MD
Lundbeck: C
None
Courtney E Francis MD
Neil R Miller MD
None
None
Todd A Goodglick MD
Heather Moss MD PhD
None
M Tariq Bhatti MD
None
Clearview Healthcare Partners: C
Eli Lily & Company: S
H Lundbeck A/S: C
Illinois Society for Prevention of
Blindness: S
National Eye Institute: S
Research to Prevent Blindness: S
Guy V Jirawuthiworavong MD
Nancy J Newman MD
None
Gensight: C
Santhera: C
Trius/Cubist: C
Marie D Acierno MD
None
Anthony C Arnold MD
None
Rudrani Banik MD
National Eye Institute: S
Jeffrey Bennett MD PhD
Novartis Pharmaceuticals Corp.: C,L
Valerie Biousse MD
GenSight: C
Gabrielle R Bonhomme MD
Lynn K Gordon MD PhD
None
Gena Heidary MD
Randy H Kardon MD PhD
None
Department of Defense TATRC: S
Medface: O
National Eye Institute: S
Novartis Pharmaceuticals Corp.: C
Veterans Administration: S
John J Chen MD
Bradley J Katz MD
None
Axon Optics, LLC: O
Pfizer, Inc.: O
None
Michael C Brodsky MD
Sophia Mihe Chung MD
Eli Lilly & Company: S
Fiona E Costello MD
None
Sarita B Dave MD
New York State Department of
Health: S
Lanning B Kline MD
None
Gregory S Kosmorsky DO
None
Jacqueline A Leavitt MD
None
Michael S Lee MD
Jeffrey G Odel MD
Bayer Healthcare Pharmaceuticals: C
Regeneron Pharmaceuticals: C
Paul H Phillips MD
None
Stacy L Pineles MD
None
John Pula MD
Questor: C
Jacinthe Rouleau MD
Allergan: C
Janet C Rucker MD
None
Disclosures current as of 10/9/2015
Check the online program/mobile meeting guide for the most current financial disclosures
2015 Subspecialty Day | Neuro-Ophthalmology
Alfredo A Sadun MD PhD
Edison Pharmaceutical: S
Stealth Peptides: S
Peter J Savino MD
None
Harold E Shaw Jr MD
Abbott Laboratories: O
AbbVie: O
Bristol Myers Squibb: O
Johnson & Johnson Consumer &
Personal Products Worldwide: O
Novartis Pharmaceuticals Corp.: O
Pfizer, Inc.: O
S Tonya Stefko MD
None
Mitchell B Strominger MD
None
Prem S Subramanian MD PhD
NASA: C
National Eye Institute: S
Novartis Pharmaceuticals Corp.: S
US Department of Defense: S
Gregory P Van Stavern MD
None
Nicholas J Volpe MD
None
Subspecialty Day
Advisory Committee
William F Mieler MD
Genentech: C
Donald L Budenz MD MPH
Alcon Laboratories, Inc.: C
Ivantis: C
Faculty Financial Disclosures
Jonathan B Rubenstein MD
Alcon Laboratories, Inc.: C
Bausch + Lomb: C,L
R Michael Siatkowski MD
National Eye Institute: S
Nicholas J Volpe MD
None
Daniel S Durrie MD
Abbott Medical Optics: S
AcuFocus, Inc.: C,L,O,S
Alcon Laboratories, Inc.: L,O,S
Allergan: S
Alphaeon: C,O
Avedro: L,O,S
Strathspey Crown LLC: C,L,O
Wavetec: C,O,P
Francis S Mah MD
Abbott Medical Optics Inc.: L
Alcon Laboratories, Inc.: C,S
Allergan: C,L
Bausch + Lomb: C,L
CoDa: C
ForeSight: C
Imprimis: C
Ocular Therapeutix: C,S
Omeros: C
PolyActiva: C
Shire: C
TearLab: C
Kimberly M Winges MD
None
Disclosures current as of 10/9/2015
Check the online program/mobile meeting guide for the most current financial disclosures
AAO Staff
Christa Fernandez
None
Ann L’Estrange
None
Melanie Rafaty
None
Debra Rosencrance
None
Beth Wilson
None
49
50
Presenter Index
Acierno, Marie D 2, 28
Bennett*, Jeffrey 6, 31
Bhatti*, M Tariq 23, 43
Bonhomme, Gabrielle R 14, 36
Chen, John J 6, 31
Costello, Fiona E 26, 44
Demer*, Joseph L 17
Dinkin*, Marc J 22, 39
Foroozan*, Rod 11, 34
Francis, Courtney E 15, 38
Goodglick, Todd A 26, 45
Gordon, Lynn K 8, 12
Heidary, Gena 7, 32
Jirawuthiworavong, Guy V 2, 29
Katz*, Bradley J 14, 35
Moss*, Heather 24, 44
Phillips, Paul H 15, 37
Pineles, Stacy L 22, 41
Pula*, John 10, 33
Rouleau*, Jacinthe 3, 30
Rucker, Janet C 16, 38
Sadun*, Alfredo A 4
Shaw*, Harold E 25
Stefko, S Tonya 10, 33
Van Stavern, Gregory P 23, 42
Winges, Kimberly M 21, 39
* Indicates that the presenter has financial interest.
No asterisk indicates that the presenter has no financial interest.
2015 Subspecialty Day | Neuro-Ophthalmology